SM Review Questions Flashcards

1
Q

Which of the following locations on the body is appropriate to prescribe high potency steroids?

A. Face
B. Groin
C. Chest
D. Hands

A

D. Hands

High potency steroid use is appropriate on areas of the body where the skin tends to be thicker in nature. Thicker skin includes areas such as the hands and feet. As a general rule, high potency steroid use is almost always avoided in areas such as the face and groin. There are however occasional exceptions with conditions such as the lichen skin disorders.

How well did you know this?
1
Not at all
2
3
4
5
Perfectly
2
Q

Which of the following topical agents is the most easily absorbed?

A. Creams
B. Lotions
C. Ointments
D. Solutions

A

A. Creams

Creams, while not the most potent formulation of topical steroids, are the most easily absorbed into the skin. This is due to their high water content and their ability to be cover large areas of skin without drying out.

How well did you know this?
1
Not at all
2
3
4
5
Perfectly
3
Q

A 19-year-old patient was diagnosed with atopic dermatitis at today’s visit. When picking a topical steroid to prescribe, the nurse practitioner understands which of the following is the most important factor to consider?

A. Age of the patient
B. Level of absorption
C. Potency
D. Size of the rash

A

C. Potency

The most important factor to consider when prescribing topical steroids is the level of potency. Class 1 are the most potent, which should never be prescribed in primary care. Primary care most commonly prescribes topical steroids in classes 3 and 4. Age of the patient, level of absorption, and the size of the rash are all factors to consider, but the potency and the area of the body where the patient will be applying the topical steroid are the most important factors to consider.

How well did you know this?
1
Not at all
2
3
4
5
Perfectly
4
Q

An 8-year-old child presents to the clinic with a pruritic lesion near their nose. Upon assessment, the nurse practitioner notes golden drainages crusted on. What is an important education point recording the treatment of this condition?

A. “Follow up is needed if there is not improvement in the next few days”
B. “It is important to start this antiviral as soon as possible”
C. “These lesions are not likely to be spread”
D. “You need to stay hydrated to protect the kidneys”

A

A. “Follow up is needed if there is not improvement in the next few days”

This child is presenting with non-bullous impetigo as seen by the characteristic honey-crusted lesion. Non-bullous impetigo is extremely contagious. Mupirocin, also known as Bactroban, is the typical first line treatment and patients should notice improvement within a few days. If there is no improvement or there is worsening of symptoms, the patient and family need to be instructed to follow up. This condition is not viral in nature and does not warrant antivirals. However, when patients are taking antivirals for something like herpes simplex, it is important to start them as soon as possible for maximum efficacy, as well as stay very hydrated as they can be harsh on the kidneys.

How well did you know this?
1
Not at all
2
3
4
5
Perfectly
5
Q

An older adult with a past medical history of type II diabetes mellitus and psoriasis is diagnosed with onychomycosis. What is the preferred treatment?

A. Mebendazole (Emverm)
B. Mupirocin (Bactroban)
C. Selenium sulfide (Selsun Blue)
D. Terbinafine (Lamisil)

A

D. Terbinafine (Lamisil)

Onychomycosis is a fungal infection of the nail bed and treated with an antifungal, most commonly terbinafine (Lamisil). Mebendazole (Enverm) is used for parasitic infections such as pinworms and mupirocin is used for bacterial conditions like impetigo. Selenium sulfide (Selsun Blue) is another antifungal. However, this is typically in a shampoo formulation and used for things like tinea capitis, dandruff, or tinea versicolor.

How well did you know this?
1
Not at all
2
3
4
5
Perfectly
6
Q

After performing a scotch tape test, the nurse practitioner diagnoses a thirteen-year-old patient with enterobiasis. How should the NP proceed with treatment?

A. Prescribe Mebendazole (Emverm) for the patient only
B. Prescribe Mebendazole (Emverm) for the patient and her entire immediate family
C. Prescribe Permethrin (Nix) for the patient and her entire immediate family
D. Educate the patient’s mother that this will likely resolve on its own without treatment

A

B. Prescribe Mebendazole (Emverm) for the patient and her entire immediate family

Enterobiasis, also known as pinworm infection, is treated with anthelmintics such as mebendazole and albendazole. All household contacts in addition to caregivers should be treated alongside the child due to the high occurrence of reinfection. Daily showers in addition to laundering help also to reduce the risk of reinfection.

How well did you know this?
1
Not at all
2
3
4
5
Perfectly
7
Q

What is the first line treatment for acne?

A. Benzoyl peroxide (Benzagel)
B. Tretinoin (Retin-A)
C. Isotretinoin (Accutane)
D. Topical antibiotic creams

A

A. Benzoyl peroxide (Benzagel)

The correct answer is benzoyl peroxide (Benzagel). This medication works for mild to moderate acne and can be used in conjunction with additional acne treatments. This medication works by drying and peeling skin on the face so that bacteria is shed off. Tretinoin, isotretinoin, and topical antibiotic creams are used after benzoyl peroxide treatment has failed.

How well did you know this?
1
Not at all
2
3
4
5
Perfectly
8
Q

What is the mechanism of action of tretinoin (Retin-A)?

A. Acts as an anti-inflammatory and decreases redness
B. Clogs pores with medication, decreasing risk of acne
C. Dissolves old, dead skin, unclogs pores, and allows skin to heal
D. Stops bacterial overgrowth on the skin

A

C. Dissolves old, dead skin, unclogs pores, and allows skin to heal

The correct answer that it dissolves old, dead skin, unclogs pores, and allows skin to heal. This medication works by actually irritating the skin and dissolving the old, dead skin, which unclogs pores and allows the skin to heal itself.

How well did you know this?
1
Not at all
2
3
4
5
Perfectly
9
Q

A 21-year-old male patient presents to the clinic reporting acne treatment failure. The patient reports using both benzoyl peroxide (Benzagel) and tretinoin (Retin-A) and has had no relief of symptoms. Which of the following options would the nurse practitioner consider next?

A. Refer to dermatology
B. Prescribe isotretinoin (Accutane)
C. Prescribe a tetracycline
D. Prescribe the patient topical salicylic acid

A

C. Prescribe a tetracycline

The correct answer is to prescribe a tetracycline. At this time, prescribing an antibiotic would be most ideal to start as the next step of treatment. If this does not work, we would consider referring this patient to dermatology. Isotretinoin (Accutane) is only prescribed by dermatology, not primary care. Salicylic acid can be obtained over the counter and should be tried before benzoyl peroxide (Benzagel) is initiated.

How well did you know this?
1
Not at all
2
3
4
5
Perfectly
10
Q

A 32-year-old female patient with a history of seizures presents to the clinic to follow up on her newly initiated medication, phenytoin (Dilantin). She has been taking this for the past month. Side effects have been minimal and the medication seems to be working well. What education would the nurse practitioner want to reinforce at this visit?

A. Thyroid function needs to be monitored frequently
B. This medication can increase the risk of bleeding
C. The patient may experience possible hair loss with this medication
D. Decreased efficacy of birth control with this medication

A

D. Decreased efficacy of birth control with this medication

The correct answer is decreased efficacy of birth control with this medication. Phenytoin (Dilantin) can decrease the overall efficacy of birth control in general, and for women of childbearing age, this would be a very important education point. Monitoring for this medication would include liver function rather than thyroid function. As well, hirsutism, or increased hair growth, would be a side effect of this medication, not hair loss. There is no increased risk for bleeding with phenytoin (Dilantin) use.

How well did you know this?
1
Not at all
2
3
4
5
Perfectly
11
Q

Which anticonvulsant medication is also used as a treatment for trigeminal neuralgia?

A. Phenytoin (Dilantin)
B. Valproic acid (Depakene)
C. Carbamazepine (Tegretol)
D. Gabapentin (Neurontin)

A

C. Carbamazepine (Tegretol)

The correct answer is carbamazepine (Tegretol). Carbamazepine (Tegretol) is an anticonvulsant that also reduces pain signals to the brain, making it a great option for treating trigeminal neuralgia. Valproic acid (Depakene) is used in treatment for bipolar disorders as well as seizures. Gabapentin (Neurontin) can be used for post-herpetic neuralgia following shingles or neuropathy. Phenytoin (Dilantin) is mostly used for seizures or status epilepticus.

How well did you know this?
1
Not at all
2
3
4
5
Perfectly
12
Q

The mother of a 6-year-old has voiced concerns to the nurse practitioner about her daughter’s frequent daydreaming episodes. The mother states she seems not to hear her sometimes and seems to stare blankly for 10-15 second segments. What is the most appropriate treatment?

A. Valproic acid (Depakote)
B. Neurontin (Gabapentin)
C. Carbamazepine (Tegretol)
D. Amitriptyline (Elavil)

A

A. Valproic acid (Depakote)

Absence seizures are commonly described as if they are daydreaming or staring off into space. Appropriate treatment for an absence seizure consists of valproic acid (Depakote) or Dilantin, also known as phenytoin. The majority of children will outgrow these, but in the meantime treatment with an anti-seizure medication is key. It would additionally be appropriate to refer this child to neurology for follow-up.

How well did you know this?
1
Not at all
2
3
4
5
Perfectly
13
Q

A 44-year-old male patient comes into the clinic today with symptoms of depression. He says he has “a lot of down days”, is not sleeping well, and has lost interest in many aspects of his life. He has a history of migraines and is unsure about starting a medication because he does not want them to get worse. What medication would be the most beneficial for this patient?

A. Sertraline (Zoloft)
B. Topiramate (Topamax)
C. Amitriptyline (Elavil)
D. Gabapentin (Neurontin)

A

C. Amitriptyline (Elavil)

The correct answer is amitriptyline (Elavil). Tricyclic antidepressants, such as amitriptyline (Elavil), can be used in treatment for both migraines and depression. Sertraline (Zoloft) is a great option for the treatment of depression and anxiety, but it can increase the risk of serotonin syndrome when taken in conjunction with migraine medications like sumatriptan (Imitrex). Topamax is a treatment for migraine headaches, but would not help with depression. Gabapentin is given for many conditions, but typically used for seizures or neuropathy, and has no role in the treatment of depression and anxiety.

How well did you know this?
1
Not at all
2
3
4
5
Perfectly
14
Q

Topiramate (Topamax) is an anticonvulsant that can be used for seizure prevention. What other condition can this medication help prevent?

A. Neuropathy
B. Neuralgia
C. Migraine headaches
D. Cluster headaches

A

C. Migraine headaches

The correct answer is migraine headaches. Topiramate (Topamax) can be given for prophylaxis of migraine headaches as well as seizures. It should be avoided in patients with renal or liver disease. It should be used with caution in patients with eating disorders due to its added effect of weight loss. Topiramate (Topamax) has no beneficial use in neuropathy, neuralgia, or cluster headaches.

How well did you know this?
1
Not at all
2
3
4
5
Perfectly
15
Q

A 45-year-old man with a history of COPD has concerns of multiple episodes of a severe headache that radiates from the left side of his face, behind his eye. He has clear nasal drainage that accompanies the pain. The episodes appear spontaneously throughout the day and last about an hour. What is an appropriate treatment during an episode?

A. Administer high-dose oxygen
B. Assess any possible triggers
C. Administer sumatriptan (Imitrex)
D. Dim the lights and limit screen time during episodes

A

C. Administer sumatriptan (Imitrex)

Cluster headache treatment consists of prophylactic treatment with calcium channel blockers such as verapamil (Verelan). After the cluster headache has begun, we can give abortive treatment which consists of an injection of a drug such as sumatriptan (Imitrex) in addition to high flow oxygen 100% at 7-15 unless they have COPD. We can also sometimes see something like intranasal lidocaine used as needed for cluster headaches.

How well did you know this?
1
Not at all
2
3
4
5
Perfectly
16
Q

A 75-year-old female was diagnosed with Parkinson’s disease approximately one year ago. Her symptoms have worsened, and they are interfering with her ability to perform activities of daily living (ADL’s). The nurse practitioner decides to start her on carbidopa/levodopa (Sinemet). Which of the following symptoms of Parkinson’s disease is improved the most by this medication?

A. Tremors
B. Bradykinesia
C. Rigidity
D. Memory

A

B. Bradykinesia

The correct answer is bradykinesia. Carbidopa/levodopa (Sinemet) has the greatest effect in improving bradykinesia associated with Parkinson’s Disease. It has less of an effect on the other symptoms such as tremors, rigidity, and has no overall effect on memory. Given that bradykinesia is the most debilitating symptom of Parkinson’s disease, this medication is an excellent treatment option.

How well did you know this?
1
Not at all
2
3
4
5
Perfectly
17
Q

A 75-year-old patient with a history of Parkinson’s disease was recently started on medication to help with his slowed pace and consistent halts in his movement. He states since starting the new medication he has had issues with dizziness, blurred vision, and has fallen a few times. What should be the priority assessment by the nurse practitioner?

A. Check the patient’s blood pressure
B. Ask the patient “when is the last time he had anything to eat”
C. Assess the patient’s gait
D. Check the patient’s pulse

A

A. Check the patient’s blood pressure

Levodopa-carbidopa (Sinemet) and dopamine agonists like ropinirole (Requip) are common Parkinson’s disease medications. It is possible to see concerns with hypotension in patients taking these drugs, so that is something we want to be sure to warn our patients about and assess if symptoms arise. The priority action would be to check this patient’s blood pressure as hypotension is a known side effect and can lead to falls in the older population.

How well did you know this?
1
Not at all
2
3
4
5
Perfectly
18
Q

A 62-year-old patient with a new diagnosis of Parkinson’s disease has concerns of leg edema that has become worse over the past week. She was recently started on a new medication but does not remember the name. She has no pain or redness on the leg. What medication could be a possible cause of her symptoms?

A. Ropinirole (Requip)
B. Levodopa-carbidopa (Sinemet)
C. Monoamine oxidase B (MAO B)
D. Benztropine (Cogentin

A

A. Ropinirole (Requip)

Ropinirole (Requip) can cause issues with both impulse control and leg edema after starting it. In addition, like other medications for Parkinson’s disease such as levodopa-carbidopa (Sinemet), it can be common to see issues with hypotension.

How well did you know this?
1
Not at all
2
3
4
5
Perfectly
19
Q

A 70-year-old male patient comes into the clinic stating he is having episodes of dizziness. He only notices the dizziness when he changes positions, like when standing up from the bed or chair. The patient describes the sensation “like the room is spinning around him” and he reports nausea during episodes. The nurse practitioner diagnoses him with BPPV, or benign paroxysmal positional vertigo, and wants to prescribe meclizine (Antivert). Which of the following would not be a benefit of this medication?

A. Mild anticholinergic effects
B. Begins working quickly
C. Increases gastrointestinal motility
D. Long-lasting benefits

A

C. Increases gastrointestinal motility

The correct answer is increases gastrointestinal motility. Meclizine (Antivert) has many benefits including its mild anticholinergic effects, which makes it a good option for elderly patients. It has a quick onset of action and a long half-life, giving patients quick and lasting relief from BPPV. Meclizine (Antivert) does not cause increased gastrointestinal motility, and this side effect would be associated with other medications such as metoclopramide (Reglan).

How well did you know this?
1
Not at all
2
3
4
5
Perfectly
20
Q

A 70-year-old female patient with a history of diabetes comes in today with complaints of tingling and numbness in her fingers and a red tongue. The only medication she takes is metformin (Glucophage) for diabetes. What prescription does the nurse practitioner anticipate prescribing?

A. Cobalamin (Vitamin B12) supplement
B. Ferrous sulfate (Iron) supplement
C. Folate (Folic acid)
D. Cholecalciferol (Vitamin D3)

A

A. Cobalamin (Vitamin B12) supplement

Cobalamin (Vitamin B12) deficiency is a macrocytic anemia associated with neurological symptoms as well as a beefy red tongue. This anemia can be a side effect of taking metformin (Glucophage) long term. If patients are experiencing this, it is appropriate to initiate Vitamin B12 supplementation.

How well did you know this?
1
Not at all
2
3
4
5
Perfectly
21
Q

Which of the following types of iron tablets is best absorbed by the body?

A. Ferrous citrate
B. Ferrous fumarate
C. Ferrous gluconate
D. Ferrous sulfate

A

D. Ferrous sulfate

There are many different types of iron supplements available to patients. Ferrous sulfate is best absorbed by the body and is the preferred supplement for this reason. Other ways to enhance absorption include taking the tablet on an empty stomach if possible or to take with an acidic drink such as orange juice, as it is best absorbed in an acidic environment.

How well did you know this?
1
Not at all
2
3
4
5
Perfectly
22
Q

A 34-year-old male patient presents to the clinic with concerns about his depression medications. He has tried medications in different classes, but still has not had relief of his symptoms. He is prescribed a new medication, and, as part of the education, the nurse practitioner tells him to avoid eating aged meats, cheeses, and fermented foods. The new medication is most likely in which of the following medication classes?

A. Selective serotonin reuptake inhibitors (SSRIs)
B. Tricyclic antidepressants (TCAs)
C. Monoamine oxidase inhibitors (MAOIs)
D. Selective norepinephrine reuptake inhibitors (SNRIs)

A

C. Monoamine oxidase inhibitors (MAOIs)

The correct answer is monoamine oxidase inhibitors (MAOIs). Tyramine is an amino acid that occurs naturally in the body and helps regulate blood pressure. Monoamine oxidase is an enzyme that helps break down tyramine. When a patient takes a MAOI, monoamine oxidase is blocked, and therefore tyramine levels increase. This lack of breaking down tyramine can cause the patient to experience dangerous spikes in blood pressure. So, patients who take these medications should avoid eating foods that are high in tyramine such as aged meats, cheeses, or fermented foods like beer and kombucha. Patients who are on SSRIs, TCAs, or SNRIs do not need to avoid foods high in tyramine because these medications have no effect on monoamine oxidase.

How well did you know this?
1
Not at all
2
3
4
5
Perfectly
23
Q

A 18-year-old female patient came into the clinic today accompanied by her mother. They both report that she is struggling with symptoms of depression including sleep disturbance, lack of interest in activities, and weight gain. The nurse practitioner wants to start the patient on fluoxetine (Prozac). What should be assessed first prior to starting this medication?

A. Height/weight
B. Suicide risk
C. Kidney function
D. Liver function

A

B. Suicide risk

The correct answer is suicide risk. Antidepressant medications carry a black box warning of increased risk of suicidal ideation with initiation of treatment and dose changes. It is important to assess for signs of suicide risk before starting any antidepressant medication. While there may be weight gain associated with this drug class, the need to monitor height and weight are not as high of priority when compared to suicide risk. There is no need to check kidney and liver function prior to starting SSRIs.

How well did you know this?
1
Not at all
2
3
4
5
Perfectly
24
Q

A patient who has been treated with duloxetine (Cymbalta) for the past 3 months states they are starting to feel much better and they are hoping to stop the medication. How should the nurse practitioner proceed?

A. Allow the patient to stop the medication right away
B. Educate the patient that they will need to take this medication permanently
C. Let the patient know they can taper off the medication after another 6 months of improved symptoms
D. Educate the patient the medication has to be taken for a full year for the maximum benefit.

A

C. Let the patient know they can taper off the medication after another 6 months of improved symptoms

The correct answer is let the patient know they can taper off the medication after another 6 months of improved symptoms. Antidepressant medication should be taken until the patient is feeling better and then for an additional 6 months. After this time period, the patient can attempt to taper off the medication under supervision of their healthcare provider to make sure their symptoms are not worsening. The maximum effect of these medications is reached at about 4-6 weeks and they should not be stopped abruptly.

How well did you know this?
1
Not at all
2
3
4
5
Perfectly
25
Q

A patient presents to the office with a chief complaint of new onset anxiety. The anxiety started about 1 month ago and began when the patient started a new job. Despite several attempts at anxiety control on their own, they are unable to fully manage the anxiety at work. They are requesting medication to help with their anxiety, but they would like to avoid any controlled substances. What would be the best choice of prescription for this patient?

A. Buspirone (Buspar)
B. Alprazolam (Xanax)
C. Paroxetine (Paxil)
D. Fluoxetine (Prozac)

A

A. Buspirone (Buspar)

The correct answer is buspirone (Buspar). Buspirone (Buspar) is a non-benzodiazepine medication used to treat anxiety. Alprazolam (Xanax) is a benzodiazepine, and therefore a controlled substance. This patient requests medication that is not a controlled substance. Paroxetine (Paxil) and fluoxetine (Prozac) are both SSRI’s. While SSRIs are not controlled substances, they are best used for chronic anxiety disorders, not short-term anxiety.

How well did you know this?
1
Not at all
2
3
4
5
Perfectly
26
Q

During a wellness exam, a patient asks the nurse practitioner if there is a medication that could be prescribed for their upcoming magnetic resonance imaging (MRI) scan. The patient reports that they have had an MRI scan in the past, and they experienced severe nervousness, shaking, difficulty breathing, tachycardia, and claustrophobia. What is the best medication to prescribe for acute anxiety?

A. Escitalopram (Lexapro)
B. Buspirone (Buspar)
C. Clonazepam (Klonopin)
D. Haloperidol (Haldol)

A

C. Clonazepam (Klonopin)

The correct answer is clonazepam (Klonopin). Benzodiazepines, like clonazepam (Klonopin), are the most helpful for acute anxiety episodes. These are medications that can be used on an as-needed basis and do not require consistent use prior to the event. This makes benzodiazepines helpful for magnetic resonance imaging (MRI’s), computed tomography (CT) scans, dental work, and various other medical procedures that cause patients severe anxiety. Buspirone (Buspar) is an anti-anxiety medication that requires several weeks of consistent use for best results. Selective serotonin reuptake inhibitors like escitalopram (Lexapro) are best for treatment of long-term anxiety. Haloperidol (Haldol) is an antipsychotic medication and typically used for schizophrenia.

How well did you know this?
1
Not at all
2
3
4
5
Perfectly
27
Q

On the Generalized Anxiety Disorder 7-item (GAD-7) screening tool, a patient has progressively reported higher scores over the last 2 years. Past scores were between 6 and 11. The patient reports that their anxiety has been an issue in the past, but that it has gotten worse lately and they would like medication to help. The GAD-7 score today is 13, increased from 11. The nurse practitioner knows that which of the following medications helps best with long-term anxiety?

A. Lorazepam (Ativan)
B. Buspirone (Buspar)
C. Sertraline (Zoloft)
D. Doxepin (Prudoxin)

A

C. Sertraline (Zoloft)

The correct answer is sertraline (Zoloft). Sertraline (Zoloft) is a part of the selective serotonin reuptake inhibitor (SSRI) medication class. SSRIs can be helpful in the management of long-term anxiety disorders. Lorazepam (Ativan) is a benzodiazepine, which is helpful for acute anxiety episodes. Buspirone (Buspar) is helpful for short-term anxiety, and works best when taken daily over the course of several weeks. Doxepin (Prudoxin) is a tricyclic antidepressant and not a first-line treatment for anxiety treatment.

How well did you know this?
1
Not at all
2
3
4
5
Perfectly
28
Q

A 32-year-old female patient presents to the clinic reporting a positive pregnancy test. This patient has a history of bipolar disorder and is currently taking lithium carbonate (Priadel). The nurse practitioner knows that lithium carbonate (Priadel) is what pregnancy category medication?

A. Pregnancy category A
B. Pregnancy category B
C. Pregnancy category C
D. Pregnancy category X

A

D. Pregnancy category X

The correct answer is pregnancy category X. This medication is known to cause teratogenic effects to the fetus and should not be taken during pregnancy. The NP should refer this patient to both psych and her obstetrician for the proper management of her bipolar disorder during her pregnancy.

How well did you know this?
1
Not at all
2
3
4
5
Perfectly
29
Q

Which of the following patients should the nurse practitioner consider prescribing quetiapine (Seroquel) for their atypical depression?

A. A 26-year-old female with an hemoglobin A1C of 8.6%
B. A 30-year-old male patient with a history of gout
C. A 32-year-old female patient with a BMI of 36
D. A 29-year-old male patient with triglyceride levels of 220.

A

B. A 30-year-old male patient with a history of gout

The correct answer is the 30-year old male with a history of gout. Atypical antidepressants, like quetiapine (Seroquel) can cause weight gain, an increased blood glucose level, and hyperlipidemia, making them not ideal choices for the other patient options, as it may worsen their comorbidities.

How well did you know this?
1
Not at all
2
3
4
5
Perfectly
30
Q

A 54-year-old patient presents to the office after three months of lifestyle modifications due to his recent diagnosis of stage 1 hypertension. The patient has a history of hyperlipidemia and diabetes. The nurse practitioner knows that thiazide diuretics increase all of the following lab levels except:

A. Glucose
B. Triglycerides
C. Uric acid
D. Potassium

A

D. Potassium

The correct answer is potassium. Thiazide diuretics can increase blood glucose levels, triglycerides, and uric acid levels. A good way to remember this is thinking of (HCTz) H-hyperglycemia, C-crystals for uric acid, T-triglycerides. Thiazide diuretics can rarely cause low potassium levels. This is a negative stem question, asking “except.”

How well did you know this?
1
Not at all
2
3
4
5
Perfectly
31
Q

An ACE inhibitor, such as lisinopril (Zestril), should be discontinued in which of the following patients?

A. A patient in stage 1 chronic kidney disease
B. A patient with a GFR of 29
C. A patient with increased liver function tests
D. A patient with a new diagnosis of heart failure

A

B. A patient with a GFR of 29

The correct answer is a patient with a GFR of 29. With ACE inhibitor medications, like lisinopril (Zestril), it is important to monitor kidney function. These medications are renal protective in the early stages of kidney disease, but, as it progresses, they can be harmful. A GFR of 29 would be considered the fifth stage of chronic kidney disease, so we would need to put them on a different medication. ACE inhibitors are often used in the treatment for heart failure and do not affect liver function.

How well did you know this?
1
Not at all
2
3
4
5
Perfectly
32
Q

A 60-year-old female patient has just been diagnosed with heart failure. She has been noticing swelling in her ankles lately, despite taking her diuretics. Which of the following medications may be responsible for this symptom?

A. Verapamil (Verelan)
B. Bumetanide (Bumex)
C. Hydrochlorothiazide (HCTZ)
D. Propranolol (Inderal)

A

A. Verapamil (Verelan)

The correct answer is verapamil (Verelan). Non-dihydropyridine calcium channel blockers like verapamil (Verelan) may have the side effect of ankle edema and should not be given to patients with a history of heart failure. Hydrochlorothiazide (HCTZ) and Bumetanide (Bumex) would help with peripheral edema. Propranolol (Inderal) does not have the side effect of ankle swelling.

How well did you know this?
1
Not at all
2
3
4
5
Perfectly
33
Q

A 47-year-old male patient presents to the clinic today after recently being treated for a heart failure exacerbation. He states that his chest feels more enlarged than normal and there appears to be an increase in the amount of breast tissue upon physical exam. The nurse practitioner is aware that which of the following medications can lead to this side effect?

A. Enalapril (Vasotec)
B. Spironolactone (Aldactone)
C. Furosemide (Lasix)
D. Bumetanide (Bumex)

A

B. Spironolactone (Aldactone)

The correct answer is spironolactone (Aldactone). A known side effect of spironolactone use in men is the potential for gynecomastia. Gynecomastia occurs because this medication reduces production of testosterone and displaces androgen. Usually cessation of spironolactone resolves this particular side effect.

How well did you know this?
1
Not at all
2
3
4
5
Perfectly
34
Q

Your 63-year-old male patient was recently diagnosed with heart failure. When doing a medication reconciliation, which medication would be most appropriate to discontinue?

A. Enalapril (Vasotec)
B. Acetaminophen (Tylenol)
C. Sertraline (Zoloft)
D. Diclofenac (Voltaren)

A

D. Diclofenac (Voltaren)

The correct answer is diclofenac (Voltaren). Non-steroidal anti-inflammatory drugs (NSAIDs) are known to cause potential fluid retention and edema. Heart failure patients commonly have issues with excess fluid volume, so taking NSAIDs increases their risk for a potential exacerbation.

How well did you know this?
1
Not at all
2
3
4
5
Perfectly
35
Q

A middle aged female patient presents to the clinic with increasing cough and peripheral edema. Based on her past medical history of heart failure, which of the following interventions would be least appropriate to order at this time?

A. Obtain a chest x-ray
B. Initiate furosemide (Lasix)
C. Initiate digoxin (Lanoxin)
D. Assess the patient’s daily weight log

A

C. Initiate digoxin (Lanoxin)

The correct answer is to initiate digoxin (Lanoxin). Based on the patient’s presentation, it is likely that she is currently experiencing a heart failure exacerbation. A chest x-ray is appropriate given the patient’s cough and likely pulmonary congestion. Furosemide is appropriate to decrease the fluid volume overload. Assessing the patient’s daily weight log can also be a good indicator of overall fluid status. While digoxin can be administered to heart failure patients, it is not indicated at this time.

How well did you know this?
1
Not at all
2
3
4
5
Perfectly
36
Q

In which of the following patients would prescribing nitroglycerin require further intervention by the nurse practitioner?

A. A patient taking tadalafil (Cialis)
B. A patient taking doxazosin (Cardura)
C. A patient taking finasteride (Proscar)
D. A patient taking valacyclovir (Valtrex)

A

A. A patient taking tadalafil (Cialis)

Nitrates like nitroglycerin are potent vasodilators used for angina. However, when patients are on PDE-5 inhibitors like tadalafil (Cialis) there is the high potential for the blood pressure to drop to dangerous ranges and possibly kill the patient. Therefore, taking nitrates while on any PDE-5 inhibitors is absolutely contraindicated.

How well did you know this?
1
Not at all
2
3
4
5
Perfectly
37
Q

The nurse practitioner is caring for a patient who was recently prescribed nitroglycerin in the ER after being diagnosed with stable angina. He is asking if he should call 911 or take the nitroglycerin if he experiences chest pain again. The nurse practitioner should tell him:

A. “As soon as you feel any chest pain, you should call 911 immediately”
B. “If your chest pain hasn’t resolved within 5 minutes after the second dose of you taking your nitroglycerin tablet, call 911”
C. “If your chest pain continues 10 minutes past your first dose of nitroglycerin, call 911”
D. “Take 3 tablets every 5 minutes then wait 30 minutes to see if the pain subsides before calling 911”

A

B. “If your chest pain hasn’t resolved within 5 minutes after the second dose of you taking your nitroglycerin tablet, call 911”

Patients who are taking nitrates need to be instructed that they can take a tablet as soon as they experience chest pain, and then every 5 minutes for up to 3 doses (or 15 minutes). If the pain hasn’t resolved within 5 minutes after the second dose however, the patient should be instructed to call 911 as they are at a high risk of a cardiovascular event.

How well did you know this?
1
Not at all
2
3
4
5
Perfectly
38
Q

In which route are nitroglycerin tablets administered?

A. Intravenous
B. Oral
C. Subcutaneous
D. Sublingual

A

D. Sublingual

Nitroglycerin tablets are administered sublingually, which allows the patient to be able to take it at home and the effects of the medication to occur rapidly. A tablet should be taken as soon as the patient experiences chest pain, and then every 5 minutes for up to 3 doses (or 15 minutes). If the pain is not resolved by the second dose, the patient should be instructed to call 911.

How well did you know this?
1
Not at all
2
3
4
5
Perfectly
39
Q

All of the following are potential complications or side effects of amiodarone (Pacerone) except:

A. Hypothyroidism
B. Optic neuropathy
C. Pulmonary toxicity
D. Bowel obstruction

A

D. Bowel obstruction

The correct answer is a bowel obstruction. Amiodarone can cause a plethora of side effects and complications across various body systems. It is known to cause thyroid dysfunction, both hypothyroidism and hyperthyroidism, due to its high iodine content. Amiodarone can lead to optic disc swelling and ultimately optic neuropathy as a result. It can also cause pulmonary toxicity due to its accumulation in the fatty tissues and its cytotoxic effects.

How well did you know this?
1
Not at all
2
3
4
5
Perfectly
40
Q

A 59-year-old male patient presents to the clinic for a routine annual exam. He was recently placed on amiodarone (Pacerone) by his cardiologist after a recent hospital stay where he experienced atrial fibrillation with a rapid ventricular response. When doing his medication reconciliation, the nurse practitioner is aware that which of the following medications on his chart may need a dose adjustment?

A. Ibuprofen (Advil)
B. Omeprazole (Prilosec)
C. Apixaban (Eliquis)
D. Amoxicillin (Amoxil)

A

C. Apixaban (Eliquis)

The correct answer is apixaban (Eliquis). Anticoagulants commonly interact with the antiarrhythmic amiodarone. When taking apixaban specifically with amiodarone, the blood levels of apixaban will likely increase. This will put the patient at a higher risk of bleeding and complications. Therefore, if the patient is taking both concurrently, it will likely be necessary to decrease the anticoagulant dosage.

How well did you know this?
1
Not at all
2
3
4
5
Perfectly
41
Q

The nurse practitioner is concerned that a patient may be experiencing signs and symptoms of digoxin (Lanoxin) toxicity. Which of the following would not be an indication of possible toxicity?

A. Confusion
B. Visual changes
C. Ventricular tachycardia
D. Constipation

A

D. Constipation

The correct answer is constipation. Neurological changes such as confusion, visual changes such as green-yellow halos, and abnormal heart rhythms (AV block, premature ventricular contractions, ventricular tachycardia, bradycardia, etc) are all possible signs of digoxin toxicity. With digoxin toxicity, it is much more likely for the patient to experience diarrhea.

How well did you know this?
1
Not at all
2
3
4
5
Perfectly
42
Q

A patient has an LDL level of 205 mg/dL and the nurse practitioner is wanting to start medication. Which of the following is an appropriate cholesterol lowering medication for this patient?

A. Atorvastatin (Lipitor)
B. Colestipol (Colestid)
C. Fenofibrate (Tricor)
D. Simvastatin (Zocor)

A

A. Atorvastatin (Lipitor)

The correct answer is atorvastatin (Lipitor). For patients with an LDL level >190 mg/dL, high intensity statins are recommended as first line treatment for decreasing LDL levels. Atorvastatin (Lipitor) and Rosuvastatin (Crestor) are examples of high intensity statins. Simvastatin (Zocor) is a moderate intensity statin, fenofibrate (Tricor) is best at lowering triglycerides, and bile acid sequestrants like Colestipol (Colestid) have many GI side effects, which make them not a first-line treatment option.

How well did you know this?
1
Not at all
2
3
4
5
Perfectly
43
Q

A 62-year-old female patient has a history of hyperlipidemia, specifically high triglyceride levels. At her last visit, she was prescribed niacin (Niacor). She says that after she takes the medication, she feels very flushed. What education would the nurse practitioner give to this patient to help with these side effects?

A. Take this medication with water only
B. Take this medication at night, before bed
C. Do not take this medication with warm liquids
D. Only take this medication every other day

A

C. Do not take this medication with warm liquids

The correct answer is “do not take this medication with warm liquids.” One of the side effects of niacin (Nicacor) is facial flushing, occurring about 30-60 minutes after taking the medication. To avoid this, we can educate the patient to take aspirin along with the niacin (Niacor) and to avoid taking the medication with warm liquids. This medication should be taken with food and should be taken daily.

How well did you know this?
1
Not at all
2
3
4
5
Perfectly
44
Q

A 64-year-old male with a history of hypertriglyceridemia has been on niacin (Niacor) for one month, but returns to the clinic wanting to start a different medication due to the amount of side effects he is experiencing. Which of the following medication options is the best at lowering triglyceride levels, and may be used in place of niacin (Niacor)?

A. Atorvastatin (Lipitor)
B. Fenofibrate (Tricor)
C. Rosuvastatin (Crestor)
D. Colestipol (Colestid)

A

B. Fenofibrate (Tricor)

The correct answer is fenofibrate (Tricor). Fenofibrate (Tricor) is a highly effective medication for lowering triglycerides. Atorvastatin (Lipitor) or rosuvastatin (Crestor) are both high intensity statins that are very effective in lowering total and LDL cholesterol levels. Colestipol (Colestid) is a bile acid sequestrant and can have a lot of GI side effects, making it not an ideal option when the patient is already experiencing a large amount of medication side effects.

How well did you know this?
1
Not at all
2
3
4
5
Perfectly
45
Q

The nurse practitioner is aware that when a patient is prescribed enoxaparin (Lovenox) long term, levels of this drug can be monitored with which of the following labs?

A. INR
B. Anti-Xa
C. Fibrinogen
D. CBC

A

B. Anti-Xa

The correct answer is anti-Xa. The gold standard for determining the concentration of enoxaparin (Lovenox) in the blood is an anti-Xa assay. INR is used to monitor warfarin (Coumadin) dosing. A CBC would show platelet counts, but not drug concentration. Fibrinogen levels help with blood clotting but also do not offer insight into drug concentration.

How well did you know this?
1
Not at all
2
3
4
5
Perfectly
46
Q

A 57-year-old female patient was recently placed on warfarin (Coumadin) due to atrial fibrillation. When checking her INR today, it is noted that the INR level is 6.9. The patient’s physical exam overall is benign, but she did note a little bleeding around her gums when brushing her teeth last night. Which of the following interventions would be most appropriate by the nurse practitioner?

A. Increase the warfarin dose, initiate PO Vitamin K
B. Decrease the warfarin dose, initiate PO Vitamin K
C. Increase the warfarin dose, refer to ED for IV Vitamin K
D. Decrease the warfarin dose, refer to ED for IV Vitamin K

A

B. Decrease the warfarin dose, initiate PO Vitamin K

The correct answer is to decrease the warfarin dose and initiate PO vitamin K. The ideal INR for a patient taking warfarin for atrial fibrillation specifically is 2-3. Therefore, this INR is elevated and the patient is at increased risk for bleeding. The patient is mildly symptomatic with bleeding gums, so PO vitamin K would be an appropriate choice. In addition, decreasing the warfarin dose is imperative to get back to a target INR range of 2-3.

How well did you know this?
1
Not at all
2
3
4
5
Perfectly
47
Q

A 61-year-old male patient presents to the clinic today with an erythematous, edematous left lower extremity. A deep vein thrombosis is confirmed via venous doppler. Which of the following medications would be the least appropriate to initiate for this diagnosis?

A. Acetylsalicylic acid (Aspirin)
B. Apixaban (Eliquis)
C. Warfarin (Coumadin)
D. Rivaroxaban (Xarelto)

A

A. Acetylsalicylic acid (Aspirin)

The correct answer is acetylsalicylic acid (Aspirin). To treat a deep vein thrombosis, an anticoagulant is necessary. Acetylsalicylic acid (Aspirin) is an antiplatelet, not an anticoagulant. Aspirin can be used to help prevent a deep vein thrombosis but it will not help to dissolve a blood clot. Apixaban (Eliquis), warfarin (Coumadin), and rivaroxaban (Xarelto) are all anticoagulants.

How well did you know this?
1
Not at all
2
3
4
5
Perfectly
48
Q

Which medication would be most appropriate to prescribe to a group A COPD patient?

A. Salmeterol (Serevent)
B. Prednisone (Rayos)
C. Roflumilast (Daliresp)
D. Tiotropium (Spiriva)

A

A. Salmeterol (Serevent)

The correct answer is salmeterol (Serevent). According to the GOLD guidelines, treatment group A COPD patients can be treated with short or long acting beta agonists. These medications are typically used for quick relief of COPD symptoms by bronchodilating the patient’s airways.

How well did you know this?
1
Not at all
2
3
4
5
Perfectly
49
Q

A COPD patient is prescribed tiotropium (Spiriva) daily. Based on this medication class, the nurse practitioner knows which of the following is the most likely side effect?

A. Diarrhea
B. Dry mouth
C. Bradycardia
D. Oral thrush

A

B. Dry mouth

The correct answer is dry mouth. Long acting muscarinic antagonists (LAMAs) are anticholinergic medications, and improve COPD symptoms by bronchodilating and drying up secretions. Anticholinergic side effects may include dry mouth, blurred vision, urinary retention, and constipation. Oral thrush is more commonly seen in inhaled corticosteroid agents such as budesonide (Pulmicort). As well, anticholinergic medications can lead to tachycardia, not bradycardia.

How well did you know this?
1
Not at all
2
3
4
5
Perfectly
50
Q

A new 72-year-old male patient, with a pre-existing diagnosis of COPD, tells the nurse practitioner his pulmonologist stated he was in COPD treatment group A, but he isn’t sure what this means. He does not have his medication list with him, but tells you that his pulmonologist stated the COPD medication he is currently taking works in a similar way to his previous medication but “takes longer to work.” The nurse practitioner knows which of the following medications best fits this description from the patient?

A. Levalbuterol (Xopenex)
B. Formoterol (Foradil)
C. Albuterol (ProAir HFA)
D. Tiotropium (Spiriva)

A

B. Formoterol (Foradil)

The correct answer is formoterol (Foradil). According to the COPD GOLD guidelines, Group A COPD patients can be treated with a short acting beta agonist or long acting beta agonist. These medications provide symptom relief by bronchodilating the airways. Short acting beta agonists, such as levalbuterol (Xopenex HFA) or albuterol (ProAir HFA) provide the quickest relief, working within minutes of administration. According to the patient’s description, he is taking a Group A medication that has a slower onset. The medication that best fits this description is formoterol (Foradil HFA), which have a delayed onset compared to short acting beta agonist medications. Tiotropium is a long acting muscarinic antagonist, which would be appropriate for use in COPD treatment groups B, C, and D.

How well did you know this?
1
Not at all
2
3
4
5
Perfectly
51
Q

A 23-year-old male patient with asthma recently lost his health insurance and abruptly stopped taking his fluticasone (Flonase). Which of the following is not a possible consequence of this abrupt discontinuation?

A. Adrenal suppression
B. Thrush
C. Withdrawal symptoms
D. Joint pain

A

B. Thrush

The correct answer is thrush. While thrush is a potential side effect of taking an inhaled corticosteroid, it is not related to the discontinuation of the medication. With corticosteroids, especially if used long term, there is risk for adrenal suppression with subsequent withdrawals and joint pain if discontinued abruptly.

How well did you know this?
1
Not at all
2
3
4
5
Perfectly
52
Q

A 32-year-old female presents to her primary care office for follow-up on her asthma. Her current asthma medication is inhaled budesonide 80 mcg/formoterol 4.5mcg (Symbicort) as needed only. She reports to the provider that for the past few months she has needed to use the inhaler three times a week. Based on the most recent Global Initiative for Asthma (GINA) guidelines, how should the provider approach this patient’s asthma management?

A. Continue current plan for budesonide/formoterol (Symbicort) as needed.
B. Stop budesonide/formoterol (Symbicort) and start albuterol (Ventolin) as needed.
C. Continue budesonide/fomoterol as needed and start beclomethasone (Qvar) daily.
D. Add montelukast (Singular) daily in addition to her current medication.

A

C. Continue budesonide/fomoterol as needed and start beclomethasone (Qvar) daily.

The correct answer is to continue budesonide/formoterol (Symbicort) as needed and start beclomethasone (Qvar) daily. This patient was prescribed an inhaled corticosteroid-long acting beta-agonist (ICS-LABA) as needed only which is asthma step one treatment. Since she is needing to use the inhaler more than two times a week, we need to intensify treatment and move to step two. According to the GINA guidelines, Step two treatment is an ICS daily. Beclomethasone (Qvar) is an ICS. Albuterol (Ventolin) is a SABA, and a SABA alone is no longer recommended as treatment. Adding montelukast (Singular), a leukotriene receptor antagonist, occurs in step three treatment for asthma.

How well did you know this?
1
Not at all
2
3
4
5
Perfectly
53
Q

A 42-year-old male presents for a visit for follow-up on an emergency room visit for an asthma exacerbation. His current asthma medications include budesonide 160 mcg/formoterol 4.5mcg (Symbicort) daily and montelukast (Singulair) daily. The patient is asking what steps can be taken to help his asthma. What is the most appropriate response from the nurse practitioner?

A. Oral corticosteroids such as prednisone can prevent exacerbations.
B. You should be referred to a pulmonologist for further management.
C. The dose of budesonide/formoterol (Symbicort) can be increased.
D. There is nothing that can be added to prevent exacerbations.

A

B. You should be referred to a pulmonologist for further management.

The correct answer is “you should be referred to a pulmonologist for further management.” This patient is on a daily inhaled corticosteroid long acting beta-agonist combination (ICS-LABA) and leukotriene receptor antagonist (LTRA), indicating he is being managed at step three of asthma treatment. As well, he is having exacerbations, so his treatment likely needs to be intensified. Any patient in step four and above should be referred to pulmonology and not managed in a primary care setting. Oral corticosteroids are used to treat acute exacerbations, not as prevention. While there are options that can be added to prevent exacerbations, and although it might be possible to increase his ICS-LABA dose, this is beyond the scope of primary care, and should be done by a pulmonary specialist.

How well did you know this?
1
Not at all
2
3
4
5
Perfectly
54
Q

Your 65-year-old male patient was recently started on levofloxacin (Levaquin) for a left lower lobe pneumonia. Which of the following patient statements would warrant an immediate visit?

A. “I have begun to have some diarrhea.”
B. “I am urinating more often than usual.”
C. “The back of my foot is hurting.”
D. “I have begun to lose my appetite.”

A

C. “The back of my foot is hurting.”

The correct answer is “the back of my foot is hurting.” A black box warning pertaining to the fluoroquinolone drug class is potential achilles tendon rupture. It is imperative that patients should be educated to follow-up if any new heel pain begins to occur after taking this medication.

How well did you know this?
1
Not at all
2
3
4
5
Perfectly
55
Q

A 64-year-old female patient presents with nasal congestion and a mild sore throat. The nurse practitioner diagnoses her with an upper respiratory infection. She has a past medical history of GERD, diabetes, and hypertension. Which of the following would be inappropriate to prescribe to this patient?

A. Pseudoephedrine (Sudafed)
B. Saline nasal spray
C. Chloraseptic throat lozenges
D. Acetaminophen (Tylenol)

A

A. Pseudoephedrine (Sudafed)

The correct answer is pseudoephedrine (Sudafed). Decongestants, such as pseudoephedrine, work in the body to relieve nasal congestion by constricting blood vessels. When vasoconstriction occurs, there is potential for an increase in blood pressure. If a patient already has a history of hypertension, it is prudent to ideally avoid this class of medication and use other medications such as saline nasal spray or acetaminophen for symptom relief.

How well did you know this?
1
Not at all
2
3
4
5
Perfectly
56
Q

A 32-year-old male patient with a history of GERD, anxiety, and asthma presents to the clinic today with chest congestion and fever. The nurse practitioner diagnoses him with a right middle lobe pneumonia. Which antibiotic should be initiated for this patient?

A. Amoxicillin (Amoxil)
B. Levofloxacin (Levaquin)
C. Doxycycline (Vibramycin)
D. Azithromycin (Z-pak)

A

B. Levofloxacin (Levaquin)

The correct answer is levofloxacin (Levaquin). Due to this patient’s history of a significant respiratory condition, it is necessary to start a medication from the fluoroquinolone drug class for heavy antibiotic coverage. Patients with an underlying respiratory history are at a higher risk for complications and overall mortality.

How well did you know this?
1
Not at all
2
3
4
5
Perfectly
57
Q

A 77-year-old patient is being seen in the office today for difficulties urinating. They also report dry mouth, constipation, blurred vision, tachycardia and decreased sweating. These symptoms started about one week ago. The patient believes that these symptoms arose after beginning to take an over-the-counter (OTC) allergy medication for seasonal allergies. What medication is the most likely cause of these symptoms?

A. Sertraline (Zoloft)
B. Budesonide (Pulmicort)
C. Fluticasone propionate (Flonase)
D. Diphenhydramine (Benadryl)

A

D. Diphenhydramine (Benadryl)

The correct answer is diphenhydramine (Benadryl). First-generation antihistamines are known to cause anticholinergic symptoms, especially in the elderly population. Anticholinergic symptoms include: dry mouth, blurred vision, urinary retention, constipation, tachycardia, decreased sweating, confusion, and impaired memory. Fluticasone propionate (Flonase) is an intranasal corticosteroid and budesonide (Pulmicort) is an inhaled corticosteroid. These medications are not known to cause anticholinergic effects. Sertraline (Zoloft) is a selective serotonin reuptake inhibitor (SSRI). While SSRIs can cause dry mouth and blurred vision, they will not cause anticholinergic effects.

How well did you know this?
1
Not at all
2
3
4
5
Perfectly
58
Q

The nurse practitioner is examining a middle-age patient who has chief complaints of dry cough, runny nose, congestion, and watery eyes. Symptoms began three weeks ago, when the patient was mowing their lawn. They have noticed the symptoms decrease slightly until they mow their lawn again each week. The patient reports that this is the first time they have experienced these types of symptoms since they were a teenager. They have not taken any medications for their symptoms. Upon examination, the nurse practitioner notes: nasal turbinates are pale and boggy with clear mucus discharge, injected conjunctiva, periorbital edema, clear lung fields with a sporadic dry cough, and multiple sneezes during the exam. What would be the most appropriate treatment for this patient?

A. Loratadine (Claritin)
B. Fluticasone (Flonase)
C. Diphenhydramine (Benadryl)
D. Dextromethorphan (Delsym)

A

B. Fluticasone (Flonase)

The correct answer is fluticasone (Flonase). First-line treatment for allergic rhinitis is an intranasal corticosteroid, like fluticasone spray (Flonase). Diphenhydramine (Benadryl) and loratadine (Claritin) are both antihistamines, which would be a second-line treatment option. Dextromethorphan (Delsym) is antitussive. This patient is experiencing a dry cough, but it is likely due to a post-nasal drip that would likely be resolved with intranasal corticosteroid treatment.

How well did you know this?
1
Not at all
2
3
4
5
Perfectly
59
Q

In the classic triad of atopy (asthma, allergies, and atopic dermatitis), inflammation is an underlying cause of each of the associated ailments. What class of medication is first-line in the treatment of these disorders?

A. Cholinergic agonists
B. Antihistamines
C. Corticosteroids
D. Leukotriene receptor antagonists

A

C. Corticosteroids

The correct answer is corticosteroids. In all of these conditions, corticosteroids are indicated for first-line treatment to help decrease the inflammatory response. Allergic rhinitis first-line treatment is an intranasal corticosteroid, atopic dermatitis treatment begins with topical corticosteroids, and, per GINA guidelines, the first step of asthma treatment is with inhaled corticosteroids.Antihistamines are second-line treatment for allergic rhinitis, but they are not indicated for the entire atopy triad. Leukotriene receptor antagonists provide anti-inflammatory effects in the lungs for asthmatic patients only, and cholinergic agonists are largely used to treat urinary retention, glaucoma, and myasthenia gravis. It is not an appropriate drug class for all three of the listed conditions.

How well did you know this?
1
Not at all
2
3
4
5
Perfectly
60
Q

Which of the following is an appropriate time to discontinue glucophage (Metformin)?

A. The patient develops B12 deficiency anemia
B. The patient has diarrhea after beginning the medication
C. The patient has a GFR of 27
D. The patient loses ten pounds after beginning the medication

A

C. The patient has a GFR of 27

The correct answer is when the patient has a GFR of 27. Glucophage (Metformin) is commonly known to be weight neutral or induce weight loss. In addition, with long term use, it may precipitate a B12 deficiency anemia but that can be treated with additional supplements as needed, and is not a reason for medication discontinuation. Gastrointestinal side effects such as nausea and diarrhea may be dose limiting, but tend to improve over time. Having a GFR of 27 places a patient in stage 4 chronic kidney disease. The risks outweigh the benefits of this medication once the GFR is less than 30, so it is imperative to discontinue the Metformin.

How well did you know this?
1
Not at all
2
3
4
5
Perfectly
61
Q

A 63-year-old male patient who is diabetic is on the max dosage of glucophage (Metformin) and his hemoglobin A1C is still high. The nurse practitioner educates the patient that he will be starting a new medication that requires him to keep hydrated. He is also educated to report any new urinary issues. Which medication is the nurse practitioner likely starting him on?

A. Pioglitazone (Actos)
B. Glipizide (Glucotrol)
C. Sitagliptin (Januvia)
D. Dapagliflozin (Farxiga)

A

D. Dapagliflozin (Farxiga)

The correct answer is dapagliflozin (Farxiga). SGLT2 inhibitors, such as dapagliflozin, work in the body by preventing the kidneys from being able to absorb glucose. Glucose is then excreted via the urine. Due to the presence of glucose in the urine, this class of medication is not ideal for those with recurrent yeast infections or urinary tract infections. It is also imperative to keep well-hydrated due to this medication’s diuretic effects. Pioglitazone (Actos), glipizide (Glucotrol), and sitagliptin (Januvia) are a part of other diabetic medication classes that do not have these same effects on the body.

How well did you know this?
1
Not at all
2
3
4
5
Perfectly
62
Q

The nurse practitioner is concerned about prescribing an antidiabetic medication that may cause hypoglycemia because the patient has a history of falls. Which of the following medications would be most appropriate to prescribe?

A. Glucophage (Metformin)
B. Glipizide (Glucotrol)
C. Repaglinide (Prandin)
D. Insulin lispro (Humalog)

A

A. Glucophage (Metformin)

The correct answer is glucophage (Metformin). Sulfonylureas, insulin, and meglitinides are all known to cause potential issues with hypoglycemia. Due to its mechanism of action, glucophage (Metformin) does not present this same risk as it does not impact the amount of insulin the pancreas secretes.

How well did you know this?
1
Not at all
2
3
4
5
Perfectly
63
Q

Upon reviewing a patient’s lab results, it noted that the TSH is 10 and the T3/T4 are decreased. Which of the following medications should the nurse practitioner initiate for this patient?

A. Propranolol (Inderal)
B. Propylthiouracil (PTU)
C. Levothyroxine (Synthroid)
D. Methimazole (Tapazole)

A

C. Levothyroxine (Synthroid)

The correct answer is levothyroxine (Synthroid). When the TSH is elevated and the active thyroid hormones (T3/T4) are decreased, the patient has hypothyroidism. The medication used to treat hypothyroidism is levothyroxine (Synthroid). This medication acts as a synthetic thyroid hormone replacement ideally to achieve euthyroid.

How well did you know this?
1
Not at all
2
3
4
5
Perfectly
64
Q

A 44-year-old female patient makes an appointment with the nurse practitioner with concerns about fatigue. She says she has been fatigued for a while now, and she has noticed that she has been gaining a significant amount of weight. The nurse practitioner checks labs, including thyroid function. Which lab results would suggest that this patient has hypothyroidism?

A. Elevated TSH, decreased T3, decreased T4
B. Decreased TSH, decreased T3, decreased T4
C. Normal TSH, elevated T3, decreased T4
D. Decreased TSH, elevated T3, decreased T4

A

A. Elevated TSH, decreased T3, decreased T4

The correct answer is an elevated TSH, decreased T3, and decreased T4. In hypothyroidism, the thyroid is not functioning properly and not producing enough thyroid hormones such as triiodothyronine (T3) and thyroxine (T4). This causes the pituitary gland to produce more thyroid stimulating hormone (TSH) in an attempt to prompt the thyroid into proper functioning. The end result would be an elevated TSH and decreased levels of T3 and T4.

How well did you know this?
1
Not at all
2
3
4
5
Perfectly
65
Q

A 44-year-old female patient comes into the clinic for her six week follow up appointment after starting treatment for hypothyroidism. She started taking 50 mcg of levothyroxine (Synthroid) daily. Her lab results show that her thyroid stimulating hormone (TSH) is more elevated than at her last appointment. The nurse practitioner asks how the patient is taking her medication. Which statement by the patient would suggest a need for further education?

A. “I take my thyroid medication every day.”
B. “I take my thyroid medication with a glass of water thirty minutes before eating.”
C. “I take my thyroid medication with breakfast to avoid nausea.”
D. “I do not take multiple doses of my thyroid medication.”

A

C. “I take my thyroid medication with breakfast to avoid nausea.”

The correct answer is “I take my thyroid medication with breakfast to avoid nausea.” Levothyroxine (Synthroid) should be taken daily, as prescribed, 30-60 minutes before eating breakfast. Taking this medication with food will decrease the efficacy and we will not see the desired result.

How well did you know this?
1
Not at all
2
3
4
5
Perfectly
66
Q

A 36-year-old pregnant female presents with complaints of worsening anxiety and tremors. She has noticed that despite increasing food intake, she is not gaining weight. She is at 18 weeks gestation. Her lab results are as follows: TSH 0.02 mU/L, free T4 12 ng/dL. What treatment is most appropriate in this patient?

A. Radioactive iodine
B. Propylthiouracil (PTU)
C. No treatment indicated. Monitor labs only.
D. Methimazole (Tapazole)

A

D. Methimazole (Tapazole)

The correct answer is methimazole (Tapazole). With a low TSH and high free T4, this patient’s labs are consistent with hyperthyroidism, requiring treatment. Methimazole (Tapazole) is used for treating hyperthyroidism in pregnancy after the first trimester, and this patient is in their second trimester. PTU is only used in the first trimester, and radioactive iodine is contraindicated in pregnancy.

How well did you know this?
1
Not at all
2
3
4
5
Perfectly
67
Q

Which of the following labs is the most important to monitor in a patient being treated for hyperthyroidism with propylthiouracil (PTU)?

A. Hepatic function panel
B. TSH
C. CBC
D. EKG

A

C. CBC

The correct answer is a CBC. A CBC is the most important to monitor in patients on PTU due to the potential complication of agranulocytosis. A TSH and hepatic function panel may be included in the lab monitoring of a patient with hyperthyroidism, but a CBC would be of higher priority. An EKG is not routinely monitored with PTU administration.

How well did you know this?
1
Not at all
2
3
4
5
Perfectly
68
Q

A 41-year-old female with hyperthyroidism is complaining of anxiety and palpitations. She asks the nurse practitioner for something to help her symptoms. What can the nurse practitioner prescribe to directly treat her symptoms?

A. Sertraline (Zoloft)
B. Methimazole (Tapazole)
C. Buspirone (Buspar)
D. Propranolol (Inderal)

A

D. Propranolol (Inderal)

The correct answer is propranolol (Inderal). Propranolol (Inderal) is a beta blocker that is commonly used to alleviate symptoms in patients with hyperthyroidism. Sertraline (Zoloft) and buspirone (Buspar) do treat generalized anxiety disorder, but in this case we are treating the anxiety and palpitations due to the underlying cause of hyperthyroidism. While methimazole (Tapazole) does treat hyperthyroidism, it does not directly treat her symptoms of anxiety and palpitations.

How well did you know this?
1
Not at all
2
3
4
5
Perfectly
69
Q

When teaching a patient newly diagnosed with Addison’s disease about medication administration, what patient response would warrant further patient education?

A. “I need to take steroids daily because my body does not make enough of its own.”
B. “When I am sick or nauseous, I do not need to take my steroids.”
C. “When I am sick or injured, I may require more steroids.”
D. “I should take the larger dose of steroids in the morning and a smaller dose at night.”

A

B. “When I am sick or nauseous, I do not need to take my steroids.”

The correct answer is “when I am sick or nauseous, I do not need to take my steroids.” Steroids should never be abruptly stopped, especially not for patients with Addison’s disease. In times of illness, injury, or stress, it is even possible that patients will need an increased dose of steroids to provide sufficient support to their healing body during that time. Therefore, if a patient were to make the comment: “When I am sick or nauseous, I do not need to take my steroids” the nurse practitioner would need to continue to educate this patient on proper treatment methods for their condition. All of the other statements were true.

How well did you know this?
1
Not at all
2
3
4
5
Perfectly
70
Q

The nurse practitioner is evaluating a 43-year-old patient in the primary care office for an acute illness. This illness began 4 days ago with a fever and vomiting. The patient reports that they have been unable to avoid vomiting shortly after taking their medications. Vitals signs are: BP 90/52, HR 48, temperature of 100.9F, RR of 19, and O2 saturation of 97% on room air. A random blood glucose finger stick result is 48. The patient has a history of hypothyroidism, Addison’s disease, and mild intermittent asthma. What is the best next step of care for this patient?

A. Ondansetron (Zofran) and instruct the patient to drink apple juice
B. Instruct the patient to take a double dose of their morning steroids
C. Acetaminophen (Tylenol) and symptom management
D. Refer the patient to the emergency department

A

D. Refer the patient to the emergency department

The correct answer is to refer the patient to the emergency department. Addisonian crisis occurs when there is a dangerously low level of cortisol in the body. Symptoms of this condition, also known as acute adrenal insufficiency, include: hypotension, hypoglycemia, and hyperkalemia. This is a life-threatening emergency and requires immediate transfer to the emergency department for further treatment and evaluation. The options of administering ondansetron (Zofran), taking a double dose of morning steroids, and taking acetaminophen (Tylenol) do not address the underlying crisis.

How well did you know this?
1
Not at all
2
3
4
5
Perfectly
71
Q

When being examined for their yearly wellness check, a 61-year-old patient asks if there are any long-term effects from the steroids they were prescribed for their asthma and illnesses as a child. Which of the following is correct?

A. Steroids strengthen the body’s immune system
B. Mood changes and aggression depicted in movies is a false depiction of patients on long-term steroids
C. Long-term use of steroids can inhibit how the body absorbs calcium
D. Steroids cause low blood sugars because it causes the body to release extra insulin

A

C. Long-term use of steroids can inhibit how the body absorbs calcium

The correct answer is that long-term use of steroids can inhibit how the body absorbs calcium. Steroids reduce the body’s natural ability to absorb calcium and put patients at higher risk of osteopenia with long-term steroid use. Steroids can also cause patients to experience mood changes or agitation, hyperglycemia, and may impair the body’s natural immune response.

How well did you know this?
1
Not at all
2
3
4
5
Perfectly
72
Q

A 74-year-old patient presents for a follow-up after a recent low trauma fracture. Their past medical history includes hypertension, type 2 diabetes mellitus, asthma, and gastroesophageal reflux disease. When doing medication reconciliation, which of the following medications would warrant further intervention by the nurse practitioner?

A. Fluticasone (Flovent HFA)
B. Hydrochlorothiazide (HCTZ)
C. Pantoprazole (Protonix)
D. Pioglitazone (Actos)

A

C. Pantoprazole (Protonix)

Patients with low trauma fractures or at increased risk of osteoporosis should be cautious with proton pump inhibitors (PPIs) like pantoprazole (Protonix). Long term use of PPIs has been shown to decrease bone density and put patients at an increased fracture risk. Fluticasone is an inhaled corticosteroid and would not be of concern given his medical history. Thiazide diuretics like hydrochlorothiazide can actually help stimulate osteoblasts and retain calcium, making them ideal for patients with osteoporosis. Pioglitazone is a diabetic medication contraindicated in heart failure, but not in osteoporosis.

How well did you know this?
1
Not at all
2
3
4
5
Perfectly
73
Q

For which of the following conditions would esomeprazole (Nexium) not be an appropriate treatment?

A. Clostridium difficile (C. diff)
B. Duodenal ulcer
C. Gastroesophageal reflux disease
D. Helicobactor pylori (H. pylori)

A

A. Clostridium difficile (C. diff)

How well did you know this?
1
Not at all
2
3
4
5
Perfectly
74
Q

A 64-year-old female presents with concerns of consistent abdominal pain for the last few days. She recently sprained her wrist and has been taking ibuprofen (Motrin) for the last two weeks. She reports the pain occasionally improves during mealtime, but comes back shortly after. Her H. pylori test is negative. Which of the following would be an important teaching point about the mainstay of treatment for this patient’s condition?

A. “A combination of four different medications is needed to treat this”
B. “Finish the entire course of the antibiotic even if you start feeling better”
C. “We need to monitor your lipids to ensure your triglyceride level goes down”
D. “You can expect to be on this medication for at least a month”

A

D. “You can expect to be on this medication for at least a month”

How well did you know this?
1
Not at all
2
3
4
5
Perfectly
75
Q

What diagnostic test needs to be monitored in a patient regularly taking ondansetron (Zofran)?

A. Glomerular filtration rate (GFR)
B. Electrocardiogram (EKG)
C. Renal ultrasound
D. Complete blood count (CBC)

A

B. Electrocardiogram (EKG)

The correct answer is EKG. With routine use, this medication can cause QT prolongation, and therefore an EKG should be obtained. Zofran is not associated with renal toxicities or CBC changes. Although a patient who is regularly taking Zofran may be an oncology patient who needs a CBC, GFR, or renal ultrasound monitored, this specific question is focusing on monitoring needed specifically due to Zofran itself, not the condition being treated.

How well did you know this?
1
Not at all
2
3
4
5
Perfectly
76
Q

A patient with severe nausea is prescribed prochlorperazine (Stemetil). What side effects will the nurse practitioner ensure to educate the patient about?

A. Hepatic impairment
B. Respiratory depression
C. Dry mouth and urinary retention
D. QT prolongation

A

C. Dry mouth and urinary retention

The correct answer is dry mouth and urinary retention. Prochlorperazine (Stemetil) can cause anticholinergic side effects including dry mouth, urinary retention, and blurred vision. It can cause sedation, but respiratory depression is an adverse effect more associated with promethazine (Phenergan). Hepatic impairment and QT prolongation are caused by ondansetron (Zofran).

How well did you know this?
1
Not at all
2
3
4
5
Perfectly
77
Q

The parent of a 20-month-old child asks the nurse practitioner for a prescription of promethazine (Phenergan) for their child during a severe case of gastroenteritis. The child has normal renal and hepatic function and weighs 14 kg. What response is most appropriate?

A. This is contraindicated in any person under 2 years of age.
B. The regular dose can be prescribed.
C. This is only prescribed when a person is admitted to the hospital.
D. A lower dose can be prescribed.

A

A. This is contraindicated in any person under 2 years of age.

The correct answer is “this is contraindicated in any person under 2 years of age.” Promethazine (Phenergan) is contraindicated in anyone under 2 years old due to risk of respiratory depression, regardless of dosing. Adults may be prescribed this medication in an outpatient setting, but they will need to be counseled on risk for respiratory depression and sedation.

How well did you know this?
1
Not at all
2
3
4
5
Perfectly
78
Q

A 79-year-old male patient is complaining of intermittent issues with constipation and states that he recently passed out while using the restroom. Which medication would be the best initial medication option for this patient’s complaint?

A. Lactulose (Generlac)
B. Magnesium hydroxide (Milk of Magnesia)
C. Sucralfate (Carafate)
D. Docusate sodium (Colace)

A

D. Docusate sodium (Colace)

The correct answer is docusate sodium (Colace). Syncope while using the restroom is often indicative of underlying constipation and straining during bowel movements. In order to avoid future straining and patient risk for falls, a stool softener such as docusate sodium can be prescribed. This medication moves more water into the stool, making it easier to pass.

How well did you know this?
1
Not at all
2
3
4
5
Perfectly
79
Q

The nurse practitioner is following up with a patient today about chronic constipation. The patient has been taking docusate sodium (Colace) for a few weeks now, but has not seen much of a difference. The nurse practitioner decides to have the patient take psyllium fiber (Metamucil) to help prevent constipation. What is an important piece of education for this medication?

A. Take with food to prevent nausea
B. Take with a full glass of water
C. Start taking when feeling constipation start
D. Stop taking once constipation is resolved

A

B. Take with a full glass of water

The correct answer is take with a full glass of water. Psyllium fiber (Metamucil) is a bulk laxative that works by absorbing fluids into the intestines allowing the stool to pass easier. Metamucil is used in both the treatment and prevention of constipation, so it should be taken regularly. It has minimal side effects, and the patient should always take it with a full glass of water to prevent the possibility of intestinal obstruction.

How well did you know this?
1
Not at all
2
3
4
5
Perfectly
80
Q

Which of the following medications would be the most effective treatment option for a patient with inflammatory bowel disease?

A. Docusate sodium (Colace)
B. Psyllium fiber (Metamucil)
C. Diphenoxylate/atropine (Lomotil)
D. Loperamide (Imodium)

A

C. Diphenoxylate/atropine (Lomotil)

The correct answer is diphenoxylate/atropine (Lomotil). This is the most appropriate treatment option for a patient with inflammatory bowel disease due to its antidiarrheal and pain relieving properties. Loperamide (Imodium) is an over-the-counter antidiarrheal medication that does not offer any pain relief. Psyllium fiber (Metamucil) and docusate sodium (Colace) are used tot treat constipation and could possibly worsen diarrheal symptoms.

How well did you know this?
1
Not at all
2
3
4
5
Perfectly
81
Q

A young adult female patient calls into the clinic today because she states she accidentally forgot to take her ethinyl estradiol/norethindrone acetate (Loestrin Fe) yesterday. What should the nurse practitioner advise her to do?

A. Go ahead and take the missed pill, and use a backup method for the next month.
B. Throw out her current pack, and start a new pack today.
C. Go ahead and take the missed pill, even though it means taking two pills in one day.
D. Skip the missed pill, and continue taking her pills as normal.

A

C. Go ahead and take the missed pill, even though it means taking two pills in one day.

The correct answer is “go ahead and take the missed pill, even though it means taking two pills in one day.” When taking a combined oral contraceptive such as Loestrin Fe, a patient can miss one pill and still be protected against pregnancy. It is imperative to instruct the patient to take the missed dose as soon as possible and then continue on with their remaining pills on time.

How well did you know this?
1
Not at all
2
3
4
5
Perfectly
82
Q

The estrogen component of combined oral contraceptives is responsible for all of the following symptoms except:

A. Nausea
B. Worsening acne
C. Breast tenderness
D. Breakthrough bleeding

A

B. Worsening acne

The correct answer is worsening acne. This symptom is likely due to progesterone levels instead of estrogen levels. Progesterone may increase sebum production, which then can lead to a higher incidence of clogged pores and ultimately acne. Nausea, breast tenderness and breakthrough bleeding are all attributed to the estrogen component of combined oral contraceptives.

How well did you know this?
1
Not at all
2
3
4
5
Perfectly
83
Q

In which of the following patients would it be appropriate to prescribe a combined oral contraceptive?

A. A 39-year-old woman who smokes and has GERD
B. A 17-year-old woman who has occasional migraines with an aura
C. A 23-year-old woman who is breastfeeding
D. A 29-year-old woman with endometriosis

A

D. A 29-year-old woman with endometriosis

The correct answer is a 29-year-old woman with endometriosis. Combined oral contraceptives are frequently used to treat the pain associated with endometriosis, and often will help patients from developing further endometriosis. Being older than 35 and smoking, migraines with an aura, and breastfeeding are all contraindications to this class of medication. Smoking increases risk for thrombus while taking this medication and the estrogen component of COCs impacts milk supply in those who are breastfeeding. In addition, there is an increased risk for stroke while taking COCs if the patient has a history of migraines with an aura.

How well did you know this?
1
Not at all
2
3
4
5
Perfectly
84
Q

A 65-year-old postmenopausal woman presents for a gynecology visit with complaints of severe hot flashes that are disrupting her quality of life. She has tried herbal supplements and other non-pharmacologic approaches with no improvement. She has a past medical history of osteoporosis, prediabetes, and gout. She has no personal or family history of cancer. She has a past surgical history of a total abdominal hysterectomy (TAH) due to uterine fibroids. What will the nurse practitioner offer the patient for treatment of her menopausal symptoms?

A. Hormone replacement therapy (HRT) with an estrogen and progestin combination
B. A mental health referral for psychotherapy
C. Continue herbal supplements, as there are no other options given her history and comorbidities
D. Hormone replacement therapy (HRT) with estrogen only

A

D. Hormone replacement therapy (HRT) with estrogen only

The correct answer is hormone replacement therapy (HRT) with estrogen only. This patient has tried and failed options for addressing her vasomotor symptoms (hot flashes) due to menopause. The patient has no history of cancer, cardiac disease, or blood clots and thus is a candidate for HRT. Since she no longer has a uterus due to the TAH, she will only need estrogen replacement. Progestin is only indicated for women with an intact uterus in order to prevent endometrial cancer. Although this patient may benefit from the mental health referral, the treatment that is most likely to help her vasomotor symptoms specifically is HRT.

How well did you know this?
1
Not at all
2
3
4
5
Perfectly
85
Q

A 55-year-old postmenopausal woman is seeking help for her mood swings and hot flashes. Her BP is 150/92 and HR 88. Her current medications include the following: losartan (Cozaar) 100mg, rosuvastatin (Crestor) 10mg, metformin (Glucophage) 1,000mg, and apixaban (Eliquis) 5mg BID. What treatment is most appropriate for this patient?

A. Paroxetine (Paxil)
B. Gabapentin (Neurotin)
C. Estradiol (Climara)
D. Estradiol (Climara) and Progesterone (Prometrium)

A

A. Paroxetine (Paxil)

The correct answer is paroxetine (Paxil). Selective serotonin reuptake inhibitors (SSRIs) have shown to improve both menopausal vasomotor symptoms, including hot flashes, as well as menopausal mood disorders. Paroxetine (Paxil) is an SSRI and is particularly beneficial for these symptoms. Gabapentin (Neurontin) is not commonly used for menopausal mood disorders. The patient’s blood pressure is currently elevated, and it appears she is being treated for hypertension, hyperlipidemia, and is on an anticoagulant for a potentially previous clotting issue. With these cardiac risks, this patient would not be an ideal candidate for hormone replacement therapy (HRT), either estrogen alone, or in combination with progesterone.

How well did you know this?
1
Not at all
2
3
4
5
Perfectly
86
Q

What herbal supplement is commonly used for menopausal symptoms?

A. St. John’s wort
B. Turmeric
C. Black cohosh
D. Saw palmetto

A

C. Black cohosh

The correct answer is black cohosh. This is most commonly used in menopause. St. John’s wort is most often used for depression. Turmeric is most often used as an anti-inflammatory for arthritis. Saw palmetto is most often used for prostate issues.

How well did you know this?
1
Not at all
2
3
4
5
Perfectly
87
Q

Which of the following assessments is critical prior to prescribing a PDE-5 inhibitor for erectile dysfunction?

A. CMP
B. Echocardiogram
C. EKG
D. CBC

A

C. EKG

The correct answer in an EKG. A thorough cardiac assessment, in addition to an EKG, is imperative prior to starting a PDE-5 inhibitor such as sildenafil. This will help to establish whether the patient’s heart will be able to tolerate taking the medication. Often, it is found that those with erectile dysfunction have underlying cardiac disease, which would need to be addressed prior to this medication class being prescribed. While a CMP, echocardiogram, and CBC may be of some use when identifying underlying cardiac disease, the EKG is the most efficient and cost-effective way of evaluating the heart directly.

How well did you know this?
1
Not at all
2
3
4
5
Perfectly
88
Q

A 67-year-old male patient is currently being treated for benign prostatic hyperplasia. He is concerned that his finasteride (Proscar) is not adequately treating his symptoms. When assessing further, which of the following questions is most important to ask this patient?

A. How long have you been taking this medication?
B. Do you also have a history of erectile dysfunction?
C. Are you taking herbal supplements with your medication?
D. When was the last time you had a PSA level drawn?

A

A. How long have you been taking this medication?

The correct answer is “how long have you been taking this medication?” Finasteride (Proscar) works in the body by decreasing the size of the prostate gland over time. This reduction in size then alleviates the patient’s symptoms. However, it may take up to six months to see the full effect of the drug. Inquiring about erectile dysfunction, if they are taking herbal supplements, and the last time the patient had a PSA level drawn does not directly address how well this medication is treating the patient’s BPH.

How well did you know this?
1
Not at all
2
3
4
5
Perfectly
89
Q

A 76-year-old male patient was recently diagnosed with benign prostatic hyperplasia and started on medication. At today’s visit, he reports that he took his new medication with lunch and became so light headed and clammy that his wife took him to the emergency room. The nurse practitioner knows that this reaction was most likely caused by which of the following medications?

A. Nitroglycerin (Nitrostat)
B. Finasteride (Proscar)
C. Enalapril (Innovace)
D. Terazosin (Hytrin)

A

D. Terazosin (Hytrin)

The correct answer is terazosin (Hytrin). First-line medication for benign prostate hyperplasia includes the use of alpha blockers such as terazosin. These medications are notorious for a first-time dose effect of profound orthostatic hypotension which can result in the patient experiencing true syncope. Nitroglycerin and enalapril are used in other conditions related to the cardiovascular system. While finasteride is used for BPH, it belongs to the 5-alpha reductase inhibitors medication classification, and these do not have the same first dose effect as the alpha blockers, and have no overall effect on the blood pressure.

How well did you know this?
1
Not at all
2
3
4
5
Perfectly
90
Q

A 75-year-old female patient presented to the clinic today after being diagnosed with osteoporosis and started on a new medication called alendronate (Fosamax). She asks the nurse practitioner how this medication works to help build her bone mass. What is the mechanism of action of bisphosphonates?

A. Increase absorption of calcium
B. Promote osteoblast absorption
C. Inhibits osteoclast resorption
D. Decrease absorption of calcium

A

C. Inhibits osteoclast resorption

The correct answer is that they inhibit osteoclast resorption. This mechanism allows osteoblasts to be more effective, helping to promote bone growth and reducing bone breakdown. With bisphosphonates, there may be an overall alteration in calcium levels, there is no overall effect on calcium absorption itself. With these medications, there is no osteoblast absorption that occurs.

How well did you know this?
1
Not at all
2
3
4
5
Perfectly
91
Q

A 70-year-old female patient returns for her follow-up appointment after having her first bone DEXA scan performed. A nurse practitioner is reviewing the results. Which value indicates the patient has osteopenia?

A. T-score of -1.3
B. T-score of 2.5
C. T score of -0.5
D. T-score of -2.5

A

A. T-score of -1.3

The correct answer is a T-score of -1.3. DEXA scans are a tool that providers use to help determine if a patient has decreased bone mass, and results are reported in T-scores. The lower the T-score, the weaker the bones are. Any T-score that is less than -2.5 is considered osteoporosis. A T-score from -1.1 to -2.4 is considered osteopenia. A T-score equal to or above -1.0 is considered normal bone density.

How well did you know this?
1
Not at all
2
3
4
5
Perfectly
92
Q

A patient is being seen in the clinic today for a follow up on her osteoporosis. She currently takes alendronate (Fosamax) once a week. When the nurse practitioner asks how she takes her medication, she states that she takes it at night after her last meal of the day. What should the nurse practitioner address today in regards to her method of taking her alendronate (Fosamax)?

A. Continue taking the medication in the evening
B. Direct the patient to take the medication in the morning, on an empty stomach, with a full glass of water, once per week
C. Take the medication with a glass of orange juice
D. Take the medication at the same time as her calcium and vitamin D supplements

A

B. Direct the patient to take the medication in the morning, on an empty stomach, with a full glass of water, once per week

The correct answer is to direct the patient to take the medication in the morning, on an empty stomach, with a full glass of water, once per week. This allows the medication to be absorbed properly and prevents potential esophageal erosion. As well, patients cannot take this medication concurrently with juices or other medications.

How well did you know this?
1
Not at all
2
3
4
5
Perfectly
93
Q

A 65-year-old female patient with a history of heart failure presents to the clinic with a new diagnosis of osteoarthritis in her right knee. She has been having difficulty managing the pain, and it is limiting her daily activities. Which of the following medications will be most appropriate for this patient?

A. Diclofenac sodium (Voltaren)
B. Acetaminophen (Tylenol)
C. Naproxen (Aleve)
D. Indomethacin (Indocin)

A

B. Acetaminophen (Tylenol)

The correct answer is acetaminophen (Tylenol). Diclofenac sodium, naproxen, and indomethacin are all NSAIDs. These medications should not be used in patients diagnosed with heart failure due to sodium and fluid retention which can further exacerbate the condition. Tylenol is not part of this drug class, and can be utilized to help manage the patient’s pain.

How well did you know this?
1
Not at all
2
3
4
5
Perfectly
94
Q

Which of the following medications are considered first-line treatment for a patient diagnosed with rheumatoid arthritis?

A. Naproxen (Aleve)
B. Prednisone (Prednisolone)
C. Methotrexate (Trexall)
D. Tofacitinib (Xeljanz)

A

C. Methotrexate (Trexall)

The correct answer is methotrexate (Trexall). This is a part of the disease-modifying antirheumatic drug (DMARD) class. These medications can slow disease progression and help preserve joint function. DMARDs are typically prescribed only by rheumatologists, not by nurse practitioners working in primary care; therefore, patients needing these medications should be referred.

How well did you know this?
1
Not at all
2
3
4
5
Perfectly
95
Q

A female patient is in the clinic for her annual follow up. She has a history of rheumatoid arthritis, but is otherwise healthy. She reports taking her methotrexate (Trexall) as prescribed by her rheumatologist. The nurse practitioner orders a CBC as a part of her annual lab work. Which of the following anemias is the nurse practitioner monitoring for, which can be common in those taking this particular medication?

A. Folic acid deficiency
B. Vitamin B12 deficiency
C. Iron-deficiency anemia
D. Anemia due to lead toxicity

A

A. Folic acid deficiency

The correct answer is a folic acid deficiency. As this medication can deplete folate levels, patients who take methotrexate (Trexall) should have their folate levels monitored and take a folate supplement. Methotrexate (Trexall) has no effect on vitamin B12, iron, or lead levels.

How well did you know this?
1
Not at all
2
3
4
5
Perfectly
96
Q

A 54-year-old male is in office for his annual well-check. He has a history of gout, and is currently taking allopurinol (Zyloprim) for maintenance and prevention. He has an otherwise unremarkable health history. Which of the following labs should the nurse practitioner routinely monitor while a patient is on allopurinol (Zyloprim)?

A. Complete metabolic panel (CMP)
B. Amylase and lipase
C. Glycosylated hemoglobin
D. Complete blood count (CBC)

A

D. Complete blood count (CBC)

The correct answer is a complete blood count (CBC). A complete blood count (CBC) should be checked routinely as allopurinol (Zyloprim) can cause bone marrow suppression. Liver and kidney function tests are also other labs that should be monitored while taking allopurinol. Allopurinol (Zyloprim) has no overall effect on amylase, lipase, complete metabolic panel (CMP) or glycosylated hemoglobin levels.

How well did you know this?
1
Not at all
2
3
4
5
Perfectly
97
Q

A 56-year-old male patient presents to the clinic with an erythematous metatarsal joint of his big toe, and reports excruciating pain. Which of the following medications should the nurse practitioner not consider starting at this time?

A. Colchicine (Colcrys)
B. Indomethacin (Indocin)
C. Allopurinol (Zyloprim)
D. Naproxen (Aleve)

A

C. Allopurinol (Zyloprim)

The correct answer is allopurinol (Zyloprim). Allopurinol works by decreasing the amount of uric acid made by the cells in the body, preventing excess uric acid from depositing in the joints. Allopurinol should not be initiated or discontinued during an acute attack because it can prolong the acute phase. However, colchicine, indomethacin, and naproxen are all indicated for use during the acute onset of gout attacks.

How well did you know this?
1
Not at all
2
3
4
5
Perfectly
98
Q

Which of the following medications works by binding to white blood cells, preventing their ability to replicate and cause inflammation during an acute attack?

A. Colchicine (Colcrys)
B. Indomethacin (Indocin)
C. Prednisone (Prednisolone)
D. Naproxen (Aleve)

A

A. Colchicine (Colcrys)

The correct answer is colchicine (Colcrys). Colchicine treats inflammation associated with acute gout attacks by disrupting the normal inflammatory response of the body and preventing the accumulation of white blood cells at the site of the deposition of gout crystals. Indomethacin, prednisone, and naproxen can also be used to control inflammation during an acute attack, but do not have this same mechanism of action within the body.

How well did you know this?
1
Not at all
2
3
4
5
Perfectly
99
Q

A 15-year-old female patient presents to the clinic with a classic presentation of strep throat. When the nurse practitioner prescribes the patient antibiotics, the patient states that penicillin makes her itchy. Which of the following medications would be the most appropriate alternative for this patient?

A. Cephalexin (Keflex)
B. Ciprofloxacin (Ciproxin)
C. Doxycycline (Vibramycin)
D. Sulfamethoxazole/trimethoprim (Bactrim)

A

A. Cephalexin (Keflex)

The correct answer is cephalexin (Keflex). The first-line drug treatment for strep throat related to group A streptococcus is penicillin or amoxicillin. However, since the patient reported an non-anaphylactic allergy to this medication class, an alternative medication from the cephalosporin drug class would be appropriate. Other alternative options include medications from the macrolide drug class, such as azithromycin and clarithromycin. Ciprofloxacin, doxycycline, and sulfamethoxazole/trimethoprim are not indicated in the treatment of group A strep.

How well did you know this?
1
Not at all
2
3
4
5
Perfectly
100
Q

A middle-aged man calls into the clinic today stating that he recently started an antibiotic, and today he received a first degree sunburn while gardening for a couple of hours outside. Which of the following antibiotics is he likely taking based on this finding?

A. Amoxicillin (Amoxil)
B. Levofloxacin (Levaquin)
C. Cephalexin (Keflex)
D. Doxycycline (Vibramycin)

A

D. Doxycycline (Vibramycin)

The correct answer is doxycycline (Vibramycin). Photosensitivity is a relatively common side effect of the tetracycline drug class. Photosensitivity signs and symptoms include manifestations such as sunburn and blistering. This side effect may last up to several months after cessation of tetracycline use so patient education is imperative. Penicillin, cephalosporin, and fluoroquinolone drug classes have no associated photosensitivity.

How well did you know this?
1
Not at all
2
3
4
5
Perfectly
101
Q

Metronidazole (Flagyl) can be used to treat all of the following conditions except:

A. Trichomoniasis
B. Diverticulitis
C. Liver abscess
D. Erysipelas

A

D. Erysipelas

The correct answer is erysipelas. Traditionally, erysipelas is treated with penicillin. If an allergy is present, then a first generation cephalosporin can be prescribed instead. Metronidazole (Flagyl) does not provide coverage for the typical bacterias that cause erysipelas and cellulitis, but would be indicated in the treatment of trichomoniasis, diverticulitis, and liver abscesses

How well did you know this?
1
Not at all
2
3
4
5
Perfectly
102
Q

A 24-year-old female patient presents to the clinic reporting purulent, green, frothy vaginal discharge. Upon further assessment, the nurse practitioner obtains a vaginal swab and visualizes flagella under the microscope. The nurse practitioner knows to treat this condition with which of the following antibiotics?

A. Penicillin G (Bicillin)
B. Ceftriaxone (Rocephin)
C. Metronidazole (Flagyl)
D. Azithromycin (Zithromax)

A

C. Metronidazole (Flagyl)

The correct answer is metronidazole (Flagyl). The nurse practitioner is aware that seeing flagella under the microscope is indicative of trichomoniasis, which is treated with metronidazole (Flagyl). Penicillin G (Bicillin) is used to treat syphilis, ceftriaxone (Rocephin) is used to treat gonorrhea, and azithromycin (Zithromax) is used to treat chlamydia.

How well did you know this?
1
Not at all
2
3
4
5
Perfectly
103
Q

A 26-year-old female patient presents to the clinic for her four week postpartum visit. The patient reports a painful and tender right breast. Upon further assessment, the nurse practitioner palpates a warm and boggy breast and diagnoses the patient with mastitis. The patient has a history of an anaphylactic reaction to penicillin. Which of the following would be the best option to treat her mastitis?

A. Sulfamethoxazole-trimethoprim (Bactrim)
B. Clindamycin (Cleocin)
C. Dicloxacillin (Diclocil)
D. Doxycycline (Vibramycin)

A

B. Clindamycin (Cleocin)

The correct answer is clindamycin (Cleocin). The patient reported a severe penicillin allergy, and dicloxacillin (Diclocil) is a penicillin-based medication. Clindamycin (Cleocin) is a great second-line option for mastitis as it is safe for both the breastfeeding mother and her newborn child. Caution is warranted here due to the risk of GI superinfection with Clostridium difficile, so the patient should be educated to report any new signs and symptoms of diarrhea to the provider immediately. Sulfamethoxazole-trimethoprim (Bactrim) should only be used if MRSA is suspected or confirmed, and doxycycline (Vibramycin) is not indicated in the treatment of mastitis.

How well did you know this?
1
Not at all
2
3
4
5
Perfectly
104
Q

A patient has been diagnosed with tuberculosis and will need to be on rifampin (Rifadin) for 9 months. Which of the following lab tests should be obtained routinely while the patient is taking this medication?

A. Amylase and lipase
B. Liver function tests
C. Glycosylated hemoglobin
D. Triglyceride levels

A

B. Liver function tests

The correct answer is liver function tests. Rifampin (Rifadin) is metabolized in the liver and is a drug that can be severely hepatotoxic. Patients on long-term rifampin (Rifadin) will need serial liver function panels. Rifampin (Rifadin) has no overall effect on amylase, lipase, glycosylated

How well did you know this?
1
Not at all
2
3
4
5
Perfectly
105
Q

Long-term use of proton pump inhibitors will likely lead to which of the following conditions?

A. B12 deficiency anemia
B. Barrett’s esophagus
C. Iron deficiency anemia
D. Chronic bronchitis

A

A. B12 deficiency anemia

Long term use of proton-pump inhibitors has been shown to increase the risk of B12 deficiency anemia, as the mechanism by which these medications work often stops the production of gastric acid and intrinsic factor, which is needed to absorb B12.

How well did you know this?
1
Not at all
2
3
4
5
Perfectly
106
Q

A patient comes in for an annual exam and Pap smear. Upon examination by speculum, there is green frothy discharge seen coming from the patient’s cervical os. The patient confirms they have had green-yellow discharge over the last few weeks. Which of the following medications would be the most appropriate to prescribe?

A. Metronidazole (Flagyl)
B. Ceftriaxone (Rocephin)
C. Doxycycline (Vibramycin)
D. Azithromycin (Zithromax)

A

A. Metronidazole (Flagyl)

Metronidazole (Flagyl) is used to treat a variety of conditions, including trichomoniasis and bacterial vaginosis. Green frothy discharge is a common sign of a trichomoniasis infection. Doxycycline (Vibramycin) and azithromycin (Zithromax) can both be used to treat chlamydia infections, and ceftriaxone (Rocephin) is the antibiotic of choice for gonorrheal infections.

How well did you know this?
1
Not at all
2
3
4
5
Perfectly
107
Q

A 79-year-old man with previously well-controlled BPH is now having increasing issues with urinary retention. He states he recently started a new medication, but is unsure of its name. All of the following medications could lead to increased urinary retention except:

A. Amitriptyline
B. Diphenhydramine (Benadryl)
C. Sertraline (Zoloft)
D. Hydroxyzine (Atarax)

A

C. Sertraline (Zoloft)

Urinary retention is an anticholinergic side effect, which is most commonly seen with tricyclic antidepressants such as amitriptyline, and antihistamines such as diphenhydramine (Benadryl) and hydroxyzine (Atarax). Sertraline (Zoloft) is a selective serotonin reuptake inhibitor, and is a preferred antidepressant in the elder-adult population, and is not known to have anticholinergic side effects.

How well did you know this?
1
Not at all
2
3
4
5
Perfectly
108
Q

An adolescent presents to the clinic with a large bullae in between his fingers. The nurse practitioner diagnoses him with bullous impetigo. Which bacteria is the likely cause?

A. Pseudomonas aeruginosa
B. Staphylococcus aureus
C. Corynebacterium minutissimum
D. Escherichia coli

A

B. Staphylococcus aureus

The most common bacterial causes of bullous impetigo include Staphylococcus aureus and Streptococcus pyogenes. Pseudomonas aeruginosa is a common cause of otitis externa, while Escherichia coli is the most common cause of a urinary tract infection and Corynebacterium minutissimum causes erythrasma, a superficial skin infection.

How well did you know this?
1
Not at all
2
3
4
5
Perfectly
109
Q

A 23-year-old pregnant patient has what appears to be erythema migrans on her left lower leg. Which of the following would be the best treatment option for this patient?

A. Doxycycline (Vibramycin)
B. Azithromycin (Z-pak)
C. Levofloxacin (Levaquin)
D. Amoxicillin (Amoxil)

A

D. Amoxicillin (Amoxil)

This patient has Lyme disease, which is commonly characterized by erythema migrans, which is a stereotypical bull’s eye lesion at the site of the tick bite. First-line treatment for Lyme disease, regardless of age, is doxycycline (Vibramycin), but the CDC still recommends using amoxicillin (Amoxil) in pregnancy for the treatment of Lyme disease.

How well did you know this?
1
Not at all
2
3
4
5
Perfectly
110
Q

A parent presents with their two young children, both of whom have erythematous papules and burrows around their ankles and some tracking lines between their toes. Which of the following would be the best treatment for this condition?

A. Griseofulvin (Gris-PEG)
B. Permethrin (Nix)
C. Terbinafine (Lamisil)
D. Mebendazole (Emverm)

A

B. Permethrin (Nix)

This is a classic description of scabies, which is treated with permethrin (Nix). Scabies almost always presents between the fingers and toes, as well as around the lower ankles and feet. Scabies commonly affects everyone in the household at once. Griseofulvin (Gris-PEG) and terbinafine (Lamisil) are both antifungal medications commonly used to treat things such as ringworm or onychomycosis. Mebendazole (Emverm) is commonly used to treat enterobiasis, or pinworms.

How well did you know this?
1
Not at all
2
3
4
5
Perfectly
111
Q

A 37-year-old patient presents to the clinic today with painful clustered vesicles on an erythematous base near the lip. Which of the following is the most likely cause?

A. Herpes simplex virus type 1 (HSV-1)
B. Herpes simplex virus type 2 (HSV-2)
C. Human papillomavirus (HPV)
D. Coxsackievirus

A

A. Herpes simplex virus type 1 (HSV-1)

This description is most consistent with herpes simplex virus type 1 (HSV-1), or a cold sore. Herpes simplex virus type 2 (HSV-2) is most commonly the cause of genital herpes, although HSV-1 can also cause genital herpes from oral-genital exposure. Human papillomavirus (HPV) is commonly asymptomatic, or appears as genital warts. Coxsackievirus, also known as hand, foot, and mouth disease, usually occurs in children and appears as numerous vesicles around the affected areas.

How well did you know this?
1
Not at all
2
3
4
5
Perfectly
112
Q

A 43-year-old female presents to the office today for evaluation of what she believes to be persistent aphthous stomatitis. She reports that it has been present for the last two months and she has not found any relief with the usual symptomatic treatments. Upon further assessment, the nurse practitioner finds several large oral lesions and ulcers. Which of the following may have caused this?

A. Lichen sclerosus
B. Lichen simplex chronicus (LSC)
C. Coxsackievirus A16
D. Lichen planus

A

D. Lichen planus

This patient is suffering from chronic ulcerative stomatitis (CUS), which is commonly characterized by oral lesions that are larger in size and number that last weeks to months and do not respond to symptomatic treatment. While it is not fully understood, lichen planus has been linked to CUS as a known cause. Coxsackievirus A16 causes hand, foot, and mouth disease and usually occurs in children, and appears as multiple vesicles in the affected areas. Lichen sclerosus causes patchy and discolored skin, while lichen simplex chronicus (LSC) results from a chronic itch-scratch-itch cycle.

How well did you know this?
1
Not at all
2
3
4
5
Perfectly
113
Q

A 52-year-old male presents to the office today for evaluation of what he believes to be persistent aphthous stomatitis. He reports that it has been present for the last two months and he has not found any relief with the usual symptomatic treatments. Upon further assessment, the nurse practitioner finds several large oral lesions and ulcers. What would be the most appropriate treatment option to offer today?

A. Hydroxychloroquine (Plaquenil)
B. Nystatin (Mycostatin)
C. Diphenhydramine hydrochloride/Dexamethasone/Nystatin (Magic Mouthwash)
D. Hydrocortisone (Cortef)

A

A. Hydroxychloroquine (Plaquenil)

This patient is suffering from chronic ulcerative stomatitis (CUS), which is commonly characterized by oral lesions that are larger in size and number that last weeks to months and do not respond to symptomatic treatment. Hydroxychloroquine (Plaquenil) is the drug of choice for treatment. Nystatin (Mycostatin) is used to treat fungal infections in the mouth, such as candidiasis. Magic mouthwash, while used as symptomatic treatment for aphthous stomatitis, is not beneficial for CUS, as CUS requires treatment of the underlying problem. Hydrocortisone (Cortef) will not be beneficial for CUS, but can be used to treat a wide variety of conditions such as eczema.

How well did you know this?
1
Not at all
2
3
4
5
Perfectly
114
Q

An 80-year-old female, who is a new patient to the nurse practitioner, is brought to the office today by her caregiver. The caregiver reports that the patient has a past medical history of a traumatic brain injury (TBI) and is primarily bed bound at the living facility. While bathing the patient this morning, the caregiver noticed an area of increased redness on the patient’s heel. Upon further assessment, the nurse practitioner finds a 3 cm X 3 cm erythematous but intact area on the posterior heel. The area is non-blanchable. What treatment plan should the nurse practitioner recommend today?

A. A prescription for cephalexin (Keflex) as this is likely erysipelas

B. Pressure reduction accomplished by frequent position changes and a pressure relief ankle foot orthosis (PRAFO) boot as this is a stage II pressure injury

C. A prescription for sulfamethoxazole/trimethoprim (Bactrim) as this is suspicious for cellulitis

D. Daily foam dressing changes and use of a pressure relief ankle foot orthosis (PRAFO) boot as this is a stage I pressure injury

A

D. Daily foam dressing changes and use of a pressure relief ankle foot orthosis (PRAFO) boot as this is a stage I pressure injury

Since this patient is primarily bed bound, she is at an increased risk of pressure injury. Pressure injuries commonly occur in bony places such as the heels, elbows, and sacrum. Foam dressing changes and pressure reduction with PRAFO boots, along with frequent position changes are all beneficial in treating and preventing pressure injuries. Since the skin is intact, this is a stage I pressure injury. If the skin was broken open, this would then classify as a stage II pressure injury. Antibiotics are not helpful in treating these, unless an open sore becomes infected.

115
Q

The nurse practitioner is discussing next steps in treatment of acne with a 16-year-old and her mother. The patient has not been responding to her current treatment regimen of tretinoin (Retin-A). Which next step in treatment might the nurse practitioner suggest?

A. Topical clindamycin (Cleocin T)
B. Oral cephalexin (Keflex)
C. Oral doxycycline (Vibramycin)
D. A referral to dermatology for isotretinoin (Accutane)

A

C. Oral doxycycline (Vibramycin)

Acne treatment is commonly approached using a step up method. Patients should begin treatment with topical washes such as benzoyl peroxide. If this is unsuccessful, treatment will usually step up to topical antibiotics or tretinoin (Retin-A). Stepping up treatment from there would include a prescription for an oral antibiotic, such as doxycycline (Vibramycin) and lastly, a referral to dermatology for something stronger such as isotretinoin (Accutane) if needed.

116
Q

The nurse practitioner is evaluating a patient with a history of methicillin-resistant Staphylococcus aureus(MRSA) who presents today with a superficial, erythematous rash with well-demarcated borders on the anterior aspect of the left lower leg. Which of the following would be the most appropriate treatment option for this patient today?

A. Sulfamethoxazole/trimethoprim (Bactrim)
B. Doxycycline (Vibramycin)
C. Ciprofloxacin (Cipro)
D. Amoxicillin (Amoxil)

A

D. Amoxicillin (Amoxil)

This patient is suffering from erysipelas, or superficial cellulitis. The key finding in this question is that the erythema has a well-demarcated border. Erysipelas is best treated with a penicillin or cephalosporin, such as cephalexin (Keflex). Although the patient has a history of MRSA, there is not currently a known MRSA infection at this time, therefore sulfamethoxazole/trimethoprim (Bactrim) and doxycycline (Vibramycin) are not indicated. Ciprofloxacin (Cipro) is not commonly a first line treatment option for skin infections.

117
Q

A 38-year-old female presents with an acneiform facial rash covering the cheeks, nose, and nasolabial folds. Which of the following would be an appropriate diagnosis?

A. Lupus
B. Rosacea
C. Erysipelas
D. Cystic acne

A

B. Rosacea

The facial rash for rosacea and lupus present similarly; however, the malar rash associated with lupus spares the nasolabial folds while rosacea does not spare this area. Additionally, rosacea can be identified by its classic acneiform presentation. Erysipelas can occur on the face, but is most commonly described as having sharply demarcated borders. Cystic acne does not typically present as one large facial rash.

118
Q

A parent brings their 5-year-old son into the clinic with complaints that he is not able to distinguish between various colors on his school assignments. He also seems to have an increased sense of smell. Upon exam, his visual acuity is 20/40. Which of the following is the most appropriate action for the nurse practitioner to perform next?

A. Ishihara test
B. Peripheral visual field test
C. Tonometry
D. Fluorescein stain test

A

A. Ishihara Test

Color blindness is an often inherited color vision deficiency. Color blindness involves the inability to distinguish between shades of red and green. A common test used to diagnose color blindness is the Ishihara test, which is a vision test used for the detection of red-green color deficiencies.

119
Q

Which of the following eye complaints is most likely seen in a patient with diabetic retinopathy?

A. Cotton wool spots
B. AV nicking
C. Flame hemorrhages
D. Copper wire arterioles

A

A. Cotton wool spots

Common findings on a diabetic eye exam consist of cotton wool spots, microaneurysms, neovascularization, and possibly dot/blot hemorrhages. While there can be some overlap between diabetes and hypertension eye exam findings, cotton wool spots are most commonly seen in those with diabetes.

120
Q

A parent brings their 5-year-old son to the clinic with complaints of his right eye burning and redness over the last few days. Over the last 24 hours, the same symptoms have begun in his left eye. During the physical exam, the patient’s eyes appear reddened bilaterally with no purulent exudate. The patient’s throat is slightly erythematous, and he has clear nasal drainage. The patient has bilateral preauricular lymphadenopathy. Which of the following is the most likely differential diagnosis?

A. Bacterial conjunctivitis
B. Viral conjunctivitis
C. Corneal abrasion
D. Allergic conjunctivitis

A

B. Viral conjunctivitis

Viral conjunctivitis can present with or without a cold, flu, or other respiratory symptoms. It typically starts unilaterally and spreads to the other eye, typically within a few days. In viral conjunctivitis, discharge from the eye is normally described as watery. Preauricular lymph node involvement is also common with viral cases of conjunctivitis. Viral conjunctivitis treatment consists of symptomatic treatment and usually resolves on its own within 7-14 days.

121
Q

An 80-year-old patient comes in today for an annual exam. The nurse practitioner notes a triangular thickening of the conjunctiva on the left side by the nasal bridge that is noted to be encroaching on the cornea. She has no complaints of vision changes or eye discomfort and mentions being able to see fine as she is outside gardening. Which of the following is the most likely diagnosis?

A. Pinguecula
B. Pterygium
C. Arcus senilis
D. Xanthelasma

A

B. Pterygium

A pterygium is a benign non-cancerous overgrowth of the conjunctiva that encroaches on the cornea. A pterygium will start to clear on its own with no treatment. The key difference between a pterygium and a pinguecula is that with a pinguecula, the white overgrowth does not overlap the cornea. With a pterygium, the white triangular growth does cross over the cornea. Arcus senilis is a gray halo around the iris, typically caused by hyperlipidemia. Xanthelasma is another complication of hyperlipidemia and presents as yellow fat-like deposits on the eyelids.

122
Q

A 50-year-old patient presents to the nurse practitioner with a new onset of a severe headache and “flickering” vision. The nurse practitioner notes bilateral erythematous and swollen optic discs. Which of the following vital signs are most consistent with the likely diagnosis?

A. Blood pressure 194/99 mmHg, pulse 78, respirations 14, oxygen 99% on room air

B. Blood pressure 116/45 mmHg, pulse 80, respirations 14, oxygen 98% on room air

C. Blood pressure 145/56 mmHg, pulse 99, respirations 24, oxygen 97% on room air

D. Blood pressure 165/80 MMHg, pulse 50, respirations 23, oxygen 96% on room air

A

A. Blood pressure 194/99 mmHg, pulse 78, respirations 14, oxygen 99% on room air

Swelling of the optic discs, as described in this scenario, is called papilledema. Papilledema is an emergent complication that often occurs from extremely high blood pressure. Common symptoms of severely high blood pressure include severe headache, flickering vision, and a swollen optic disc. Papilledema requires immediate referral to the emergency department as it can result in permanent blindness if not treated.

123
Q

The nurse practitioner is seeing a 56-year-old female patient for a follow-up related to a recent episode of facial paralysis. The episode lasted a few weeks before spontaneous resolution. The patient now has concerns about blurry vision accompanied by pain and tearing in the left eye. Which of the following is an appropriate diagnostic test to perform at today’s visit?

A. Fluorescein stain
B. Fundoscopic exam
C. Snellen chart
D. Tonometry

A

A. Fluorescein stain

This patient’s presenting symptoms are most likely resulting from a corneal abrasion. A corneal abrasion is a tearing of the cornea which commonly presents with eye irritation, tearing, pain, and blurry vision. Patients with Bell’s palsy or facial weakness/paralysis are at an increased risk due to the increased dryness of the eyes, which this patient previously had symptoms of. Corneal abrasions are diagnosed with a fluorescein stain in which dye is placed into the eye and a blacklight is shone into the affected eye. Abrasions will take up the dye and glow when the blacklight is shone over the area.

124
Q

The nurse practitioner is assessing a patient using the Snellen chart. Which cranial nerve is the nurse practitioner assessing?

A. CN II
B. CN III
C. CN IV
D. CN VI

A

A. CN II

Cranial nerve II is the optic nerve, and is responsible for visual acuity. Visual acuity can be assessed using the Snellen chart. Cranial nerves III, IV, and VI are responsible for extraocular movements, not vision itself.

125
Q

A 23-year-old male patient is being seen by the nurse practitioner for an annual exam. Upon assessment, the nurse practitioner notes a gray halo around the iris of both eyes and yellow deposits near the inner eyelid. Which of the following questions would be pertinent to ask next?

A. Do you have a family history of hyperlipidemia?

B. Have you had any recent headaches or skull trauma?

C. Do you have any history of muscle weakness?

D. Do you have any visual changes?

A

A. Do you have a family history of hyperlipidemia?

Arcus senilis, which is a gray halo around the iris, and xanthelasma, which are yellow fatty deposits near the eyelid, are caused by increased cholesterol. It can be a benign finding in the elderly but is something that needs to be further investigated in the younger population. If found in the younger population, family history of serious hyperlipidemia should be inquired about, and a lipid panel obtained.

126
Q

An adolescent presents to the clinic with complaints of sudden localized swelling of the eyelid with purulent eye drainage. The nurse practitioner diagnosis the patient with a hordeolum. Which bacteria is the most likely cause?

A. Staphylococcus aureus
B. Pseudomonas aeruginosa
C. Streptococcus pneumoniae
D. Moraxella catarrhalis

A

A. Staphylococcus aureus

A hordeolum, also knows as a stye, is a common eye condition resulting in pain and swelling of the eyelid occasionally accompanied by purulent drainage from the infected area. A hordeolum is most often caused by a bacterial infection from Staphylococcus aureus.

127
Q

An 80-year-old patient comes into the office with concerns of a reddened area in his right eye for the last 2 days. He denies irritation, vision changes, or headache. He states he has been sneezing a lot due to his allergies. The only supplement he takes is ginkgo biloba which he uses to help his cognitive function. Which is a correct statement by the nurse practitioner about this condition?

A. “I am going to refer you to see an optometrist.”

B. “This will resolve on its own and no treatment is needed at this time.”

C. “This is an emergency and you need to go to get a CT scan as soon as possible.”

D. “I am going to prescribe you ciprofloxacin (Cipro) eye drops that you will take for the next two weeks.”

A

B. “This will resolve on its own and no treatment is needed at this time.”

This patient is presenting with a subconjunctival hemorrhage. Subconjunctival hemorrhages are typically asymptomatic, and appear as “bloody” or reddened areas in the eye. Subconjunctival hemorrhages resolve spontaneously. They can commonly occur in response to repetitive sneezing, vomiting, or facial pressure. Symptomatic treatment consists of eye drops, such as artificial tears.

128
Q

A 35-year-old male presents to urgent care today with complaints of a headache. He reports it started a few days ago and comes and goes. He has tried both ibuprofen (Advil) and acetaminophen (Tylenol) without improvement but states the headache does eventually go away. After further questioning the nurse practitioner concludes that it only occurs on the left side and is a “pounding” type pain. When it occurs, the patient also feels as if his vision is blurry. What is the top differential based on the patient’s symptoms?

A. Cluster headache
B. Tension headache
C. Migraine
D. Temporal arteritis

A

D. Temporal arteritis

Temporal arteritis presents as a pulsating, unilateral temple pain that can cause headaches, jaw pain, and can even lead to vision changes resulting in permanent blindness. The pain can come and go, but it is typically very severe in nature. Due to the inflammatory nature of the disorder, long-term, high-dose steroids are required for treatment.

129
Q

Which of the following vitamins is commonly deficient in alcoholics which can lead to Wernicke-Korsakoff Syndrome?

A. Vitamin B12
B. Vitamin B7
C. Vitamin B1
D. Folic Acid

A

C. Vitamin B1

Wernicke- Korsakoff Syndrome is due to the deficiency of vitamin B1, also known as thiamine. This vitamin B1 deficiency ultimately leads to damage to both the thalamus and hypothalamus causing confusion, ataxia, and vision abnormalities.

130
Q

The nurse practitioner is seeing a 42-year-old female for routine follow up. When asked about her new diagnosis, the patient mentions her neurologist recently diagnosed her with Ménière’s disease. What is the nurse practitioner’s top concern with this new diagnosis?

A. Permanent hearing loss
B. Short-term hearing loss
C. Degradation of the tympanic membrane
D. Permanent vision loss

A

A. Permanent hearing loss

Ménière’s disease is a disorder of the inner ear caused by an abnormality of the labyrinth. This leads to both vertigo and hearing loss. The main concern for this diagnosis would be progression to permanent hearing loss due to the degradation of the cochlea, which is responsible for relaying sounds to the brain for hearing.

131
Q

The nurse practitioner is performing the Weber test on a patient with a suspected cholesteatoma. If the patient does have a cholesteatoma, which of the following would be the expected response to the Weber test?

A. The patient will be able to hear equally in both ears

B. There will be lateralization to the bad ear

C. There will be lateralization to the good ear

D. The patient will not be able to hear anything at all

A

B. There will be lateralization to the bad ear

Cholesteatomas are described as a collection of skin cells within the ear. They often cause a large mass in the inner ear that requires surgical removal. This condition can lead to conductive hearing loss. During the Weber test, if a patient is experiencing conductive hearing loss there will be lateralization to the bad ear, in this case, the ear with the cholesteatoma.

132
Q

The nurse practitioner is following up with an older male patient who has been diagnosed with Alzheimer’s disease. His wife says his diagnosis has been hard lately as he no longer recognizes who his grandchildren are. Which of the following cognitive dysfunction describes this phenomenon?

A. Aphasia
B. Agnosia
C. Apraxia
D. Agraphia

A

B. Agnosia

The inability to not recognize familiar faces and people is characterized as agnosia. Apraxia is the inability to carry out movements or gestures, agraphia refers to the inability to write, and aphasia refers to the inability to comprehend things such as reading, writing, or expression by others.

133
Q

Which of the following cranial nerves is responsible for raising the eyebrows?

A. Cranial Nerve I
B. Cranial Nerve V
C. Cranial Nerve VIII
D. Cranial Nerve VII

A

D. Cranial Nerve VII

Cranial nerve VII, the facial nerve, is responsible for facial expression such as raising the eyebrows, puffing out the cheeks, and smiling.

134
Q

The nurse practitioner is educating the family members of a patient recently diagnosed with Parkinson’s disease. Which of the following is appropriate education?

A. Parkinson’s disease is a degenerative disease that is caused by a depletion of serotonin

B. Parkinson’s disease is a degenerative disease that is caused by a depletion of endorphins

C. Parkinson’s disease is a degenerative disease that is caused by a depletion of oxytocin

D. Parkinson’s disease is a degenerative disease that is caused by a depletion of dopamine

A

D. Parkinson’s disease is a degenerative disease that is caused by a depletion of dopamine

Parkinson’s disease occurs when there is a depletion of the neurotransmitter dopamine leading to degradation of the basal ganglia leading to progressive symptoms affecting the nervous system. Oxytocin, endorphins, and serotonin are not typically affected in this disease process.

135
Q

A 36-year-old male presents to the clinic with a headache. He reports it started this morning and has gotten progressively worse. His vital signs today are: BP: 146/88 HR: 88 O2: 99% RR: 16 Temp: 98F. His current medications include lisinopril (Zestril) 20 mg daily and fluoxetine (Prozac) 40 mg daily. Which of the following would be the most appropriate next step for the nurse practitioner?

A. Advise him to avoid triggers and take ibuprofen (Advil) 600 mg every 4-6 hours

B. Prescribe sumatriptan (Imitrex)

C. Increase his lisinopril (Zestril) to 30 mg daily

D. Prescribe propranolol (Inderal) 20 mg daily

A

C. Increase his lisinopril (Zestril) to 30 mg daily

Due to this patient’s symptoms and his increased blood pressure this is likely a hypertensive headache. The best option for this patient would be increasing his lisinopril (Zestril) to 30mg daily. Additionally, due to his use of fluoxetine (Prozac) it is important to note that starting sumatriptan (Imitrex) would be contraindicated due to the risk of serotonin syndrome. Ibuprofen (Advil) is not likely to help as it will not lower his blood pressure. Propranolol (Inderal) may help, but is not the best option in this scenario for management of his blood pressure.

136
Q

The nurse practitioner is assessing a patient who presented to the clinic with a fever of 102℉, a stiff and sore neck, and persistent nausea and vomiting. The nurse practitioner suspects meningitis and performs Brudzinski’s sign to help make his diagnosis. Which of the following is the appropriate way to perform Brudzinski’s sign?

A. The nurse practitioner has the patient lay on their back with their hips and knees flexed and bent at 90 degrees. Slowly, the nurse practitioner tries to extend one knee at a time

B. The nurse practitioner places his hand on the back of the patient’s neck while eliciting passive flexion of the neck

C. The nurse practitioner has the patient lay on their back with both the hip and knee flexed at 90 degrees. The NP then internally rotates the hip by moving the patient’s ankle away from the body

D. The nurse practitioner has the patient lay on their side with their knees extended. The NP then places their hand on the patient’s right thigh, passively extending the hip

A

B. The nurse practitioner places his hand on the back of the patient’s neck while eliciting passive flexion of the neck

Both Brudzinski’s sign and Kernig’s sign are helpful in making the diagnosis of meningitis. Bruzinkski’s sign is performed by placing the hand on the back of the patient’s neck while eliciting passive flexion of the neck. A positive sign would lead to the patient’s hips and knee flexing while the neck is flexed. When this sign is positive, the patient should be referred for an emergent spinal tap and admission due to suspected meningitis.

137
Q

The nurse practitioner is providing education to the family of an 80-year-old female recently diagnosed with Alzheimer’s disease. Which of the following statements made by the family member reveals a good understanding of the Alzheimer’s diagnosis?

A. “There are many treatments for my mother, and she will get better soon.”

B. “We will set up hospice soon as we know she will pass in the next few months.”

C. “There is no cure for this diagnosis, only medications to slow the progression and we will focus on her quality of life.”

D. “With the right cognitive therapy we will be able to maintain her long-term memory for a long time”

A

C. “There is no cure for this diagnosis, only medications to slow the progression and we will focus on her quality of life.”

The statement that shows that the family members of a patient diagnosed with Alzheimer’s disease understand the disease process would include their knowledge of the fact that there is no cure for the diagnosis. There are medications that can slow the progression but there is, unfortunately, not any medication that can treat the disease process itself.

138
Q

The nurse practitioner is reviewing a patient’s screening questionnaire at their annual wellness exam. Upon review, the patient has a positive patient health questionnaire-2 (PHQ-2). What would be the next appropriate questionnaire to give to the patient to fill out?

A. Generalized Anxiety Disorder-7 (GAD-7)

B. CAGE Questionnaire

C. Patient health questionnaire-9 (PHQ-9)

D. Beck Depression Inventory (BDI)

A

C. Patient health questionnaire-9 (PHQ-9)

he GAD-7 is incorrect as this is the Generalized Anxiety Disorder scale and would not be the next step after the PHQ-2, which screens for depression. The CAGE questionnaire is a questionnaire to assess alcohol use. The PHQ-9 is correct as it is the next step to the PHQ-2, and it is a more in-depth depression questionnaire. The BDI is a questionnaire for depression, but not the next step in this situation as the PHQ-9 should always follow a positive PHQ-2 screening.

139
Q

A 26-year-old female patient presents to the clinic with diarrhea since starting sertraline (Zoloft) five days ago. She reports taking this medication as prescibed and other than the diarrhea, is having no other symptoms. What would be the appropriate next step?

A. Decrease the sertraline (Zoloft) dose until the gastrointestinal side effects subside

B. Start a new SSRI to mitigate the gastrointestinal side effects

C. Educate the patient that this is a common side effect and should subside without intervention

D. Instruct the patient to take this medication with orange juice to increase absorption and decrease gastrointestinal distress

A

C. Educate the patient that this is a common side effect and should subside without intervention

Gastrointestinal side effects are common side effects when starting SSRIs such as sertraline (Zoloft). The patient should be educated that these side effects usually subside within a few weeks, and to remain well hydrated. Decreasing the dose or starting a new SSRI would not be the appropriate action at this time as it typically takes 4-6 weeks to see full effects of SSRIs and for the gastrointestinal side effects to subside. Taking this medication with orange juice is not an appropriate action.

140
Q

A 20-year-old male patient presents to the clinic with concerns of feeling “down” and exhaustion. He reports an inability to concentrate and recently lost his job. Based on the symptoms the patient has described, what the main differential diagnosis the nurse practitioner should consider?

A. Multiple Personality Disorder

B. Schizoaffective disorder

C. Bipolar disorder

D. Attention-Deficit-Hyperactivity-Disorder (ADHD)

A

C. Bipolar disorder

Bipolar disorder consists of emotional highs and lows. Emotional lows due to bipolar disorder are oftentimes confused for depression, so nurse practitioners should always ask patients about all of their symptoms to form a full diagnosis. This patient is showing classic symptoms of emotional highs such as inability to concentrate, as well as lows with his symptoms of exhaustion and feeling “down”.

141
Q

Treatment of seasonal affective disorder includes which of the following?

A. Seasonal use of benzodiazepines such as lorazepam (Ativan)

B. Exposure to sunlight

C. Daily exercise

D. Use of selective serotonin reuptake inhibitors such as sertraline (Zoloft)

A

B. Exposure to sunlight

Seasonal affective disorder is a type of depression related to the lack of sunlight in the winter months which causes patients to have a change in their circadian rhythm. Approved treatments include exposure to more real or artificial sunlight to help reset the patient’s circadian rhythm, as well as talk therapy.

142
Q

The nurse practitioner is looking over the lab results of a 47-year-old male patient who is currently taking lisinopril (Prinivil), hydroxychloroquine (Plaquenil), probenecid (Probalan), a once a day multivitamin, and quetiapine (Seroquel). The patient’s lab results show that the patient’s Hemoglobin A1C is 6.3% and his triglyceride level is 250 mg/dL. What medication does the nurse practitioner suspect could be contributing to these lab results?

A. Lisinopril (Prinivil)
B. Hydroxychloroquine (Plaquenil)
C. Multivitamin
D. Quetiapine (Seroquel)

A

D. Quetiapine (Seroquel)

Quetiapine is an atypical antipsychotic. Atypical antipsychotics can increase lipid levels and blood glucose levels. They may in turn cause weight gain. For these reasons, atypical antipsychotics are not typically prescribed to patients with a history of these issues or metabolic syndrome.

143
Q

A 27-year-old female patient presents to the clinic with complaints of recurring nightmares, feeling hypervigilant, and feeling extremely anxious. Upon reviewing her chart, the nurse practitioner sees that the patient has openly spoken about a history of physical abuse at prior visits. Based on these symptoms, what differential should the nurse practitioner consider for this patient?

A. Generalized anxiety disorder
B. Major depressive disorder
C. Social anxiety disorder
D. Post-traumatic stress disorder

A

D. Post-traumatic stress disorder

This patient is exhibiting tell-tale signs of PTSD, including recurring nightmares, anxiety, and hypervigilance. PTSD affects females at higher rates.

144
Q

The nurse practitioner diagnoses a patient with post-traumatic stress disorder. What is the first line medication class that is typically used?

A. Serotonin norepinephrine reuptake inhibitors (SNRIs)

B. Tricyclic Antidepressants (TCAs)

C. Benzodiazepines

D. Selective serotonin reuptake inhibitors (SSRIs)

A

D. Selective serotonin reuptake inhibitors (SSRIs)

SSRIs are the medication class of choice for a patient with post-traumatic stress disorder. The typical SSRIs of choice are escitalopram (Lexapro) or fluoxetine (Prozac), although others may be used as well.

145
Q

A 45-year-old male patient of Asian descent presents to the clinic with complaints of continuous back pain. The nurse practitioner has recently prescribed tramadol (Ultram) for symptom relief and he states he has taken the medication as prescribed with no improvement. The nurse practitioner knows which enzyme may be contributing to this medication’s decreased overall effectiveness?

A. CYP3A4
B. CYP2D6
C. CYP2C19
D. CYP1A2

A

C. CYP2C19

Patients of Asian descent can have the CYP2C19 enzyme, which leads to their bodies poorly metabolizing certain pain medications such as tramadol (Ultram). It is imperative to do a thorough assessment and evaluation to determine the best plan of care for patients with this enzyme moving forward as they may require higher doses of medication, or different medications altogether.

146
Q

Which selective serotonin reuptake inhibitor (SSRI) would the nurse practitioner consider safest for a 75-year-old patient diagnosed with depression?

A. Fluoxetine (Prozac)
B. Paroxetine (Paxil)
C. Vilazodone (Viibryd)
D. Escitalopram (Lexapro)

A

D. Escitalopram (Lexapro)

Of the medications listed, escitalopram (Lexapro) is the safest antidepressant to prescribe to the elderly due to its low side effect profile. Fluoxetine (Prozac) is not an ideal option as it often causes patients to feel jittery, which may lead to falls in the elderly. Paroxetine (Paxil) is very sedating and can have prolonged effects in the elderly. Vilazodone (Viibryd) is also not a great option, as it can cause hyponatremia in elderly patients.

147
Q

The nurse practitioner is seeing a 25-year-old male patient who was prescribed paroxetine (Paxil) 5 weeks ago. The patient states that overall he is feeling much better in regards to his depression. However, he states that he is still having issues with fatigue throughout the day. Which of the following lab orders would be the most appropriate to evaluate the patient’s concerns further?

A. Lipid panel

B. Urinalysis

C. Thyroid stimulating hormone (TSH)

D. Complete metabolic panel (CMP)

A

C. Thyroid stimulating hormone (TSH)

Thyroid stimulating hormone (TSH) should be drawn in this patient to assess for hypothyroidism as this patient is still having symptoms of fatigue that is not resolving with medication. Fatigue is a common sign of possible hypothyroidism. A lipid panel, urinalysis, or complete metabolic panel (CMP) are not indicated at this time.

148
Q

How often should a lipid profile be ordered on a healthy adult patient?

A. Every 6 months
B. Annually
C. Every 5 years
D. Every 3 months

A

C. Every 5 Years

According to the USPSTF, in a healthy individual with no underlying risk factors, a lipid panel can be ordered every 5 years. Risk factors would include an increased BMI, family history of cardiovascular disease, or smoking.

149
Q

According to the JNC 8 guidelines, at which blood pressure reading would the nurse practitioner start pharmacologic treatment for hypertension in patients greater than 60 years old?

A. 130/80 mmHg
B. 120/80 mmHg
C. 150/90 mmHg
D. 140/90 mmHg

A

C. 150/90 mmHg

According to the JNC 8 Guidelines, individuals who are older than 60 years should have a goal blood pressure of <150/90 mmHg. Therefore, anything greater than 150/90 mmHg would initiate treatment after 3 months of lifestyle modifications. In contrast, AHA guidelines have a goal blood pressure of 130/80 mmHg.

150
Q

A 78-year-old female with a past medical history of falls and osteoporosis is being seen for a 3-month follow-up for hypertension after an initiation of lifestyle modifications. Her blood pressure today is 144/92 mmHg. How should the nurse practitioner treat the patient in accordance with the AHA/ACC hypertension guidelines?

A. Start the patient on verapamil (Verelan)

B. Continue lifestyle modifications

C. Start the patient on chlorothiazide (Diuril)

D. Start the patient on lisinopril (Zestril)

A

C. Start the patient on chlorothiazide (Diuril)

This patient needs treatment with antihypertensives as she is in stage 2 hypertension according to the AHA/ACC hypertension guidelines. An ACE-I like lisinopril (Zestril), a calcium channel blocker like verapamil (Verelan) or thiazide diuretic like chlorothiazide (Diuril) could be used as first-line treatment. However, given this patient’s history of osteoporosis, a thiazide diuretic would be the better choice as it stimulates the osteoblasts to build bone and can improve bone density.

151
Q

A 67-year-old male presents to the clinic today for concerns with ongoing heartburn. He has a past medical history of hypertension and diabetes. When reviewing his medications, what medication does the nurse practitioner suspect is contributing to the patient’s concern?

A. Omeprazole (Pepcid)
B. Calcium carbonate (Tums)
C. Nifedipine (Procardia)
D. Glipizide (Glucotrol)

A

C. Nifedipine (Procardia)

Calcium channel blockers are generally avoided in patients with GERD, also sometimes called “heartburn,” due to the medication relaxing the esophageal sphincter, which then can cause stomach acid to regurgitate, worsening symptoms.

152
Q

A 47-year-old male presents to the clinic today for his annual exam. He is concerned about his family history of cardiovascular disease as his dad was 44-years-old when he had a myocardial infarction. The patient denies any current medications, and has not had medical care for several years. What would be the nurse practitioner’s first step in this patient’s plan of care today?

A. Start on low dose statin therapy

B. Start on high dose statin therapy

C. Order a lipid profile

D. Encourage lifestyle modifications to prevent myocardial infarction.

A

C. Order a lipid profile

The nurse practitioner would need to order a lipid panel to determine if the patient is an appropriate candidate for statin therapy. Given the little information about the patient, further work up would determine the next course of action in the patient’s treatment plan.

153
Q

An 82-year-old patient came into the clinic today with complaints of pitting edema. His latest echocardiogram results showed an ejection fraction of 40%. Which of the following conditions would the nurse practitioner suspect?

A. Right-sided heart failure
B. Peripheral arterial disease (PAD)
C. Left-sided heart failure
D. Venous insufficiency

A

A. Right-sided heart failure

Right sided heart failure causes fluid retention in the periphery of the body, especially the extremities. Left sided heart failure is more likely to affect the lungs, and frequently has respiratory symptoms associated with it.

154
Q

The nurse practitioner is auscultating a patient’s heart and hears a split S2. When is a split S2 considered a physiologic finding?

A. During expiration
B. In young athletes
C. Pregnancy
D. During inspiration

A

D. During inspiration

The S2 heart sound is the result of the closure of the aortic and pulmonic valves, which causes vibration of the valve leaflets and the adjacent structures. Therefore, if heard during inspiration, it is considered a normal physiological finding.

155
Q

A patient with a history of heart failure was started on spironolactone (Aldactone). Which lab value should be monitored closely?

A. B-type natriuretic peptide (BNP)
B. Sodium
C. Potassium
D. Creatinine

A

C. Potassium

Spironolactone (Aldactone) is a potassium sparing diuretic that can increase the potassium in the blood, therefore, patients need to be educated on signs and symptoms of hyperkalemia.

156
Q

A patient calls into the clinic stating that they are having a nosebleed that will not stop. They have tried both pressure and ice over the last half hour. The patient states she takes coumadin (Warfarin) daily for her atrial fibrillation She last had her INR checked 3 days ago and it was 3.4. What is the priority action by the nurse practitioner?

A. Instruct the patient to go to the emergency department

B. Order intravenous (IV) vitamin K

C. Have the patient be seen in the clinic today

D. Instruct the patient to hold today’s dose of coumadin (Warfarin)

A

A. Instruct the patient to go to the emergency department

A normal INR for a patient with atrial fibrillation is 2-3, any higher value would indicate a need for a reduction of dosage of coumadin (Warfarin), a dose needing to be held, or an order for vitamin K. Since the patient has an INR greater than 3 and she is actively bleeding despite intervention, the patient should be instructed to present to the emergency department for further evaluation and likely vitamin K administration.

157
Q

A patient presents to the clinic for a follow up on the following lab values: Total cholesterol 445, LDL 289, HDL 30, and Triglycerides 208. The patient has a past medical history of tobacco use, stable angina, and obesity. What would be the nurse practitioner’s best plan of treatment?

A. Start atorvastatin (Lipitor)
B. Encourage lifestyle modifications for 3 months
C. Start fenofibrate (Tricor)
D. Start simvastatin (Zicor)

A

A. Start atorvastatin (Lipitor)

A moderate or high-intensity statin would be appropriate for this patient. The USPSTF recommends that clinicians prescribe a statin for the primary prevention of CVD for adults aged 40 to 75 years who have 1 or more CVD risk factors (i.e. dyslipidemia, diabetes, hypertension, or smoking) and an estimated 10-year risk of a cardiovascular event of 10% or greater. Always encourage lifestyle modifications to patients, such as smoking cessation, weight loss, and regular physical exercise.

158
Q

A 35-year-old male presents to the clinic today to establish care. His past medical history includes asthma and a tonsillectomy. When asked about his asthma, he states that it is well controlled and he very rarely has to use his rescue inhaler. After further investigation, the nurse practitioner diagnoses him with mild intermittent asthma. Which medication would the nurse practitioner prescribe as needed for symptom relief?

A. Salmeterol (Serevent)

B. Beclomethasone dipropionate (Qvar RediHaler)

C. Budesonide/formoterol (Symbicort)

D. Montelukast (Singulair)

A

C. Budesonide/formoterol (Symbicort)

This is an inhaled corticosteroid (ICS)/long acting beta agonist (LABA) combination and the preferred as needed (PRN) medication for asthma patients. This patient has been diagnosed with mild intermittent asthma. Patients with mild intermittent asthma receive Step 1 treatment according to the most recent guidelines and should be prescribed a low-dose daily inhaled corticosteroid as well as a LABA PRN. Salmeterol (Serevent) is a long-acting beta-agonist, which should never be given alone to patients with asthma. Beclomethasone dipropionate (Qvar RediHaler) is a low-dose daily inhaled corticosteroid, which is what this patient would be prescribed for daily use if they progressed to Step 2 treatment. Montelukast (Singulair) is a leukotriene receptor antagonist (LRTA) that could be prescribed as a daily control medication, but not prescribed PRN for symptom relief.

159
Q

A 24-year-old patient presented to urgent care today for 8 days of cough, fatigue, fever, and mild shortness of breath. He denies any medical history other than a penicillin allergy and being treated for strep throat a few weeks ago. His lung sounds reveal rhonchi and a chest x-ray reveals consolidation in the right lower lobe. What is the best course of treatment for this patient?

A. Azithromycin (Zithromax)

B. Doxycycline (Vibramycin)

C. Amoxicillin-clavulanate (Augmentin) and azithromycin (Zithromax)

D. Levofloxacin (Levaquin)

A

D. Levofloxacin (Levaquin)

This patient’s symptoms, assessment, and diagnostics findings are consistent with a diagnosis of pneumonia. Azithromycin (Zithromax) and doxycycline (Vibramycin) can be used for treatment of pneumonia for healthy patients with no recent antibiotic use or comorbidities. However, he has recently been on an antibiotic so we would not want to utilize these. Amoxicillin-clauvuante (Augmentin) + azithromycin (Zithromax) combo may be a good option for this pneumonia, however he is allergic to penicillin. Levofloxacin (Levaquin) would be the best option due to his recent antibiotic use and needing a stronger class of antibiotics.

160
Q

A 45-year-old patient presents to the clinic for a tuberculosis PPD skin test read. He reports recently moving to the United States from Mexico. What induration would indicate a positive skin test for this patient?

A. 4 mm induration
B. 6 mm induration
C. 9 mm induration
D. 12 mm induration

A

D. 12 mm induration

Since this patient has recently immigrated, a positive TB skin test would be greater than 10 mm. For the general public a positive TB skin test would be greater than 15 mm. If a patient has HIV, is immunocompromised, or has had recent contact with a person with tuberculosis, a positive test would be greater than 5 mm.

161
Q

An 8-year-old patient presents to the clinic accompanied by his mother. She reports that on and off over the past month he has been sick and just can’t seem to get better. She reports that he isn’t eating as much as he used to and is very fatigued most days. On exam, you see pharyngeal erythema and palatine petechiae. Which of the following is not a complication associated with the likely diagnosis if it goes untreated?

A. Glomerulonephritis
B. Morbilliform Rash
C. Rheumatic Fever
D. Scarlatina

A

B. Morbilliform Rash

Group A Strep throat is commonly diagnosed in children. Common signs and symptoms include sore throat, swollen lymph nodes, red enlarged tonsils, white patches on tonsils, fever, and palatal petechiae. Glomerulonephritis, rheumatic fever, and scarlatina are all complications of untreated Strep A, which can affect the heart and kidneys. A morbilliform rash is associated with prescribing of certain penicillins with concurrent Strep A and mononucleosis diagnoses.

162
Q

A 55-year-old male patient presents to the clinic for a chronic productive cough and dyspnea. He reports a 15-year history of smoking. What lab finding is most consistent with this diagnosis?

A. FEV <60%
B. FEV <80%
C. FVC/FEV1 Ratio < 0.70
D. FEV1/FVC Ratio < 0.70

A

D. FEV1/FVC Ratio < 0.70

These findings, in combination with the patient’s history, are consistent with COPD. A chronic productive cough, seen in chronic bronchitis, and emphysema are the two conditions that make up COPD. Other signs and symptoms can include a barrel chest, clubbing of fingers, dyspnea, or fatigue. FEV1/FVC Ratio < 0.70 is diagnostic of COPD.

163
Q

A 71-year-old female presents to the office today with complaints of cough, fatigue, and fever over the past 8 days. After evaluation it is determined that she has pneumonia. She is alert and oriented to person, place, and time, BUN level is 22 mg/dL, blood pressure 124/75 and respiratory rate 19. The nurse practitioner determines if this patient requires hospitalization. How many points does this patient score according to the CURB-65 criteria?

A. 0
B. 1
C. 2
D. 3

A

C. 2

The CURB-65 criteria is used to help providers in decision making regarding hospitalization in pneumonia patients 65 years or older. C stands for confusion. This patient is alert and oriented therefore does not score a point. U stands for urea, which is measured by the BUN. A BUN greater than 19 mg/dL qualifies as a point, therefore this patient meets this criteria. R stands for respiratory rate and, depending on what guidelines utilized, can score a point if greater than 20 or 30. However, this patient clearly has a respiratory rate within normal limits. B stands for blood pressure. A blood pressure reading of less than 90/60 would score a point, but this patient’s blood pressure is greater than this limit. Lastly, the patient is 71 years old, therefore this would be 1 point as well. This patient’s total score is a 2, therefore she will possibly require admission to the hospital. A score 3 or greater requires admission.

164
Q

A 27-year-old male presents to the clinic today with complaints of nasal congestion, productive cough, facial pressure, and a slight toothache that has been worsening over the past 12 days. Which of the following is the most likely diagnosis?

A. Acute bronchitis
B. Acute bacterial sinusitis
C. Acute upper respiratory infection
D. Allergic rhinitis

A

B. Acute bacterial sinusitis

This patient’s upper respiratory symptoms, facial pressure, and toothache are symptoms of sinusitis. Due to symptoms persisting for greater than 10 days, we will treat bacterial sinusitis with an antibiotic, such as amoxicillin/clavulanate (Augmentin). With acute bronchitis we would not see symptoms such as facial pressure and toothache. An acute viral upper respiratory infection would likely be diagnosed prior to 12 days. Allergic rhinitis may include symptoms such as sneezing and runny nose.

165
Q

A 17-year-old patient presents today for a persistent sore throat. She was seen at the clinic a week ago and diagnosed with acute nasopharyngitis and has been utilizing symptomatic treatment. On the exam her tonsils are erythematous. The right tonsil is 3+ and the left is 1+. Her uvula is enlarged and slightly deviated to the left and you note foul smelling breath. Which of the following is the most likely diagnosis?

A. Acute Pharyngitis
B. Peritonsillar Abscess
C. Mononucleosis
D. Tonsillitis

A

B. Peritonsillar Abscess

A peritonsillar abscess can form from an untreated bacterial infection, such as group A strep. Clinical findings would include enlarged erythematous tonsils and a deviated uvula. Ensuring prompt treatment and referral is essential for this patient. Acute pharyngitis is a non-specific diagnosis and could indicate a viral or bacterial cause for the sore throat. Mononucleosis is a viral infection, therefore, we would not suspect an abscess. Tonsillitis is another non-specific diagnosis that would warrant further investigation.

166
Q

A 70-year-old patient presents to the clinic with complaints of sneezing, itchy eyes, and runny nose that is worse when she is around animals. She has been avoiding animals but reports she is still experiencing symptoms. What is the best treatment option for this patient?

A. Intranasal corticosteroid
B. Intranasal antihistamine
C. Oral antihistamine
D. Oral corticosteroid

A

A. Intranasal corticosteroid

After avoiding allergy triggers, the first-line treatment for allergic rhinitis is intranasal corticosteroids, such as fluticasone (Flonase). An oral antihistamine is the second-line treatment for allergic rhinitis. However, we would want to avoid them with this patient due to the patient’s age and the unfavorable anticholinergic effects. Intranasal antihistamines and oral corticosteroids can be utilized in unresponsive allergic rhinitis.

167
Q

A 21-year-old patient is completing a peak flow reading in the clinic today. Which of the following is the best predictor of lung capacity for this patient?

A. Age
B. Gender
C. Height
D. Ethnicity

A

C. Height

Height, age, and gender impact peak flow reading results. Height is the most important factor in the adult population.

168
Q

The nurse practitioner is following up with a patient who was recently diagnosed with Cushing’s syndrome. Which of the lab results would best fit this diagnosis?

A. Potassium level of 2.1 mmol/L
B. Potassium level of 5.8 mmol/L
C. Sodium level of 128 mmol/L
D. Sodium level of 149 mmol/L

A

A. Potassium level of 2.1 mmol/L

Cushing’s syndrome typically presents with a low serum potassium level. This is due to the increased stimulation of mineralocorticoid receptors in the kidney causing increased sodium absorption leading to hypokalemia. A normal potassium level is generally considered 3.5-5 mmol/L.

169
Q

A 27-year-old female presents to the clinic today as a new patient. She reports she is constantly fatigued and often feels like she “crashes” in the afternoon. Her past medical history includes depression and steroid use for severe asthma. What should the nurse practitioner further assess for?

A. Cushing’s syndrome
B. Addison’s Disease
C. Reactive hypoglycemia
D. Diabetes Mellitus Type 1

A

B. Addison’s Disease

Due to this patient’s symptoms of fatigue and history of chronic steroid use, the most likely differential would be Addison’s disease. With chronic steroid use, the hypothalamic-pituitary- adrenal axis is disrupted, limiting the adrenal glands and impeding their ability to properly produce cortisol, which in turn can lead to Addison’s disease.

170
Q

The nurse practitioner is reviewing a continuous glucose monitor (CGM) report. Upon review, the nurse practitioner notes a persistent rise in the patient’s blood glucose from 12AM-8AM for the last seven days. What is the patient likely experiencing?

A. Reactive hyperglycemia
B. Dawn phenomenon
C. Somogyi effect
D. Reactive hypoglycemia

A

B. Dawn phenomenon

Dawn phenomenon occurs when there is an early morning rise of blood glucose levels which most commonly occurs around 3-4AM but can occur any time in the patient’s nighttime sleeping hours. This can be avoided by increasing the patient’s nighttime insulin to keep blood glucose levels steady throughout the night.

171
Q

Which of the following specialties would a patient with systemic lupus erythematosus most likely need a referral to?

A. Cardiology
B. Nephrology
C. Ophthalmology
D. Endocrinology

A

B. Nephrology

A patient with a diagnosis of systemic lupus erythematosus, also known as lupus, would most likely need a referral to a nephrologist. Between 30-50% of patients diagnosed with lupus will eventually develop lupus nephritis. During this disease process, lupus autoantibodies attack the kidneys, ultimately leading to the kidneys inability to properly filter blood and excrete waste.

172
Q

An 81-year-old male presents to the office for a follow up appointment to talk about his type 2 diabetes mellitus. His current regimen includes metformin (Glucophage) 1000 mg orally twice a day and pioglitazone (Actos) 45 mg orally three times a day. His most recent Hgb A1C was 8.7%. Which of the following medications would NOT be appropriate to add today?

A. Semaglutide (Ozempic)
B. Glipizide (Glucotrol)
C. Empagliflozin (Jardiance)
D. Sitagliptin (Januvia)

A

B. Glipizide (Glucotrol)

Glipizide (Glucotrol) belongs to the sulfonylurea drug class, which has a high risk for hypoglycemia and weight gain. Given the patient’s age, the risk of hypoglycemia and subsequent falls would not make this medication the drug of choice. Due to the patient’s age, a more appropriate drug may be semaglutide (Ozempic), empagliflozin (Jardiance), or sitagliptin (Januvia) to lower his risk of falls and hypoglycemic episodes.

173
Q

Which of the following patients would glucophage (Metformin) be contraindicated in?

A. A 70-year-old male with a history of hypertension

B. A 64-year-old female with a glomerular filtration rate of 46

C. A 32-year-old female with a history of pancreatitis

D. A 45-year-old male with a history of cirrhosis of the liver

A

D. A 45-year-old male with a history of cirrhosis of the liver

Glucophage (Metformin) would be contraindicated in a 45-year-old male with a history of cirrhosis due to the increased risk of lactic acidosis. Although glucophage (Metformin) should be used with caution in those with renal impairment, it is not contraindicated in those with a glomerular filtration rate (GFR) of 46, however, their dose should be cut in half and the medication should be discontinued if their GFR drops less than 30. Glucophage (Metformin) is not contraindicated in those with hypertension or pancreatitis.

174
Q

A 23-year-old female presents to the clinic today for a follow up for her hypothyroidism. She is currently taking levothyroxine (Synthroid) 88 mcg daily. She reports she takes it every morning on an empty stomach separately from food. She reports she is feeling well today and has not noticed any side effects. Her thyroid function tests come back with the following results: TSH: 0.25 Free T4: 1.2 Free T3: 3.1. Which of the following responses would be the most appropriate to further assess her low thyroid stimulating hormone?

A. Do you take an iodine supplement?

B. Have you been sick in the past month?

C. Do you take a daily multivitamin?

D. How are you taking your levothyroxine?

A

D. How are you taking your levothyroxine?

This patient has a history of hypothyroidism which is being treated with levothyroxine (Synthroid). However, on her lab results, her thyroid stimulating hormone (TSH) is low, suggesting there may be too much thyroid hormone on board. It is most appropriate to assess how the patient is taking her medication. Some components of multivitamins, such as vitamin B7 (Biotin), can cause affect levothyroxine absorption making it less effective.

175
Q

A patient newly diagnosed with hyperthyroidism wants to know if it is safe for her to conceive. How should the nurse practitioner respond?

A. “No, it is not safe at this time. You will need to wait until you have a thyroidectomy to conceive.”

B. “Yes, it is safe. However you will need to be on propylthiouracil (PTU) during the first trimester, and then we can switch to methimazole (Tapazole) during the second and third trimester.”

C. “Yes, it is safe. However, you will need to be on methimazole (Tapazole) during the first trimester, and then we can switch to propylthiouracil (PTU) during the second and third trimester.”

D. “No, it is not safe at this time. However if you would like to proceed with radioactive iodine ablation it will be safe for you to conceive following the procedure.”

A

B. “Yes, it is safe. However you will need to be on propylthiouracil (PTU) during the first trimester, and then we can switch to methimazole (Tapazole) during the second and third trimester.”

During pregnancy, hyperthyroidism should be treated with propylthiouracil (PTU) during the first trimester, followed by methimazole (Tapazole) during the second and third trimester. Radioactive iodine ablation and surgery are not recommended during pregnancy.

176
Q

Which of the following will be elevated in the setting of hyperparathyroidism?

A. Calcium
B. Phosphorous
C. Sodium
D. Potassium

A

A. Calcium

In the setting of hyperparathyroidism, the calcium level will be elevated. Calcium and phosphorus work in an indirect relationship. We would additionally find a decreased phosphorus level. The sodium and potassium levels are not typically affected with hyperparathyroidism.

177
Q

The nurse practitioner is providing education to a type one diabetic who is inquiring about semaglutide (Ozempic). She is wondering if this is a medication she can start as her friend with diabetes is on it and it is helping her lose weight and lowered her A1C. Which of the following is an appropriate response by the nurse practitioner?

A. “We can definitely consider starting semaglutide (Ozempic). This will work great for you!”

B. “We may consider starting it in a few months, however, I would like you to work on diet and lifestyle changes for the next three months. If your A1C continues to increase we will proceed with starting the medication.”

C. “Unfortunately, this is not a safe medication for type 1 diabetics. This medication increases your risk for diabetes ketoacidosis, therefore, I cannot prescribe it.”

D. “Unfortunately, this is not a safe medication for type 1 diabetics and increases your risk for hypoglycemia, therefore, I cannot prescribe it”

A

C. “Unfortunately, this is not a safe medication for type 1 diabetics. This medication increases your risk for diabetes ketoacidosis, therefore, I cannot prescribe it.”

Semaglutide (Ozempic) is contraindicated in those with type 1 diabetes mellitus due to the risk of diabetes ketoacidosis (DKA) and therefore, should not be prescribed.

178
Q

Avoiding which of the following before 12 months of age can help prevent iron deficiency anemia?

A. Honey
B. Cow’s milk
C. Peanut butter
D. Eggs

A

B. Cow’s milk

Avoiding cow’s milk and cow’s milk-containing products before 12 months of age will help prevent iron deficiency anemia. Cow’s milk is low in iron, and inhibits iron absorption from other sources. Additionally, cow’s milk can cause small bleeds in the gastrointestinal tract, also contributing to iron deficiency anemia, meaning option B is correct. Honey should be avoided before the age of 12 months due to the risk of botulism. Current recommendations support the early introduction, from the age of 6 months onward, of both peanut butter and eggs to help prevent allergy and atopic disease in children.

179
Q

A 7-year-old male patient has a history of sickle cell anemia. Which of the following teaching points should be emphasized to the patient’s caregiver?

A. Patients with sickle cell anemia should eat leafy green vegetables in moderation, as this can impact INR levels

B. Hemoglobin electrophoresis should be performed annually to track disease progression

C. Patients with sickle cell anemia should receive all vaccinations on time

D. If there is concern about sickle cell crisis, this can often be managed in primary care

A

C. Patients with sickle cell anemia should receive all vaccinations on time

Illness and dehydration are leading causes of sickle cell crises, so patients with sickle cell anemia should ensure they are receiving all vaccines on time to prevent illness, meaning option C is correct. Eating leafy green vegetables in moderation to help keep INR levels stable is an important teaching point for patients on warfarin (Coumadin). Hemoglobin electrophoresis is used initially to confirm diagnosis of sickle cell anemia, and does not need to be repeated annually. Sickle cell crises often require IV fluids and IV pain medications and usually require a referral to the ED for treatment.

180
Q

Which of the following findings is the most specific to iron deficiency anemia?

A. MCV 72 fL
B. Pica
C. Glossitis
D. Hgb 10 g/dL

A

B. Pica

Symptoms of iron deficiency anemia include fatigue, weakness, and pale skin. A finding that is unique to iron deficiency anemia is pica, which is a craving for non-food items like dirt, ice, clay, or paper, meaning option B is the correct answer. An MCV of 72 fL is indicative of a microcytic anemia, but this finding would not be specific to iron deficiency anemia. Glossitis is a beefy-red, swollen, painful tongue, and most commonly found in vitamin B12 deficiency anemia. While glossitis can possibly be found in iron deficiency anemia as well, it is not as specific as a finding as pica. A hemoglobin (Hgb) of 10 g/dL would indicate anemia, but this finding is not unique to iron deficiency anemia.

181
Q

A 8-year-old female patient who recently immigrated to the US presents as a new patient. Her parents have no medical records for her, but want their daughter to be able to start public school, and needs a vaccine record. The nurse practitioner knows which of the following vaccines would be included in this patient’s catch-up schedule?

A. Haemophilus influenzae type B vaccine (Hib)
B. Measles, Mumps, and Rubella (MMR)
C. Human papillomavirus (HPV)
D. Diphtheria, tetanus and pertussis (DTaP)

A

B. Measles, Mumps, and Rubella (MMR)

The catch-up vaccine schedule for a child with unknown vaccine status would include IPV, Tdap, MMR, hepatitis B, and varicella. Depending on age, Hib, HPV, and meningitis may be appropriate. B is the correct answer choice. The CDC recommends Hib vaccination in children younger than 5 only. The HPV vaccine can be given from the age of 9 onwards. If a patient is older than 7, they need the TDaP vaccine, so DTap would be an inappropriate choice for this patient.

182
Q

A 41-year-old female who is 2 years post sleeve gastrectomy surgery presents to the clinic to follow up on lab results that indicate a diagnosis of anemia. Previously, the patient made an appointment to discuss a new sensation of burning and tingling in her hands and feet combined with concerns of walking “clumsier than usual.” Given her surgical history and symptoms, which lab finding is most consistent with the anticipated diagnosis?

A. Hct 41%
B. MCV 113 fL
C. Hgb 14 g/dL
D. MCV 73 fL

A

B. MCV 113 fL

The patient’s symptoms and history are consistent with vitamin B12 deficiency anemia, a macrocytic anemia. Common findings in B12 deficiency anemia include neurological symptoms, such as paraesthesias, gait instability, or memory trouble. Furthermore, a major risk factor for vitamin B12 deficiency anemia would be sleeve gastrectomy surgery, which greatly reduces the amount of intrinsic factor in the stomach, which is needed to absorb vitamin B12. Macrocytic anemias have a mean corpuscular volume of >100 fL, meaning B is the correct answer. An MCV less than 80 fL would be indicative of microcytic anemia, such as iron deficiency anemia. While gastric surgeries, such as a sleeve gastrectomy, can also certainly lead to iron deficiency anemia, the presenting neurologic symptoms of paresthesias coupled with gait and balance issues would lead us to a probable diagnosis of Vitamin B 12 deficiency anemia. A hematocrit of 41% and a hemoglobin of 14 g/dL would be normal findings in this patient.

183
Q

A 3-year-old presents to the clinic with his parents with concerns of a 3 day history of worsening sore throat and fever. Upon examination, the nurse practitioner notes stridor, drooling, and a muffled-sounding cry. The nurse practitioner knows which of the following interventions is important in preventing the likely diagnosis?

A. Measles, mumps, and rubella (MMR) vaccine
B. Prophylactic azithromycin (Zithromax)
C. Haemophilus influenzae type B vaccine (Hib) vaccine
D. Albuterol inhaler as needed

A

C. Haemophilus influenzae type B vaccine (Hib) vaccine

The patient is exhibiting classic presentation of epiglottis, which commonly presents with drooling, respiratory distress, and muffled “hot potato” voice. The best method of preventing epiglottitis is the Haemophilus Influenzae Type b (Hib) vaccine, so answer C would be correct. The MMR vaccine is effective in preventing measles, mumps, and rubella. Prophylactic azithromycin is given to those exposed to known cases of pertussis, or whooping cough. An albuterol inhaler PRN would not be appropriate or effective in the prevention of epiglottis.

184
Q

Interpret the following lab results: HBsAg+, IgM-, IgG+, anti-HBc +, anti-HBs -

A. Acute hepatitis B infection
B. Chronic hepatitis B infection
C. Immunity due to hepatitis B vaccination
D. Immunity due to naturally occurring hepatitis B infection

A

B. Chronic hepatitis B infection

A positive hepatitis B surface antigen (HBsAg) indicates either an acute or chronic hepatitis B infection. The total hepatitis B core antibody (anti-Hbc) will be positive from the onset of infection and for life. IgM would be positive in an acute infection. A positive IgG combined with a positive HBsAg would be suggestive of a chronic hepatitis B infection, meaning B is the correct answer. Hepatitis B surface antibody (Anti-HBs) would be positive in a patient vaccinated against hepatitis B. If a patient was immune due to natural hepatitis B infection, we would expect to see both anti-HBc and anti-HBs positive, with a negative HBsAg.

185
Q

A new 45-year-old female patient is describing a hereditary condition in her family to the nurse practitioner. She cannot recall the name, but remembers that the condition involves the body absorbing too much iron. The nurse practitioner recognizes the described condition and knows the typical approach to management of this condition involves which of the following?

A. IV fluids and pain medications during period of crisis
B. Regular phlebotomy
C. Dialysis
D. Direct-acting antiviral (DAA) tablets

A

B. Regular phlebotomy

The description given by the patient is consistent with the diagnosis of hemochromatosis, a genetic condition in which the body stores too much iron in the organs. The cornerstone of treatment for hemochromatosis is regular phlebotomy, meaning B is the correct answer choice. Intravenous fluids and pain medications during times of crisis is an appropriate management strategy for sickle cell anemia. Direct-acting antiviral tablets are used to treat hepatitis C. Kidney dialysis would not be an appropriate treatment option for hemochromatosis.

186
Q

A 56-year-old male patient with a history of gout, hypertension, and alcoholism presents to the clinic with concerns of a swollen tongue for the past week. He is currently on allopurinol (Aloprim) and chlorthalidone (Thalitone) daily, and endorses compliance to his medication regimen. On exam, the nurse practitioner notes a swollen, smooth, dark red tongue. No acute respiratory distress is noted. Which of the following statements would be most appropriate for the nurse practitioner to say to the patient next?

A. More than likely, this is a mild side effect related to your blood pressure medication and requires no treatment

B. I would like to run some basic lab tests on you, including a complete blood count.

C. You are experiencing angioedema, so we will need to change your blood pressure medication

D. Your folate levels are likely low, so I will prescribe a supplement today

A

B. I would like to run some basic lab tests on you, including a complete blood count.

Alcoholism is a common contributing factor to vitamin B12 deficiency anemia. Vitamin B12 deficiency anemia can present with neurological symptoms such as paresthesias, gait disturbances, or memory issues. Another sign of vitamin B12 deficiency anemia is glossitis, or a beefy red, swollen tongue, which is consistent with the patient’s presentation. To begin a workup of suspected vitamin B12 deficiency anemia, the nurse practitioner should order a complete blood count, so answer B is the correct choice. A swollen, beefy red tongue is not a common side effect of chlorthalidone (Thalitone). The exam findings are not consistent with angioedema, a life threatening complication sometimes seen with ACE inhibitors. Furthermore, angioedema is not an expected side effect of chlorthalidone (Thalitone). While alcoholism does contribute to both folate and B12 deficiency, the patient’s current complaint of glossitis is more than likely related to B12 deficiency.

187
Q

A new 45-year-old female patient brings in her recent lab results from a previous provider. Her lab results indicate macrocytic anemia. She tells the nurse practitioner that she sees another provider for her “arthritis medicine,” but cannot remember the name of the specialist. She denies any neurological symptoms, but does endorse fatigue and “feeling tired.” What is the best next question for the nurse practitioner to ask this patient?

A. What medications do you currently take?
B. Did you have any gastric surgeries in the past?
C. Do you live in a home built before 1978?
D. Do you take a daily iron supplement?

A

A. What medications do you currently take?

Folate deficiency anemia is macrocytic and caused by poor dietary intake, alcoholism, and certain medications. Methotrexate, a common medication used in the treatment of rheumatoid arthritis, is notorious for causing folic acid deficiency. The patient’s symptoms plus statements about seeing a specialist to manage her arthritis, would lead to suspicion of an outside diagnosis of rheumatoid arthritis, which is managed by rheumatology. Accordingly, option A would be correct. Gastric surgeries place patients at higher risk for vitamin B12 deficiency anemia due to lack of gastric intrinsic factor. Vitamin B12 deficiency anemia commonly presents with neurologic symptoms, which this patient denies. Homes built before 1978 are more likely to have lead paint. Lead poisoning causes microcytic anemia and is more common in young children. Iron deficiency anemia is also microcytic, so the question would not give us any more information on the patient’s current condition.

188
Q

A 39-year-old female presents to the clinic today with complaints of upper abdominal pain on the right side. She states she has been having episodes of mild abdominal pain off and on lately, but this is much more intense. The nurse practitioner performs the Murphy’s sign which is positive, but the following ultrasound is negative for any gallstones. What should the nurse practitioner do next?

A. Order a computerized tomography (CT) scan for a possible appendicitis

B. Discharge the patient and have her follow up if the pain gets worse

C. Order a hepatobiliary iminodiacetic acid (HIDA) scan to check gallbladder function

D. Prescribe metronidazole (Flagyl) for 1 week with close follow-up

A

C. Order a hepatobiliary iminodiacetic acid (HIDA) scan to check gallbladder function

This patient has symptoms that are highly indicative of cholecystitis such as right upper quadrant abdominal pain and a positive Murphy’s sign. Being middle-aged and female are both risk factors for cholecystitis. If the patient has a negative ultrasound, the next step would be to have a HIDA scan performed to check for gallbladder function. HIDA scans can only be performed if no stones are present, so an ultrasound should be performed first.

189
Q

A 62-year-old male patient is coming in for a post-op appointment following a thyroidectomy. The patient states he has had some muscle spasms and mild numbness since the procedure. Upon tapping on the patient’s face the nurse practitioner notices that the muscles of the face start to contract on the side that was tapped. What is the sign that is being performed?

A. Trousseau’s sign
B. Chvostek’s sign
C. Tinel’s sign
D. Rovsing sign

A

B. Chvostek’s sign

This patient is exhibiting symptoms of hypocalcemia. Chvostek’s and Trousseau’s signs are used to assess for hypocalcemia. Chvostek’s sign is performed by tapping on the patient’s face. Trousseau’s sign is performed by inflating a blood pressure cuff around the arm and watching to see if the arm is drawn up in an involuntary contraction. Tinel’s sign is used to assess for signs of Carpal tunnel syndrome which would not be the case here. Rovsing sign is an abdominal sign used to assess for appendicitis.

190
Q

A 25-year-old female presents to the clinic with severe lower abdominal pain that started about two hours ago. As part of an abdominal assessment, the nurse practitioner asks the patient to raise her right leg while resistance is being placed on the leg. What sign or test is being performed?

A. Markle test
B. Psoas sign
C. Obturator sign
D. Murphy’s sign

A

B. Psoas sign

Many signs can be used to rule out appendicitis. The sign being performed in this scenario is the psoas sign- lifting the leg against resistance. The Markle test is performed by having the patient jump on one foot or drop down on their heels. The obturator sign is done by internally rotating the hip at 90 degrees and the Murphy’s sign is performed by palpating in the subcostal space in the right upper quadrant while the patient takes a deep breath.

191
Q

Which abdominal sign would be concerning for intra-abdominal bleeding?

A. Blumberg sign
B. Rovsing sign
C. Grey-Turner sign
D. Murphy sign

A

C. Grey-Turner sign

The two signs that suggest intra-abdominal bleeding are the Cullen sign, which is a blue discoloration around the umbilicus, and the Grey-Turner sign, which appears as blue discoloration around the flank. Blumberg sign is another name for rebound tenderness. Rovsing sign is performed by palpation on the left lower quadrant to see if it elicits pain on the right side. Murphy sign is performed with palpation of the subcostal space in the right upper quadrant.

192
Q

A 70-year-old male presents to the clinic with abdominal and chest pain. His vitals appear to be stable except for a slightly low blood pressure. The nurse practitioner performs an abdominal assessment and feels a small pulsating mass near the umbilicus. What should the nurse practitioner do next?

A. Schedule an ultrasound
B. Order an EKG
C. Order a CT scan
D. Refer the patient to the ED

A

D. Refer the patient to the ED

This patient is exhibiting signs of a possible ruptured abdominal aortic aneurysm (AAA) such as abdominal pain, chest pain and hypotension. If the nurse practitioner suspects an AAA or rupture, the patient should immediately be sent to the emergency room for further evaluation as it is life threatening. An ultrasound could help diagnose an abdominal aortic aneurysm, but if a rupture is suspected in an outpatient setting, the patient should be referred immediately to the emergency room.

193
Q

A 50-year-old female patient with a history of gastric bypass surgery has an appointment today for concerns about reflux that started a month ago. She reports it being mild, but hasn’t improved much with limiting spicy food and sitting up after eating. Which of the following would be the most appropriate treatment option for this patient?

A. Famotidine (Pepcid)
B. Omeprazole (Prilosec)
C. Ondansetron (Zofran)
D. Lifestyle/diet modifications

A

A. Famotidine (Pepcid)

An H2 blocker like famotidine (Pepcid) would be a good choice for this patient as her symptoms are mild. She also has a risk for developing a B12 deficiency due to her surgical history which would make use of a proton pump inhibitor (PPI) like omeprazole (Prilosec) less appropriate due to increased risk of B12 deficiency with use. Ondansetron (Zofran) would be a better option for nausea. Diet modification is an important teaching point, however, she has already implemented some of these with no improvement and has specific diet restrictions due to her gastric bypass.

194
Q

Which of the following medications would not be not used as part of quadruple therapy for H. pylori?

A. Metronidazole (Flagyl)
B. Famotidine (Pepcid)
C. Omeprazole (Prilosec)
D. Clarithromycin (Biaxin)

A

D. Clarithromycin (Biaxin)

The medications that are used in quadruple therapy are tetracycline (Sumycin), metronidazole (Flagyl), bismuth subsalicylate (Pepto-Bismol) and either an H2 blocker or a proton-pump inhibitor. Famotidine (Pepcid) is an H2 blocker and omeprazole (Prilosec) is a proton-pump inhibitor. The only option listed that would not be a part of quadruple therapy would be clarithromycin (Biaxin), which would be included in triple therapy.

195
Q

A 55-year-old male patient presents to the clinic for his annual physical exam. As the nurse practitioner reviews screenings with the patient, he states he has never had a colonoscopy and does not have any family history of colon cancer. What are the current recommendations for a colonoscopy for this patient?

A. A fecal occult blood test every 3 years

B. Begin colonoscopy at age 60

C. A fecal occult blood test and colonography annually

D. A colonoscopy every 10 years

A

D. A colonoscopy every 10 years

The current guidelines for a patient of this age and no history of colon cancer or disease allow for a few options for screening. Current guidelines suggest screening begin at 50-years-old with no family history, earlier if the patient has risk factors or family history of colon cancer. A high sensitivity fecal occult blood test every year would be one option. CT colonography is another option, but would only occur every 5 years. The patient could also opt for a sigmoidoscopy every 5 years or a colonoscopy every 10 years.

196
Q

A 45-year-old woman with a history of GERD comes into the clinic today with concerns of worsening symptoms. She states that she has been taking omeprazole (Prilosec) for the past 2 months and her reflux is not any better. She also states that it is now more difficult for her to swallow. What would be an appropriate course of action for this patient?

A. Increase dose of omeprazole (Prilosec) and reassess in one month

B. Switch her to famotidine (Pepcid)

C. Refer her for an esophagogastroduodenoscopy (EGD)

D. Implement lifestyle modifications to reduce reflux symptoms

A

C. Refer her for an esophagogastroduodenoscopy (EGD)

This patient has been on omeprazole (Prilosec) for 2 months with worsening symptoms. Current recommendations state if a patient has been treated for 8 weeks with no improvement, then they should be referred for further testing such as an EGD to rule out something more serious like Barrett’s esophagus or esophageal cancer. While H2 blockers are also a first line treatment for GERD, PPI’s are generally more effective. Lifestyle modifications are an important teaching point with GERD, but would be done earlier on before starting medications or during treatment and ordering further testing is a higher priority in this patient.

197
Q

Which part of the colon is the most likely location for polyps to occur?

A. Ascending colon
B. Transverse colon
C. Descending colon
D. Sigmoid colon

A

C. Descending colon

Polyps can develop in any part of the colon, but the descending colon is the most often affected by colon polyps/colon cancer. Polyps decrease the diameter of the colon, making it a smaller passageway for stool to get through. That is why ribbon or pencil shaped stools are a sign of colon polyps or cancer.

198
Q

A 19-year-old female presents with a complaint of vaginal discharge, which started 2 weeks ago. She denies fever, chills, dysuria, and flank pain. She believes the discharge started after sexual activity with her female partner. She describes the discharge as thin and white in color. The nurse practitioner examines the vaginal discharge in clinic using a wet mount and observes clue cells with a positive whiff test. What antibiotic will the nurse practitioner use to treat this patient?

A. Doxycycline (Vibramycin)
B. Azithromycin (Zithromax)
C. Nitrofurantoin (Macrobid)
D. Metronidazole (Flagyl)

A

D. Metronidazole (Flagyl)

Clue cells and positive whiff test indicate bacterial vaginosis (BV) which is treated with metronidazole (Flagyl). The patient’s description of the discharge as thin and white is consistent with BV. A risk factor for BV in this patient is recent sexual activity. Any sexual activity, but particularly with two female partners, is a risk for BV.

199
Q

The nurse practitioner has finished evaluating a 22-year-old male with unilateral scrotal swelling, erythema, tenderness, and a positive Prehn’s sign. The patient now shares he had unprotected sexual activity 4 weeks ago. He had gonorrhea/chlamydia testing done at another medical facility yesterday, but has not received the results yet. What treatment is most appropriate for this patient?

A. Levofloxacin (Levaquin)
B. Ceftriaxone (Rocephin) and doxycycline (Vibramycin)
C. Trimethoprim-sulfamethoxazole (Bactrim)
D. Amoxicillin (Amoxil)

A

B. Ceftriaxone (Rocephin) and doxycycline (Vibramycin)

For epididymitis, the recommendation in men under age 35 and in particular if there is a high risk of STI is to treat with ceftriaxone (Rocephin) IM and doxycycline for 10 days. As a reminder, when there is risk for STI and there are no gonorrhea or chlamydia testing results available, the guideline is to treat for both.

200
Q

Which of the following patients needs treatment for a urinary tract infection (UTI)?

A. 40-year-old male with no symptoms and mucous threads on urinalysis (UA)

B. 32-year-old pregnant female with no symptoms and leukocytes on UA

C. 50-year-old male with no symptoms and proteinuria on UA

D. 17-year-old female with no symptoms and leukocytes on UA

A

B. 32-year-old pregnant female with no symptoms and leukocytes on UA

All pregnant patients with asymptomatic bacteriuria (leukocytes or white blood cells in urine) should be treated. If the patient is not pregnant and has asymptomatic bacteriuria treatment is not recommended. Mucous threads are common findings on a UA and are not usually pathologic. Proteinuria alone (without other positive UA findings) can be found in many disease states, most commonly renal disorders such as chronic kidney disease (CKD).

201
Q

A 40-year-old male presents for evaluation of rash. It is not pruritic or painful. He has not started any new medications and denies using any new soaps or detergents. He denies any recent travel. His medications include lisinopril and atorvastatin. On exam, the nurse practitioner observes a macular rash on the palms of his hands and soles of his feet. His lab results include a normal CBC, Creatinine 0.7 mg/DL, normal ESR, FTA-ABS positive, RPR 1:64. Which of the following is the most likely diagnosis?

A. Atopic dermatitis
B. Secondary syphilis
C. Rocky Mountain spotted fever
D. Psoriasis

A

B. Secondary syphilis

Secondary syphilis can commonly present as a macular or papular rash on the trunk or palms of hands or soles of feet. (Primary syphilis would be a single ulceration.) The diagnosis of syphilis is confirmed with the treponemal (FTA-ABS) positive test and the nontreponemal RPR positive. If the FTA and RPR were negative, then you would consider Rocky Mountain spotted fever as that also presents on the palms of hands and feet. Atopic dermatitis is characterized by a pruritic, erythematous, often scaly papules. Psoriasis is more consistent with erythematous plaques.

202
Q

A patient who presents for sexually transmitted infection (STI) screening asks the nurse practitioner if test results will be shared with anyone. What is the most appropriate response?

A. Positive STI testing for HIV, syphilis, gonorrhea, and chlamydia are mandatory to report to the local department of health.

B. Any positive STI testing will be reported directly to their sexual partners by the health care provider.

C. Positive STI testing for trichomonas, bacterial vaginosis, HIV, and syphilis are mandatory to report to the local department of health.

D. No positive results will be shared with any agency.

A

A. Positive STI testing for HIV, syphilis, gonorrhea, and chlamydia are mandatory to report to the local department of health.

HIV, syphilis, gonorrhea, and chlamydia are reportable in every state. Trichomonas and bacterial vaginosis are not reportable. Due to HIPAA the health care provider cannot contact the sexual partners for notification. Health departments have partner notification programs.

203
Q

A 30-year-old female has painful vulvar vesicles and the nurse practitioner diagnoses her with herpes simplex virus (HSV). The symptoms initially started 48 hours ago. How will the nurse practitioner treat this patient?

A. She is out of the treatment window and cannot be treated

B. Administer penicillin benzathine (Bicllin) intramuscularly

C. Start acyclovir (Zovirax)

D. Start topical mupirocin (Bactroban).

A

C. Start acyclovir (Zovirax)

The first line treatment for HSV is acyclovir (Zovirax) that has to be started within a 48 to 72 hour window, and this patient is still in the treatment window. Penicillin benzathine (Bicillin) is not correct because it is the treatment for syphilis. Topical mupirocin (Bactroban) may be prescribed for mild folliculitis.

204
Q

What is the best test to diagnose and stage chronic kidney disease (CKD)?

A. Blood urea nitrogen (BUN)
B. Creatinine
C. Glomerular filtration rate (GFR)
D. Renal ultrasound

A

C. Glomerular filtration rate (GFR)

The GFR is the best indicator of kidney function, and it is used to stage CKD. Creatinine and BUN are other indicators of kidney function, but are less specific than the GFR. We may consider a renal ultrasound in the workup for CKD, but the GFR is the best indicator.

205
Q

A 50-year-old male with a past medical history of hypertension, hyperlipidemia, coronary artery disease, and osteoarthritis is being evaluated for chronic kidney disease (CKD). He has the following lab results: Hgb 10.1 (g/dL), MCV 92 fL, A1C 6.3%, Creatinine 1.9 mg/dL, GFR 45 mL/min, BUN 27mg/dL, K 4.0 mmol/L ALT 40 U/L, AST 30 U/L. What value on this patient’s urinalysis would indicate underlying glomerulonephritis causing this patient’s CKD?

A. Positive nitrites
B. Squamous epithelial cells
C. Specific gravity 1.005
D. Red blood cell casts

A

D. Red blood cell casts

In glomerulonephritis, microscopic bleeding occurs in the kidneys which leads to the red blood cells clumping together which is indicated by the red blood cell casts. This patient’s low glomerular filtration rate (GFR) is indicative of CKD, along with the elevated creatinine and BUN. Glomerulonephritis is a common cause of CKD. Squamous epithelial cells are a common benign finding on urinalysis results. The urine specific gravity listed is within normal range, and thus would not indicate an underlying problem. Positive nitrites would be more indicative of a urinary tract infection (UTI).

206
Q

What test is used to monitor for renal complications secondary to diabetes mellitus?

A. Urinalysis
B. Renal ultrasound
C. Hemoglobin A1C
D. Urine microalbumin/creatinine ratio

A

D. Urine microalbumin/creatinine ratio

Elevated microalbumin in the urine sample of a diabetic patient can indicate microvascular damage in the kidneys. A diabetic patient with an uncontrolled or elevated A1C will be more likely to have microalbuminuria and renal complications, but the A1C itself does not screen for this. A urinalysis and renal ultrasound are often part of the workup for chronic kidney disease (CKD), but the urine microalbumin is more specific to the microvascular complications from diabetes.

207
Q

Which of the following medications needs dose adjusted based on the patient’s glomerular filtration rate (GFR)?

A. Insulin glargine (Lantus)
B. Metformin (Gluocophage)
C. Atorvastatin (Lipitor)
D. Amlodipine (Norvasc)

A

B. Metformin (Gluocophage)

Metformin can only be used if the GFR is 45 mL/min or above. If a patient is on metformin and the GFR worsens to between 30-45 mL/min, we will need to decrease the dose by half. If the GFR drops below 30 mL/min, we cannot use metformin. Dosing and use of the other medications listed are not based on GFR.

208
Q

Which of the following contraceptive methods significantly increases the risk of osteoporosis?

A. Copper IUD (Paraguard)

B. Norelgestromin /ethinyl estradiol transdermal (Ortho Evra Patch)

C. Medroxyprogesterone acetate injection (Depo-Provera)

D. Etonogestrel/ethinyl estradiol vaginal ring (NuvaRing)

A

C. Medroxyprogesterone acetate injection (Depo-Provera)

The medroxyprogesterone acetate injection (Depo-Provera) decreases serum estrogen levels, leading to bone density loss. The copper IUD (Paraguard) is a nonhormonal option which has no impact on bone density. Neither etonogestrel/ethinyl estradiol vaginal ring (NuvaRing) or norelgestromin /ethinyl estradiol transdermal (Ortho Evra Patch) have a major known association with osteoporosis.

209
Q

An 11-year-old female patient and her mother present for a well-check. The mother wants to know more information about the Gardasil vaccine. The nurse practitioner correctly states that:

A. The patient will be eligible to receive Gardasil at the age of 12

B. HPV subtypes 16 and 18 account for about 70% of all cervical cancers.

C. Gardasil protects against high-risk HPV subtypes 6 and 11, which cause most cervical cancers.

D. The patient is too old to receive the Gardasil vaccine

A

B. HPV subtypes 16 and 18 account for about 70% of all cervical cancers.

The Gardasil HPV vaccine protects against 9 strains of HPV known to cause HPV-linked cervical, oropharyngeal, vulvar, vaginal, and anal cancers. High-risk HPV subtypes 16 and 18 cause around 70% of all cervical cancers; option B is the correct choice. HPV subtypes 6 and 11 are low-risk and cause over 90% of genital warts. The Gardasil vaccine can be administered in females between the ages of 9 and 45 years of age, so this patient is a candidate.

210
Q

Which of the following instances would not require parental consent in a patient under 18 years of age?

A. Treatment for dysmenorrhea

B. Consultation for Accutane to treat cystic acne

C. Testing and treatment of chlamydial infection

D. Administration of Gardasil vaccine

A

C. Testing and treatment of chlamydial infection

While the specifics of these scenarios may vary between states, parental consent is not required for STI screening or treatment, pregnancy-related care, and contraception. In general, vaccinations, treatment of dysmenorrhea, and any medical consultation for medications would require parental consent in patients under the age of 18.

211
Q

A 32-year-old female patient presents with complaints of unusual thin milky discharge from her bilateral breasts for the past month. She denies any new medication, and uses etonogestrel/ethinyl estradiol vaginal ring (NuvaRing) for birth control, and takes a multivitamin daily. Her urine pregnancy test is negative. The nurse practitioner knows that ordering which of the following lab tests is the priority action given the patient’s history and complaint?

A. Complete blood count (CBC)

B. Thyroid stimulating hormone (TSH)

C. Prolactin

D. Serum T3/T4

A

C. Prolactin

This condition can be due to breast stimulation, prolactinomas, disorders of the pituitary gland, or medications such as atypical antipsychotics. While often benign in nature, the best first diagnostic test would be a prolactin level to rule out prolactinoma as the cause. TSH, serum T3/T4, and CBC may also provide useful diagnostic information, but prolactin is the priority.

212
Q

A 56-year-old patient has not had a menstrual period in 3 years. She says she has been having intermittent moderate vaginal bleeding 2-3 times per week. Which of the following is the most appropriate next step for the nurse practitioner to take?

A. Order serum estradiol, follicle-stimulating hormone, and luteinizing hormone

B. Refer to OB/GYN

C. Provide reassurance that within the first five years of menopause, some unusual bleeding is expect

D. Ask the patient to keep a record of frequency and heaviness of bleeding, follow-up in 1 month

A

B. Refer to OB/GYN

Twelve months with no menstrual cycle is diagnostic of menopause. Any unusual vaginal bleeding after that 12 month mark is concerning for endometrial cancer and requires a prompt referral to OB/GYN for an endometrial biopsy. Serum estradiol, follicle stimulating hormone, and luteinizing hormone levels are helpful in diagnosing menopause, but would not be an appropriate next step in this patient with abnormal vaginal bleeding. Asking the patient to keep a record of bleeding and then following up would not be a priority intervention.

213
Q

A 31-year-old female is experiencing irregular periods, weight gain, acne, and unusual hair growth on her upper lip and cheeks. She reports struggling with these symptoms for most of her adult life. The nurse practitioner notes tender lumps and knots under the skin of the jaw and cheeks, as well as fine, dark hair distributed on her upper lip and jawline upon examination. Which of the following is the most likely explanation for these findings?

A. History of combined oral contraceptive usage
B. Polycystic ovarian syndrome
C. Endometriosis
D. Uterine fibroids

A

B. Polycystic ovarian syndrome

214
Q

A 36-year old female patient presents for a well-woman exam and wants to discuss birth control options. She has a history of hypothyroidism and currently smokes ½ pack of cigarettes per day. She tells you that three years ago she had a pregnancy that required surgery because the “baby was in her falllopian tube.” The nurse practitioner recognizes that which of the following would be an appropriate birth control option for this patient?

A. Norelgestromin /ethinyl estradiol transdermal (Ortho Evra Patch)

B. Levonorgestrel-releasing IUD (Mirena)

C. Copper IUD (Paraguard)

D. Norethindrone (the mini-pill)

A

D. Norethindrone (the mini-pill)

Absolute contraindications for combined estrogen/progestin contraceptives include smokers over the age of 35, history of migraines with aura, history of deep vein thrombosis, uncontrolled hypertension, breast cancer, liver tumor or liver disease, pregnancy, or undiagnosed abnormal vaginal bleeding. Furthermore, the Ortho Evra patch carries the highest risk of blood clot of the combined hormonal contraceptive options. Due to this patient’s history of ectopic pregnancy, any IUD use would be contraindicated. The best option for this patient would be a progestin-only or nonhormonal option, making option D the correct answer.

215
Q

A 54-year-old perimenopausal female patient complains of daily hot flashes and night sweats. She wants to discuss options for manage her menopausal symptoms. She has spoken to her sister, who is a nurse, about management options, and tells the nurse practitioner that she is not interested in any medication containing any ingredient that would increase her risk of any gynecological cancers. Which of the following options would be inappropriate for this patient given her request?

A. Pareoxetine (Paxil)
B. Black cohosh
C. Conjugated estrogen tablets (Premarin)
D. Evening primrose

A

C. Conjugated estrogen tablets (Premarin)

Hormon replacement therapy (HRT) in menopausal women comes with many risks, such as increased risk of blood clots, cardiovascular disease, and diabetes. Both estrogen-only and combined HRT options carry an increased risk of ovarian cancer. To honor this patient’s request of avoiding medications containing hormones, Premarin, an estrogen-only HRT option, would be an inappropriate choice for her. C is the correct answer. Paroxetine (Paxil) is a selective serotonin reuptake inhibitor known to help with menopausal symptoms such as hot flashes and night sweats. Both black cohosh and evening primrose are herbal supplements used to improve menopausal symptoms.

216
Q

A 27-year-old female presents for follow-up of abnormal pap smear results. She is sexually active with her partner of 3 years. She states their relationship is monogamous. Her results indicated abnormal cells but were not diagnostic of a lesion. Which of the following is the most effective management strategy?

A. Repeat pap smear in 6 months

B. Refer for immediate colposcopy

C. Ensure reflex HPV testing is underway

D. Provide reassurance that this finding can be normal, no more action need at this time

A

C. Ensure reflex HPV testing is underway

Atypical squamous cells of undetermined significance (ASCUS) are a category of abnormal Pap smear result. ASCUS indicates that abnormal cells were present, but did not meet the diagnostic criteria for a lesion. While we would provide reassurance that ASCUS cells can be from a variety of causes and may not be a cause for concern, the priority management strategy would be to ensure that reflexive HPV testing is underway in all patients with ASCUS results, regardless of age. Depending on the results of the reflexive HPV testing, the provider may then choose to repeat the pap smear in 6 months, or refer for a colposcopy.

217
Q

A 27-year-old first time mother presents for her 6 week postpartum check-up. She wants to discuss birth control options that are breastfeeding friendly. The nurse practitioner prescribes an oral contraceptive. Which of the following teaching points would be critical with the most likely prescribed medication?

A. This pill needs to be taken at the exact same time every day in order to be effective.

B. This medication contains two different hormones which will prevent pregnancy

C. If you have any history of migraines with aura, it is important that you tell your provider

D. As long as you take this medication within the same 8-hour window each day, it will be highly effective.

A

A. This pill needs to be taken at the exact same time every day in order to be effective.

The best oral contraceptive option for breastfeeding mothers is a progestin-only containing option, such as norethindrone (the mini-pilll). Progestin-only oral contraceptives need to be taken at approximately the same time every day in order to be most effective; A is the correct answer choice. Combined oral contraceptives contain both progestin and estrogen, and carry with them more contraindications and risk of blood clot and stroke. For combined oral contraceptive options, history of migraine with aura would be an absolute contraindication.

218
Q

A 23-year-old female patient presents to the clinic with complaints of nausea, vomiting, and tender breasts. She also reports having a positive urine pregnancy test. Which of the following categories would the positive urine pregnancy test fall under?

A. Positive
B. Presumptive
C. Probable
D. Confirmed

A

C. Probable

The probable signs of pregnancy are Hegar’s, Chadwick’s, and Goodell’s sign, as well as an HCG urine pregnancy test. A presumptive sign of pregnancy is a subjective report that the patient tells the nurse practitioner such as amenorrhea or breast tenderness. Positive signs are objectively what the nurse practitioner can feel or see such as fetal heart tones, palpation of the fetus, and ultrasounds.

219
Q

A 20-week pregnant patient presents to the clinic for a prenatal visit. Upon measurement, her abdomen is measuring 22 centimeters. What will the nurse practitioner tell the patient about this measurement?

A. This is an abnormal reading, you will need to be seen by the emergency department immediately.

B. This is a normal reading. A reading of +/- 4 centimeters is an acceptable range

C. This is an abnormal measurement. An ultrasound will need to be performed right away.

D. This is a normal reading. A measurement of +/- 2 centimeters is acceptable

A

D. This is a normal reading. A measurement of +/- 2 centimeters is acceptable

220
Q

Which of the following vaccines should a pregnant patient not receive?

A. Influenza
B. Measles, Mumps, and Rubella (MMR)
C. Tetanus, diphtheria, and pertussis (TDaP)
D. Hepatitis B

A

B. Measles, Mumps, and Rubella (MMR)

MMR (Measles, Mumps, and Rubella) is a live vaccine and live vaccines cannot be administered during pregnancy. The other options are all safe to administer during pregnancy, as long as the influenza vaccine used is not the live nasal vaccine.

221
Q

A 35-week pregnant patient presents to the clinic for her weekly prenatal appointment. What test should the nurse practitioner inform the patient about that should be obtained at today’s visit?

A. Oral glucose tolerance testing
B. Rapid plasma reagin (RPR) testing
C. Group B streptococcus test
D. HIV1/HIV2 antibody assay test

A

C. Group B streptococcus test

222
Q

An 18-week pregnant patient presents to the clinic for her Alpha Fetoprotein (AFP) test level results. Her results are low. What should the nurse practitioner inform the patient about these results?

A. This is an abnormal lab result that can be indicative of spina bifida, further testing will need to be obtained.

B. This is a normal lab result, no further testing is indicated at this time.

C. This is an abnormal lab result that can be indicative of Down syndrome, further testing is warranted

D. This is an abnormal lab result and can be indicative of a multiple gestation pregnancy.

A

C. This is an abnormal lab result that can be indicative of Down syndrome, further testing is warranted

223
Q

A 38-week pregnant patient presents to the clinic with complaints of painful contractions and vaginal bleeding. Upon assessment, the uterus feels rigid. What should the nurse practitioner do next?

A. Inform the patient that this is bloody show and is a normal part of labor beginning and to call the on call OBGYN if contractions worsen

B. Obtain an ultrasound in office

C. Send the patient to the emergency room

D. Admit to labor and delivery for fetal monitoring

A

C. Send the patient to the emergency room

A patient who is late in their third trimester who presents with painful contractions and bleeding can be experiencing placenta abruption and this is a life-threatening medical emergency.

224
Q

A 2-week postpartum patient presents to the clinic with complaints of breast pain. On examination, the nurse practitioner notes the breast is erythematous. While palpating the breast it is noted it feels it is boggy and warm. The nurse practitioner diagnoses this patient with mastitis. What medication should the nurse practitioner consider safest while breastfeeding?

A. Doxycycline (Vibramycin)

B. Dicloxicillin (Diclocil)

C. Sulfamethoxazole/trimethoprim (Bactrim)

D. Ciprofloxacin (Cipro)

A

B. Dicloxicillin (Diclocil)

Dicloxcillin is safe and effective in treating mastitis. It is safe to use while continuing to breastfeed as well. All other options are contraindicated in breastfeeding patients.

225
Q

What supplement is recommended for all women of childbearing age to prevent neural tube defects like spina bifida?

A. Ferrous sulfate (Iron)
B. Vitamin D
C. Vitamin B12
D. Folic acid

A

D. Folic acid

226
Q

A 24-week pregnant patient presents to the clinic with complaints of dysuria, hesitancy, and frequency. After further testing, the nurse practitioner diagnoses the patient with a urinary tract infection. Which antibiotic is the safest for this patient?

A. Ciprofloxacin (Cipro)

B. Amoxicillin (Amoxil)

C. Doxycycline (Vibramycin)

D. Sulfamethoxazole/trimethoprim (Bactrim)

A

B. Amoxicillin (Amoxil)

Amoxicillin is the safest option for the pregnant patients out of these choices. The other 3 options are category X medications in pregnancy.

227
Q

A 21-year-old female patient presents to the clinic with complaints of cramping and nausea. She reports a positive urine pregnancy test last week. Upon assessment, the nurse practitioner visualizes a blue-discoloration around the umbilicus of the patient. What should the nurse practitioner do next?

A. This is called Grey-Turner’s sign and is indicative of an intra-abdominal bleeding. Transfer the patient to the emergency room.

B. This is called Cullen’s sign and is indicative of an intra-abdominal bleed, refer the patient to the emergency room immediately.

C. This is a normal finding in early pregnancy due to uterine growth. The patient should be seen by OBGYN in the next week.

D. This is a normal finding in early pregnancy called the Hegar’s sign. The patient should be referred for an ultrasound.

A

B. This is called Cullen’s sign and is indicative of an intra-abdominal bleed, refer the patient to the emergency room immediately.

228
Q

A nurse practitioner has ordered terazosin (Hytrin) to a patient with benign prostatic hyperplasia. What is the most important thing the nurse practitioner should teach the patient?

A. Take this medication at bedtime

B. This can cause an erection lasting longer than 2 hours

C. Take this medication with breakfast

D. The patient’s urine may turn orange

A

A. Take this medication at bedtime

Drugs used for benign prostatic hyperplasia (BPH) are treated with alpha blocker medications like terazosin (Hytrin) and doxazosin (Cardura). These medications need to be taken at night due to the risk of orthostatic hypotension and the increased fall risk. Other symptoms associated with alpha blockers include impotence, drowsiness, dizziness, nausea, blurred vision, and a stuffy nose.

229
Q

The prostate-specific antigen (PSA) test has resulted for a patient. The nurse practitioner knows that a referral is warranted at a level of:

A. 2.1 ng/mL
B. 5.4 ng/mL
C. 0.4 ng/mL
D. 1.2 ng/mL

A

B. 5.4 ng/mL

For prostate-specific antigen (PSA) testing, a value of 4.0 ng/mL or higher requires a referral to urology. A PSA level of 4.0 ng/mL does not necessarily mean the patient has cancer, but that will be determined by the follow-up testing done by the urology specialist.

230
Q

An 18-year-old patient presents with erythema and edema of the scrotum. The patient first noted swelling of the left testicle and pain 3 days ago. This is accompanied by dysuria and urethral discharge. During the sexual history, the patient reports 4 partners within the last 2 months. They do not use condoms. What exam finding, if positive, can help the nurse practitioner diagnose this patient’s condition?

A. Murphy’s sign
B. Cremasteric sign
C. Angell’s sign
D. Prehn’s sign

A

D. Prehn’s sign

Scrotal edema, erythema, dysuria, and urethral discharge suggest epididymitis. This diagnosis can be confirmed by using the Prehn’s sign during the physical examination. The Prehn’s sign will be positive if there is relief of the patient’s scrotal pain with elevation of the scrotum. Epididymitis is most commonly caused by chlamydia or gonorrhea. A positive Prehn’s sign would warrant further sexually transmitted infection (STI) testing.

231
Q

The nurse practitioner is evaluating a patient with severe testicular pain that started 3 hours ago. The patient’s scrotum is edematous and the right testicle appears higher than the left testicle. Pain is accompanied by nausea, vomiting, and fever. What sign would help the nurse practitioner determine the patient’s condition?

A. Prehn’s sign
B. Cremasteric reflex
C. Abdominal ultrasound
D. Computerized tomography (CT)

A

B. Cremasteric reflex

During the physical exam, if the nurse practitioner is unable to elicit the cremasteric reflex by stroking the inner thigh and noting a rising of the ipsilateral testicle, then it is most likely that this patient has a testicular torsion. Prehn’s sign would not be helpful as it is positive with epididymitis, not testicular torsion. An abdominal ultrasound would be inappropriate as it would not evaluate the scrotum.

232
Q

If the nurse practitioner notes a positive Prehn’s sign, accompanying urethral discharge, testicular pain, and an edematous scrotum, what is the best course of treatment?

A. Refer to urology
B. Cefalexin (Keflex)
C. Amoxicillin (Amoxil)
D. Ceftriaxone (Rocephin)

A

D. Ceftriaxone (Rocephin)

Epididymitis is diagnosed from symptoms of: urethral discharge, an edematous scrotum, testicular pain, and + Prehn’s sign. Ceftriaxone (Rocephin) or doxycycline (Vibramycin) are first-line agents for treating this condition in the younger adult population as it is usually caused by gonorrhea or chlamydia.

233
Q

A patient is being seen for a routine yearly exam. The patient reports that they have been experiencing difficulties maintaining an erection and would like to get a prescription for tadalafil (Cialis). There is a history of unstable angina which nitroglycerin is used for. What is the most appropriate next step in management of this patient?

A. Provide the patient with a tadalafil (Cialis) prescription

B. Order an electrocardiogram and stress test

C. Refer to cardiologist

D. Refuse to prescribe tadalafil (Cialis)

A

D. Refuse to prescribe tadalafil (Cialis)

This is because it is a phosphodiesterase type 5 (PDE-5) inhibitor. PDE-5 inhibitors are contraindicated within 24-48 hours of nitroglycerin due to the potential for severe hypotension. This is caused by the widespread vasodilation of both drugs. Remember that PDE-5 inhibitors are given when the penis doesn’t erect (PDE). This class of medications all end with -afil which can be remembered as they allow the blood vessels to be easily filled with blood through vasodilation.

234
Q

Sildenafil (Viagra) is requested by a patient to treat his erectile dysfunction. He has no past cardiac conditions. Prior to initiating a prescription for sildenafil, what tests does the nurse practitioner need to order?

A. Electrocardiogram

B. Cardiac stress test

C. Echocardiogram

D. No testing is necessary at this time as the patient does not have a cardiac history

A

A. Electrocardiogram

When patients present with erectile dysfunction, it is important to rule out a cardiac cause of the erectile dysfunction. LIkewise, before prescribing phosphodiesterase type 5 (PDE-5) inhibitors, it is very important to verify there are no underlying cardiac conditions as PDE-5 inhibitors can exacerbate certain symptoms or interact with cardiac medications. Therefore, performing a baseline electrocardiogram and cardiac lab testing are important for evaluating cardiac health.

235
Q

Galactorrhea is noted in a 56-year-old patient presenting to the clinic. Current medications that this patient is take are enalapril (Innovace), omeprazole (Prilosec), spironolactone (Aldactone), and clozapine (Clozaril). Which of the following medications is most likely to cause galactorrhea?

A. Enalapril (Innovace)
B. Omeprazole (Prilosec)
C. Clozapine (Clozaril)
D. Spironolactone (Aldactone)

A

C. Clozapine (Clozaril)

Atypical antipsychotics, like clozapine, are most likely to cause galactorrhea. Spironolactone is incorrect as it is known for causing gynecomastia due to the anti-androgen effects of the drug, and in very rare cases may cause galactorrhea, but more commonly, this is a side effect of atypical antipsychotics.

236
Q

During a digital rectal exam (DRE), the nurse practitioner notes a symmetrically enlarged, rubbery prostate. This is accompanied by a patient report of increased urination frequency and some difficulties when attempting to urinate. What suspected condition does this patient have?

A. Urinary tract infection (UTI)
B. Prostatitis
C. Prostate cancer
D. Benign prostatic hyperplasia (BPH)

A

D. Benign prostatic hyperplasia (BPH)

Benign prostatic hyperplasia (BPH) will present with a symmetrically enlarged, rubbery prostate during a digital rectal exam (DRE). This enlargement of the prostate also produces urinary frequency and hesitancy symptoms. Prostate cancer will be palpable as a firm, immovable nodule of the prostate during a DRE. Prostatitis feels like an enlarged and boggy prostate on DRE.

237
Q

A patient reports taking saw palmetto for relief of benign prostatic hyperplasia. The patient states that the saw palmetto is no longer providing much relief and they would like to try a pharmaceutical drug. What is the nurse practitioner’s first choice in treatment of this patient?

A. Doxazosin (Cardura)
B. Labetalol (Trandate)
C. Verapamil (Verelan)
D. Finasteride (Proscar)

A

A. Doxazosin (Cardura)

Medications ending in -zosin are alpha blockers that are helpful in the treatment of urinary retention that is seen with benign prostatic hyperplasia (BPH). Finasteride may be used to treat BPH as well but can take up to 6 months to relieve symptoms, so it is not considered first line. Labetalol and verapamil are both medications that are primarily utilized for the treatment of hypertension.

238
Q

A 2-year-old male child presents to the office for a well-child check. He was born at 32 weeks gestation and his mother is asking the nurse practitioner if she should be concerned about delays in development. He has been slightly behind on a few milestones at previous visits. However, he has met all 2 year-old milestones assessed at this visit. The best response for this mother is:

A. He will always have a delay in reaching developmental milestones because he was born 2 months early

B. We expect the child to have developmental delays until he begins pre-school

C. Children born prematurely often catch up with normal developmental milestones by the time they are two years old

D. He can catch up developmentally, but he will need intensive physical and speech therapy sessions

A

C. Children born prematurely often catch up with normal developmental milestones by the time they are two years old

These children will not commonly have permanent developmental delay due to prematurity. Children typically begin preschool between the ages of 3 and 5, making this option inappropriate. Intensive physical and speech therapy is not necessary for all premature infants.

239
Q

The nurse practitioner is evaluating a newborn female at her 1-week newborn visit. The infant was born at 39 weeks gestation with no birth complications. The parent is asking questions about the infant’s delivery and mentions concern about the baby having vaginal bleeding a few hours after delivery. This vaginal bleeding has not persisted and was reportedly “very light”, but the parents are still very concerned. What is the best next step in management for this infant?

A. Order an abdominal ultrasound

B. Referral to a pediatric hematologist for further evaluation

C. Draw a complete blood count

D. No further evaluation is necessary at this time

A

D. No further evaluation is necessary at this time

This is a variation of normal after birth because of the mother’s hormones present in the infant’s circulation. The parent(s) should be reassured at this time. Withdrawal bleeding, seen in this scenario, does not require any additional evaluation. Furthermore, the vaginal bleeding was scant, self-limiting, and has not occurred again.

240
Q

The nurse practitioner is evaluating a 13-year-old male after an accident resulting in a fracture of the leg. On x-ray, this femoral fracture appears to be very close to the knee. He is planning immediate referral to the emergency department because the nurse practitioner is most concerned about what?

A. Salter-Harris fracture
B. Comminuted fracture
C. Transverse fracture
D. Greenstick fracture

A

A. Salter-Harris fracture

This fracture is very close to the growth plate. Fractures of the growth plate are known as Salter-Harris fractures, which warrants an immediate referral for treatment to help prevent growth delay.

241
Q

A 4-year-old child presents to the office with symptoms of red, swollen hands and feet for the last 4 days. Upon evaluation of the child’s mouth, erythematous mucosa is noted throughout the mouth and back of the throat. His tongue is erythematous, swollen, and covered in small white bumps. The parent reports that the child also has a rash that is beginning to cause peeling of the skin. What is the best treatment for this child?

A. Acetylsalicylic acid (aspirin)

B. Acetylsalicylic acid (aspirin) and IVIG (intravenous immune globulin) therapy

C. Alternating acetaminophen (Tylenol) and ibuprofen (Motrin) for the fever and symptom management

D. No treatment is necessary, it is self-limiting

A

B. Acetylsalicylic acid (aspirin) and IVIG (intravenous immune globulin) therapy

Children with Kawasaki’s disease can present with a variety of symptoms including widespread inflammation and swelling, fever, rash, and desquamation of the hands and feet. Acetylsalicylic acid (aspirin) is typically avoided in the pediatric population due to the risk of Reye’s syndrome. However, in children with Kawasaki’s disease, prompt treatment with aspirin (acetylsalicylic acid) is needed as benefits in preventing coronary artery complications far outweigh the risks. The immunoglobulin (or IVIG) is also given to those with Kawasaki’s disease as it also has anti-inflammatory effects and reduces the risk of cardiac complications.

242
Q

A 16-year-old female presents to the clinic with a chief complaint of a bump on her ear that started several months after receiving an ear piercing. The piercing site had been inflamed and reddened with some malodorous drainage a couple weeks after the piercing was received, but the patient denies any pain, itching, or discharge anymore.The bump is flesh-colored. She has tried popping it and using antibacterial creams, with no decrease in size. What is the most likely diagnosis?

A. Metal hypersensitivity
B. Hypertrophic scar
C. Chondritis
D. Keloid

A

D. Keloid

Keloids are scar tissue that grow due to infection or injury of the skin. They may develop as the result of an allergic reaction or infection, but will not be painful or draining. Hypertrophic scars are somewhat similar to keloids. These scars can be distinguished from keloids because hypertrophic scars remain within the boundaries of the site of injury and fade/regress over time. Keloids, however, will grow beyond the borders of that original injury and will not typically regress.

243
Q

While assessing a child in the clinic, the nurse practitioner notes that the child is able to throw a ball overhand and walk easily around the exam room. His parent draws several shapes, as the child attempts to trace them. He is able to copy a circle, but has difficulty tracing a cross and a square. If he is keeping up with his developmental milestones, what age do you suspect he is?

A. 4-years-old
B. 2-years-old
C. 3-years-old
D. 18-months-old

A

C. 3-years-old

Children should be able to ride a tricycle, trace a circle, and throw a ball overhand by the age of 3 (think a tricycle has 3 circular wheels and a ball is also circular to remember this!). However, they are not expected to draw a cross until the age of 4 (think the cross has 4 points to help remember this!).

244
Q

Parents bring in a child who is 9 months old. At the 8 week infant follow-up appointment, the parents reported that their child had periods where their eyes temporarily pointed inwards, towards the nose. Today, the nurse practitioner notes that the left eye appears to remain deviated towards the child’s nose. What is the best next step?

A. Refer to ophthalmologist

B. Reassess the child at the 12-month visit

C. No intervention is necessary

D. Ask the parents if the child reacts to toys and colors

A

A. Refer to ophthalmologist

The condition described here is strabismus. Strabismus can be noted as an inward (esotropic) or outward (exotropic) eye deviation. It should be resolved by the age of 4-6 months. If it does not resolve by this time, then the child should be referred to ophthalmology for further treatment.

245
Q

A newborn infant is brought into the clinic to establish care at 1 week of age. She is exclusively breastfed. Parents report that the infant eats every 1-3 hours around the clock, has 8 wet diapers each day, and she has surpassed her birth weight. The parents ask how they will know if the baby is getting enough to eat. What is the nurse practitioner’s best response?

A. There is no way to know how much the infant is eating without measuring it in a bottle

B. Look for signs of satiety, like the baby falling asleep while eating

C. 6-8 wet diapers a day and weight gain is an indication of adequate intake

D. Newborns who get enough nutrition will have at least 6 stools per day

A

C. 6-8 wet diapers a day and weight gain is an indication of adequate intake

Infants are expected to gain enough weight to be back to their birth weight by 10-14 days of life. It is unnecessary for parents to express breast milk in order to measure the amount being consumed by infants, if adequate wet diapers and weight gain are seen. Signs of satiety are not reliable for determining milk transfer. Breastfed infants may not have consistent stools every day.

246
Q

During a 12-month well-child visit, the nurse practitioner notes that the child’s first birthday is in two days. She is asking if the child will be getting his MMR (measles, mumps, and rubella) and varicella vaccines. What is the best course of action for the nurse practitioner today?

A. Allow the child to receive both vaccinations at this time

B. Administer only the MMR vaccine

C. Administer only the varicella vaccine

D. Do not administer either vaccine

A

D. Do not administer either vaccine

Even though the child is only 2 days away from being 12 months old, these live vaccines are only FDA approved for use in children 12 months and older This is typically because infants have some passive immunity they receive while in utero, so waiting until 12 months of age will help them mount the best immune response to the vaccine and get a better response of IgG antibodies going. There are certain times when MMR is administered prior to 12 months of age, but this usually revolves around exposure and/or travel, and is not the general recommendation for infants with no risk factors.

247
Q

The nurse practitioner is seeing a 26-year-old patient in the office for a wellness check. This patient reports a 5 year history of anorexia, that she received intensive therapy for from ages 12 to 17. She has been maintaining a regular diet and remains in cognitive behavioral therapy once per month. What long-term conditions should the nurse practitioner be most concerned about?

A. Dental caries and depression
B. Osteoporosis and heart disease
C. Osteoporosis and anxiety
D. Depression and anxiety

A

B. Osteoporosis and heart disease

In patients with a history of anorexia, nurse practitioners should be most concerned about heart disease and osteoporosis. Lack of nutritive intake leads to loss of electrolytes and minerals essential for heart and bone health. In addition, female anorexic patients often experience secondary amenorrhea. This amenorrheic state causes a decrease in estrogen and a subsequent decrease in bone mineral density.

248
Q

The nurse practitioner asks the patient to perform the squat and rise exam. Upon exam, it is found that there is a diminished knee jerk. Which of the following vertebrae is the root cause of this sciatica based on this finding?

A. L3
B. L4
C. L5
D. S1

A

B. L4

When determining the root cause of sciatica, if the patient has a positive squat and rise test and has a diminished knee jerk, then L4 is responsible as the root cause of the sciatica. If the patient has numbness or weakness in the big toe with heel walking, L5 is responsible. If S1 is the cause, the patient will have a diminished or absent ankle jerk and weakness while walking on their toes. L3 is not a common vertebrae associated with sciatica.

249
Q

A 43-year-old patient with alcohol dependance presents today with a gout exacerbation. Which of the following medication changes would be inappropriate to initiate during this visit?

A. Decrease the patient’s allopurinol (Zyloprim)

B. Initiate a burst pack of steroids

C. Initiate indomethacin (Indocin)

D. Discontinue his hydrochlorothiazide (Oretic)

A

A. Decrease the patient’s allopurinol (Zyloprim)

Gout exacerbations are commonly treated with potent non-steroidal anti-inflammatory drugs (NSAIDs) such as indomethacin (Indocin), steroids, or colchicine (Colcrys). Allopurinol (Zyloprim) is used as a preventive medication and should not be started, stopped, or altered during an acute gout attack. Thiazide diuretics, such as hydrochlorothiazide (Oretic), can increase uric acid crystals in joint spaces and should be discontinued in patients with gout.

250
Q

Which of the following conditions typically does not show up on imaging tests for several weeks?

A. Metacarpal fracture
B. Ankle sprain
C. Scaphoid fracture
D. Metatarsal fracture

A

C. Scaphoid fracture

A scaphoid fracture typically does not appear on an x-ray until approximately two weeks post-injury. Metatarsal and metacarpal fractures typically appear immediately after the injury on an x-ray. Ankle sprains do not usually require advanced imaging and do not typically show on x-ray.

251
Q

The nurse practitioner is evaluating a patient today with a chief concern of right elbow pain. He reports a burning pain that worsens with activity. Upon assessment, the outer elbow is painful with palpation and there is decreased grip strength. What is the most likely diagnosis?

A. Gout
B. Lateral epicondylitis
C. Medial epicondylitis
D. Olecranon bursitis

A

B. Lateral epicondylitis

Lateral epicondylitis, or tennis elbow, causes pain and inflammation to the tendon on the outer aspect of the elbow. Medial epicondylitis, or golfer’s elbow, causes pain and inflammation on the inner aspect of the elbow. Olecranon bursitis causes more generalized pain and swelling over the entirety of the elbow, while gout will likely cause more redness and warmth over the elbow.

252
Q

A patient presents today for evaluation of her elbow. The nurse practitioner diagnoses her with olecranon bursitis. Which of the following treatment options would be appropriate to discuss with the patient today?

A. Steroid injection
B. Joint aspiration
C. Cephalexin (Keflex)
D. Celecoxib (Celebrex)

A

D. Celecoxib (Celebrex)

Olecranon bursitis, also referred to as student’s elbow, is commonly treated first line with rest, ice, and non-steroidal anti-inflammatory drugs (NSAIDs) such as celecoxib (Celebrex). If the bursitis does not improve with these interventions, a joint aspiration may be performed to check for underlying pathogenic causes, at which time antibiotics may be given. Steroid injections are commonly used for bursitis in larger joints such as the shoulders or hips, but rarely in the elbow.

253
Q

A 29-year-old patient presents with complaints of pain and swelling at the base of the thumb. Which of the following assessment tools would be most appropriate for the nurse practitioner to utilize today?

A. Finklestein test
B. Tinel’s sign
C. Froment’s sign
D. Phalen’s sign

A

A. Finklestein test

Pain and swelling at the base of the thumb may be indicative of DeQuervain’s tenosynovitis, which can be assessed for using the Finklestein test. Both Tinel and Phalen’s signs are used to assess for carpal tunnel syndrome, which commonly presents with numbness, tingling, burning and weakness in the hand. Froment’s sign assesses ulnar nerve palsy, which typically affects the fourth and fifth digits rather than the thumb.

254
Q

The nurse practitioner is reviewing dual energy x-ray absorptiometry (DEXA) scan results with a patient. Which of the following medications, when used long term, may have contributed to a result of -2.8?

A. Hydrochlorothiazide (Oretic)
B. Bupropion (Wellbutrin)
C. Dulaglutide (Trulicity)
D. Prednisone (Deltasone)

A

D. Prednisone (Deltasone)

A DEXA scan result of less than -2.5 is indicative of osteoporosis. Long-term use of steroids can put a patient at an increased risk for osteoporosis. Hydrochlorothiazide (Oretic), bupropion (Wellbutrin), and dulaglutide (Trulicity) have all been shown to be beneficial for overall bone health in patients.

255
Q

A patient with a past medical history of gout, asthma, and hypertension presents today with nodes at the distal interphalangeal joint (DIP) and the proximal interphalangeal joint (PIP). They are inquiring about a possible prescription for methotrexate (Otrexup) as they have a friend who uses it for their joint problems and they say it helped immensely. Which would be the most appropriate response from the nurse practitioner?

A. “I will refer you to rheumatology today as this medication is managed by a specialist”

B. “Before starting this medication today, I would like to check your folic acid level as this medication can make you deficient”

C. “This medication is not appropriate for your diagnosis. I recommend taking extra-strength acetaminophen (Tylenol) as needed”

D. “This medication is not appropriate for your diagnosis. I recommend utilizing ibuprofen (Motrin) as needed”

A

C. “This medication is not appropriate for your diagnosis. I recommend taking extra-strength acetaminophen (Tylenol) as needed”

Nodes on the distal interphalangeal joint (DIP), also called Heberden’s nodes, and nodes on the proximal interphalangeal joint (PIP), also called Bouchard’s nodes, can be seen in patients with osteoarthritis. Heberden’s nodes are not seen in patients with rheumatoid arthritis, which is what a drug like methotrexate (Otrexup) would be used for. Methotrexate (Otrexup) is a common disease-modifying antirheumatic drug (DMARD) used to treat rheumatoid arthritis and is not indicated in the treatment of osteoarthritis, which this patient has. Although non-steroidal anti-inflammatory drugs (NSAIDs) are first-line treatment for osteoarthritis, this patient has a history of hypertension and is not a good candidate for non-steroidal anti-inflammatory drugs (NSAIDs), making acetaminophen (Tylenol) more appropriate for pain management as needed.

256
Q

Which diagnostic test would the nurse practitioner likely have ordered to help diagnose pain related to suspected ankylosing spondylitis?

A. Computed Tomography (CT) scan
B. Dual energy x-ray absorptiometry (DEXA) scan
C. Magnetic resonance imaging (MRI)
D. X-ray

A

D. X-ray

A “bamboo spine” is commonly used to describe ankylosing spondylitis, a type of arthritis that affects the back. The first step in diagnosing ankylosing spondylitis is an x-ray. After an x-ray is obtained, the patient will likely need an MRI, but an x-ray is an easy go-to that is cost-effective for the patient making it most appropriate for an initial visit. Obtaining a DEXA scan and CT scan is not indicated for ankylosing spondylitis on a routine basis.

257
Q

A 46-year-old female presents with pain in the third intermetatarsal space. She also reports a persistent burning pain that radiates in the ball of her foot. What is the most likely diagnosis?

A. Metatarsalgia
B. Morton’s neuroma
C. Ganglion cyst
D. Metatarsophalangeal (MTP) joint synovitis

A

B. Morton’s neuroma

Morton’s neuroma occur in the third intermetatarsal space, or between the third and fourth toes. Patient’s commonly report symptoms of burning pain, numbness and tingling in the toes, or the feeling of walking on a pebble. Metatarsalgia commonly presents as plantar foot pain across all of the metatarsal heads. A ganglion cyst occurs over a joint rather than in the interspace. Lastly, metatarsophalangeal (MTP) joint synovitis presents as pain and swelling on both the dorsal and plantar aspect of the foot, most commonly under the second toe.

258
Q

The nurse practitioner is participating in a research study group. She is examining data from multiple studies all from the same subject. Which of the following types of research is this?

A. Randomized control trials
B. Editorials
C. Case studies
D. Meta-analysis

A

D. Meta-analysis

Meta-analysis or systematic reviews are the strongest level of evidence in research. These are the reviews that put together multiple studies that have already been done. The second category is all the “c’s” like case studies, cohorts, and cross studies, and finally editorials or opinion pieces are the weakest level of evidence.

259
Q

A 42-year-old male arrives at the clinic today for his annual physical exam. He reports being a pack per day smoker for 20 years, and drinks five to six glasses of beer per night. His father was recently diagnosed with cirrhosis and he is interested in quitting these behaviors as he does not want to become ill like his father. This situation describes which of the following?

A. Stages of change model
B. Health Belief model
C. Lewin’s change model
D. Transitional care model

A

B. Health Belief model

The Health Belief model is a model in which a patient’s behaviors depend on a variety of factors, but especially their perceived susceptibility. This model is the correct answer because this patient is only changing his behaviors because he personally feels that his health is at risk.

260
Q

What is the term for when a nurse practitioner treats all patients equally?

A. Justice
B. Nonmaleficence
C. Beneficence
D. Autonomy

A

A. Justice

261
Q

A 68-year-old patient had a fall with subsequent fracture. Upon returning home from a rehab facility, the patient is now requiring a wheelchair in order to be mobile. What part of Medicare covers this?

A. Medicare A
B. Medicare B
C. Medicare C
D. Medicare D

A

B. Medicare B

This portion covers: second opinions, dialysis, outpatient appointments, and durable medical equipment. Part A covers inpatient as well as hospice care. Medicare part C includes advantage plans like dental and vision, and Medicare part D covers prescription drugs.

262
Q

Which act required the nurse practitioner to have a National Provider Identification (NPI) number?

A. The Budget Reconciliation Act
B. The HITECH act
C. The Balanced Budget Act
D. The Budget Balancing Act

A

C. The Balanced Budget Act

263
Q

A patient presents to the clinic to follow up about his recent diagnosis of hypertension. At his last visit he saw the physician, and today he is seeing the nurse practitioner to review his home blood pressure log. After further review, the previous plan of care is continued after the nurse practitioner discusses the patient’s log with the patient’s primary care physician, in office. The nurse practitioner bills this visit for 100% reimbursement through the patient’s Medicare plan. Was this an appropriate action?

A. No, as the patient did not need changes made today, so this visit will be billed under the physician only

B. No, the nurse practitioner can only bill at 80% for chronic conditions under Medicare

C. Yes, as the patient was being seen for the same condition as the physician in office had seen the patient for before

D. Yes, as hypertension is considered a chronic condition, so the nurse practitioner can bill at 100%

A

C. Yes, as the patient was being seen for the same condition as the physician in office had seen the patient for before

264
Q

Which of the following scenarios describes an inappropriate collaborative agreement?

A. A rural health department with a medical doctor (MD)

B. An aesthetic dentistry practice with a dentist

C. A homeopathic natural medicine clinic with a doctor of osteopathic medicine (DO)

D. A physical therapy and chiropractic office

A

D. A physical therapy and chiropractic office

265
Q

A patient with COPD is in the office today for his PPSV23 vaccine. The nurse practitioner knows that this is what level of prevention?

A. Primary prevention
B. Secondary prevention
C. Tertiary prevention
D. Quaternary prevention

A

C. Tertiary prevention

Tertiary prevention is treatment once the patient already has the disease process, so when a vaccination is given to a patient with a chronic condition to prevent worse conditions from occurring.

266
Q

A 31-year-old patient who has been single is now engaged to get married. This describes which of the following Erikson’s stages of psychosocial development?

A. Identity vs. Role Confusion
B. Trust vs. Mistrust
C. Intimacy vs. Isolation
D. Generativity vs. Stagnation

A

C. Intimacy vs. Isolation

267
Q

The nurse practitioner is assessing an 83-year-old patient who lives at home alone and takes care of herself with no help from others. When discussing options about her health care, the nurse practitioner allows the patient to make decisions for herself as she is still at capacity to do so. What term best describes this?

A. Nonmaleficence
B. Veracity
C. Autonomy
D. Fidelity

A

C. Autonomy

268
Q

The nurse practitioner is reviewing the medication list of a 48-year-old male with hypertension. He states, “I started taking a new supplement over-the-counter that is supposed to protect my heart, but I don’t remember the name of it.” The nurse practitioner suspects the patient is taking which of the following supplements?

A. St. John’s wort
B. Coenzyme Q10 (CoQ10)
C. Valerian root
D. Ginkgo

A

B. Coenzyme Q10 (CoQ10)

Research has shown that CoQ10 may have significant benefits for patients with cardiovascular disease (CVD), including lowering blood pressure, reducing the risk for repeat cardiovascular events, improving outcomes in heart failure patients, and helping mitigate side effects of statins.

269
Q

A patient is prescribed warfarin (Coumadin) due to a mechanical heart valve. He asks the nurse practitioner if he can take St. John’s wort to help with his depression. Which of the following statements by the nurse practitioner is correct?

A. “Taking St. John’s wort with warfarin (Coumadin) will not affect your INR. Just be sure to separate them by two hours.”

B. “Taking St. John’s wort with warfarin (Coumadin) will lower your INR and increase your risk of clots.”

C. “St. John’s wort and warfarin (Coumadin) can safely be taken together.”

D. “Taking St. John’s wort with warfarin (Coumadin) may increase your INR and increase your risk of bleeding.”

A

B. “Taking St. John’s wort with warfarin (Coumadin) will lower your INR and increase your risk of clots.”

Co-administration of St. John wort and warfarin (Coumadin) is not recommended. St. John’s wort, when taken with warfarin (Coumadin), may increase the risk of clots as it acts as an inducer in the CYP450 enzyme system, causing warfarin (Coumadin) to be cleared from a patient’s system faster.

270
Q

A 78-year-old male presents with complaints of memory loss and difficulty concentrating. The patient asks the nurse practitioner if there are any over the counter supplements he can take to improve his memory and cognition. What supplement might be a good option for this patient?

A. Kava kava
B. Gingko
C. Valerian root
D. Echinacea

A

B. Gingko

271
Q

An adolescent presents to the clinic with large bullae in between his fingers, and you diagnose him with bullous impetigo. Which bacteria is the likely cause?

A. Pseudomonas aeruginosa
B. Staphylococcus aureus
C. Corynebacterium minutissimum
D. Escherichia coli

A

B. Staphylococcus aureus

Impetigo, especially bullous impetigo, is most commonly caused by Staphylococcus aureus. Pseudomonas aeruginosa most commonly causes otitis externa. Corynebacterium minutissimum commonly causes erythrasma, a type of skin rash. Escherichia coli is the most common causative organism of urinary tract infections.

272
Q

The nurse practitioner is reviewing the labs of a 52-year-old patient with type 2 diabetes before they come in for their appointment. According to their most recent two lab draws, their Glycosylated Hemoglobin (HgA1C) test results were as follows: 01/21/2022: 6.3% 05/16/2022: 6.1% What would be the appropriate timeframe to recommend this patient follows up for bloodwork going forward?

A. Twice a year

B. Once a year

C. Every 4 months

D. No further testing is needed unless patient develops symptoms

A

A. Twice a year

Per the Centers for Disease Control and Prevention (CDC), patients with diabetes who are over the age of 45 should have their HgA1C checked twice yearly once they are at goal. Goal for diabetic patients is generally agreed upon by various guidelines as <7%.

273
Q

Which of the following signs and symptoms would be present in a patient with viral conjunctivitis?

A. Purulent drainage

B. Bilateral eye symptoms from the onset

C. Lymphadenopathy in the cervical chain

D. Preauricular lymphadenopathy

A

D. Preauricular lymphadenopathy

Viral conjunctivitis, known commonly by the layman’s term “pink eye” typically presents with watery discharge. As well, discharge may begin in one eye, and progress to the other. Preauricular lymphadenopathy is also common with viral conjunctivitis. Purulent drainage is a hallmark symptom of bacterial conjunctivitis, and allergic conjunctivitis often appears bilaterally from the onset, with cervical chain lymphadenopathy.

274
Q

According to AHA guidelines, the nurse practitioner should begin initiating medications for a hypertensive patient when their BP is 130-139/80-89 at what ASCVD cutoff point?

A. 7.50%
B. 10%
C. 15%
D. 20%

A

B. 10%

According to the AHA/ACC Hypertension guidelines, a patient is in Stage 1 Hypertension when the systolic BP is 130-139 mmHg or the diastolic BP is 80-89 mmHg. For patients in Stage 1, it is appropriate to initiate antihypertensives if their ASCVD risk score is 10% or greater.

275
Q

A 31-year-old female patient presented to the clinic today with a cough, fever and chills over the last three days. You diagnose her with a right lower lobe pneumonia. Her past medical history includes anxiety and treatment for a urinary tract infection approximately 3 weeks ago. She has a penicillin allergy. Which antibiotic is appropriate to prescribe for this patient?

A. Azithromycin (Z-pak)

B. Levofloxacin (Levaquin)

C. Erythromycin (Erythrocin) /Amoxicillin Clavulanate (Augmentin)

D. Doxycycline (Vibramycin)

A

B. Levofloxacin (Levaquin)

Due to this patient being treated for a urinary tract infection with antibiotics within the last 90 days, appropriate treatment would be either levofloxacin (Levaquin) or amoxicillin/clavulanate (Augmentin) plus a macrolide antibiotic. However, due to the patient’s penicillin allergy, Augmentin cannot be used, making levofloxacin (Levaquin) the ideal option.

276
Q

Which electrolyte is regulated by the hormone made by the parathyroid gland?

A. Potassium
B. Sodium
C. Calcium
D. Magnesium

A

C. Calcium

The parathyroid is responsible for making parathyroid hormone (PTH) which regulates the amount of calcium and phosphorus in the body. Both calcium and phosphorus are necessary for bone development, as well as play an integral role in muscle and nerve cell function.

277
Q

A 47-year-old male patient presents with vague abdominal complaints, and you order a hepatitis B panel. Here are the results: + IgM, - IgG, + HBsAg, - Anti-HBs. Which of the following is the correct interpretation of these results?

A. Previous vaccination
B. Chronic Hepatitis B infection
C. Acute Hepatitis B infection
D. Not immune, susceptible

A

C. Acute Hepatitis B infection

+IgM can be used as a hallmark sign of an acute infection. When IgM is positive on a hepatitis B panel, this indicates that the patient is acutely infected. Furthermore, the +HBsAg is a marker of the protein on the surface of the hepatitis B virus that is positive when the virus is present, either in acute or chronic hepatitis b. IgG becomes positive when infection is cleared, or there is a chronic infection. Anti-HBs would be positive if the patient is vaccinated, or immune from exposure to hepatitis B.

278
Q

When is it appropriate to do Alpha-Fetoprotein (AFP) testing in pregnant patients to test for possible Down syndrome or neural tubal defects?

A. 10-14 weeks
B. 15-20 weeks
C. 24-28 weeks
D. 35-37 weeks

A

B. 15-20 weeks

Alpha-Fetoprotein is made by a developing fetus’ liver. Levels can be measured in the pregnant patient between weeks 15-20 of gestation. Low AFP levels can be markers of conditions such as Down syndrome, while high AFP levels can be markers of spina bifida, or other open neural tube defects.

279
Q

Patients with polycystic ovarian syndrome (PCOS) may be at a higher risk of developing all of the following except:

A. Insulin resistance
B. Hyperlipidemia
C. Endometrial cancer
D. Breast cancer

A

D. Breast cancer

280
Q

Your middle-aged patient decides to begin lifestyle modifications such as diet and exercise to avoid the risk of going from labs being consistent with pre-diabetes to being consistent with type 2 diabetes. This is an example of which of the following?

A. Lewin’s change model
B. Stages of change
C. Health belief model
D. Family system model

A

C. Health belief model

The health belief model is used for health promotion and disease prevention. According to the health belief model, patients will take positive health actions to avoid perceived negative health outcomes. In this scenario, the patient will modify diet and exercise to avoid the negative outcome of diabetes.

281
Q

A 39-year-old female patient is diagnosed with gastroesophageal reflux disease (GERD) today during her visit. She has no significant past medical history. Which of the following is the most effective medication that can be administered for GERD?

A. Sucralfate (Carafate)
B. Cimetidine (Tagamet)
C. Famotidine (Pepcid)
D. Omeprazole (Prilosec)

A

D. Omeprazole (Prilosec)

Histamine H2 Antagonists, or H2 blockers, like cimetidine (Tagamet) and famotidine (Pepcid), as well as proton pump inhibitors like omeprazole (Prilosec) can be useful in the treatment of GERD. However, proton pump inhibitors yield greater acid suppression than H2 blockers, making them the most effective medication that can be administered for GERD. Sucralfate (Carafate), while an antacid, is not typically used in the treatment of GERD, and is instead used in the treatment of both gastric and duodenal ulcers.

282
Q

Which of the following tests would be the most helpful in diagnosing a patient with a meniscal injury?

A. Empty can test
B. Anterior drawer test
C. Lachman test
D. Apley grind test

A

D. Apley grind test

The Apley Grind test can be utilized to assess for meniscus injuries. The empty can test is used to assess the supraspinatus muscle in the rotator cuff. Both the anterior drawer and Lachman test can be used to assess for anterior cruciate ligament injuries.

283
Q

A 6-month-old infant presents with a two week history of hacking, non-productive coughing paroxysms that are sometimes followed by an inspiratory “whoop” sound. Per the parent’s history, during coughing paroxysms, the infant sometimes vomits. On examination, the nurse practitioner notes rhinorrhea and a low-grade fever. Which of the following is the most appropriate teaching point to give the patient’s parents regarding the likely treatment plan for this patient.

A. Since your child’s condition is viral, there is no medication that needs to be prescribed

B. A dose of oral steroids, such as dexamethasone, will help reduce the swelling in your child’s airway.

C. Macrolide antibiotics, such as azithromycin, will make your child’s illness less contagious

D. Prescribing your child amoxicillin will improve their symptoms and prevent complications such as rheumatic fever

A

C. Macrolide antibiotics, such as azithromycin, will make your child’s illness less contagious

Pertussis, also known as whooping cough, presents with paroxysmal coughing fits with an inspiratory “whoop.” Other symptoms include mild cold symptoms such as low-grade fever and rhinorrhea. Management of pertussis includes symptomatic management as well as macrolide antibiotics, such as azithromycin, to help prevent contagious spread. Pertussis is bacterial, not viral, in nature, and caused by the bacteria Bordetella pertussis. Oral steroids, such as dexamethasone, are often prescribed to reduce airway swelling in patients with moderate-to-severe croup. Administering amoxicillin for group A beta-hemolytic streptococcus infection, commonly known as strep throat, will improve symptoms as well as prevent complications such as rheumatic fever and glomerulonephritis.

284
Q

An 83-year-old male presents complaining of difficulty with hearing. He says that in crowded restaurants it is difficult to hear any other person speaking to him. As well, he finds it difficult to hear his wife speaking, but can hear and understand his adult son more easily. He is very concerned and wants to know what could possibly be the cause. The nurse practitioner knows which of the following interventions is a treatment option for this patient given the probable diagnosis?

A. Labyrinthectomy

B. Hearing aids

C. Mastoidectomy

D. Neomycin/polymyxin B/hydrocortisone otic solution

A

B. Hearing aids

Presbycusis is normal age-related hearing loss, which typically includes loss of high pitched sounds and frequencies. With presbycusis, women’s voices are more difficult to hear than men’s, and patients may have trouble hearing when there is background noise. For these patients, hearing aids may be helpful.